Вы находитесь на странице: 1из 34

PRINCIPIOS

DE CARTO,GRAFiA

CROMOSOMICA EN

LOS EUCARIOTAS'

~

Unlve,f"Sld.a1j

... dEl]V.allEil

DEPARTAMENTO DEBI.OLOGIA

Idees iundementeles

Dn" ge nes ~ i'hnLUU~ cerca !LLnO del. ULTO en cl rn i smo pur de cromosomas bmnulogo~ no mucsrrrnn ~egregado:n i ndepefid1.erJt.e duraetc 111 meiosis.

LI~ recombinaeion da 'Iugar !I. g~!l{)ti pos C(!!] !uN~" as wlJlltriJ1l1~m:ws de 'In.s alelos parentales,

IL~ cmrnesemus. hD1ifl(j:~ng()" pueden iutercambiar parte de 811 materialen un procese dentlm:in~Jo entrecruzaeuenrc.

Ln fecumb.i Iladon S~ _pmdll.{~ pOT N~~grega~'16Il inde:pendieme 11 per crnlrccHI1':I!:;[1'Iellt(),

L.n~ loci (.ln~81't:':q q ~~ l'IC upan Los ,genes en UII cmmesoma sc pueden ~<lm gi:~H~w n:n,{;1icIlUo Itl,~ fr~~~'LMlci.ag;d'e ,I n~ r:ccr5mbill111l:te~ rrodw::-irlo& PQr eeueemzaml ento.

L~sdiliilijtHjH!i- ~n~ loci ~)Jh(cniCl:a~ de. lQ~ valores die ia» ,[recut"netw; de reeombinaci 61~ S()I~' apl{)<x lin2ldamenle i!id~ t;j\fas.

I r ~1t~@C 110 de emreeru zarnientu puedo t1fi:c;tl!f a que IIe,l1ga, :I M,gar LI u b'tg!)mJ!J clji]tmcl~lrn.£i1'tq e n 'una l."eg~tiH a4Yl1JC~lI'It~.

Mapasdel ~roITIo§O rna 1 y del erom osoma Xfl! !jman05. r':~~'f0:.idhne~,~k: l'o~gl1:i1GS ~a 1,1aD ~"Trlgu~(Jo mediff[l,te vm'jru; t.ecttica:~. ;i]:p.IlIJa~ rl~ h'\i t: IliIJt~::n(~ dC~Lri OCjj '60 cetc capf[l1i>o y cl oS1.gnien!e, fli'IDadO dcl J,~~J'MI·~f Jof7H ikrw,rr.[J, 19Q;2.)

H t'"jn0~ !:!.l>l~Lblc:cudio va Ios fundaareuios bfLsicos dt::ia scgrcgadon y I a disrd~md6u ik Jus il1elm~, y la rel~ci(jn elMr~ estos procesos y !::~ componamlento de los cromo~ornas du rante la meiosis. ASI" en un c ruzamieem Aia ; Bib X Ail.' ; 111/7, t!spcranmN una PI'(lP0f1d6.n de fenetipos 9:3:3: i _ Come uprendimos de I01( esrudlos de Bridges sobre la .no di~rLHlcH\n (pag, (6), las excepciones a Jas predieclones mendelianas (A.H::den dsrigir la .atcncion del ex peri menrad ur hscia nue IH.lS dcscubrimientos. Fue prec i sarneute una de esias cxccpciones, {!ib~el'vada en 18 progenie- deLi n cruzarni entn di M:brkto, I i~ que surn i nj str6 la clave de lu~ importantes conce pws ii L1.C estudiurcJ.n()~ ell este cap-lmi:n_

(0 ROLAR-f 0 " , , .

En el anal'isis genmi((l. ~ .. s elt:cep.cio1'"les·a Ic~,

corn pnrtcnni'Elfltos priiivistos s:uelen Sl!'r el origl!l'I de n U!l'\!,il'5 @ imPorta~5 iJ;ieas.

___ .... ~..ojIli-.Pl'~.1Ii1.~~ ....... ~;~ __ "'"i, •• _ ...

El descubruniento del Iigamiento

.'\ i Ilk i l)~ ~Je I~· p1'~.fner.a tlfnLclr~ dclsi gl.c) xx \-Yi II i am 8alesun y R C. Pu IHlen estaban anal izandol,a here ucla ell ~na variedad de gui sante. ES,ll!dl uban dn" genes: II no ~j,rrc tlf:~~Laba al t:o~o!" del a flor CP, piirpura, _'I p. l'oj>O) y otro que ::.Ex:~aba. II I~ fnrm,~ de lo~ gr~1105 tie polen (t.,aJargado, y I. n.·.(kmilu!_ Cm7~r('Jn Hne.l11i, purns Pit? ' I lt, ((H~rp[,UL, alarg~do) x pip .. l/f'(roJo, redondo}, y aujQllcc~mda£ULI la F I hctemcigork:a para olb~.~rlc:::r h~Fl' El Cuadro 5-1 rm It~Stm las proporciones de cada tcnotipo en las pkl1lr<ll!> I-'"z'

Los fenotipos de In F 1 sc dibRvi.aron II,~ [HaG \! amerue de la pror(ll'ci6~j 9; 3:3: J esperada .. i, Que esraba ccurriendo'' E~t,[) TlO parecfa tjuc pudieraexplicarse ('lJIHu una rnedificacton de las prOplJfciones ll1f';n()eJJ<lIl~~, Observeque d[)~ de las clases l'ew)tipi~'lI:-; ~,f!!li mas aibwl.JatllC,s. de lo esperado: los fe'J~otiro.);, ~p(LIpUll1, alar~[~Ju" Y "TOjO,. redondo», Como posibl,L; L:XP lieaci 6n, lR <lrre"O!l Y Punncrt propu sieron (111 e In!,'! prudlld~lTI:a~ gametos r . L }' p , I t.le ]()~ '::f.l perados de Ia ~egrcgackln indere~diellte mendel iaua, Puesto II Lie "!.:·~Lo~ geu, ~[i:pCl~ eran !OiS mismos de los gamel().~ de I n~ lTnt:a~ rlJra~ originak5. 1o~ h~ vcstjg~d(]re~ Jlelf] s~mn qu~ nJ.g'Una for:rna de awplamhm1o. Lbi:eo entre lo~~leJos (loru:im:m~bI; P y L_ Y entl't log tllelos rt)!)t'f>iVOR l) }' t. ~.e~ in~pedil1l segregar de

n •• \ D RO _';- I Fe i'l0t~pos:'de 'gu isam~s El n 11>1 F ~ ~~'l,l:1 (Jidda

f.I[]lt eat~n 'Ij run n Btt

~s'Pera do~ dl2 1'<1 n,rPRiOoci'QfII 9:~;3;1

4!&;}J a~ ~~p 13J8. ~g~

~t)"I) 1$(J~ 13m. 435

............._

6tj~~

m~llem i ndepe nd iente en ]21. F.. No ~ahfall; R ln e III barge, ru ill pDd.i11 SCr la m:lhml!aza de este il[CClpl2u:l:lie:rllO,

L~. c(mfinnacion de J.1 hjp6~1\ de Batc~t!\tl y Punnett tU~'(loiJtI1 cs:venrr al e.~Ul!b~e.c:~rlliemo de DmsDph1'It1 como h errarn 1,enm £, .. netic-a, De~pll~5 de que se p;r.opll~lerll per prj ruera ,'e"L In idea cl\, acuplatilllicntc)_ Thomas Hunr Morg;:::ID~:n<Jon[n) un case sinl:i'lar de desviaci 6n del a seg Ulld.ll ley de Mendel cuandn esttall aba dcr genes auWs6ti1iw~. de D"o;~()rhih Uno de esros genes m'ect~ba ttl color J~ los oj~Xi (pr, PUI'p'uJ'a, y W"', roj ()} 1 el otro abo hlllgutU~ tid ala (l'g, ve~rjgi'll. y l.g~, normal). En ambos l,'"Cnes, los aiel,); ~i ~ vesrres son tos dbmil.l HIIIL:-~.MnTg;m cruz6m~ pdpr' ~gl'~~ con mL1is.c:a~ Plot/pI" . vg livS y, p(lsr~riorrt:l(!.rM.rc1l1iz(j, un ctiJrli mieruo de prueba col1138 hembras u{ibk hcternc'igmkas: pt 'Iw' !'g 1/J.Jg 'y' x !Wi{H' Vg!lig 3,

La .lJli]i":HL;it)1l del CTIl7.~m~t!.nto de prueba es de grail imp oHlin cia. 'PLl.esto que un parental (d de pHltba) cnntribuyecon g~J.ne-tos Qll'e contienen liluicamcnLc aiel (l~ recesivos, I os fenolipos ~ It! descendeuci iii. rcvclan I,~ o(lilni l:n~d6JJ de ~l~du~ en l.us. gl:lL'mtDl del otro parental doble 11~rerocig6l]c(). Espor ello que el ~ln'f:;IJ, gaciol" r~.ede con centrarse eli cl ~ntHi~i\ de l u melosi s de tall sol. uno .-I e I O~ 1:.M.fer.ttab 'f 01 vi darse de la del 0'(:.1"0. E!;rocQ!lU:<t.S~ con el aml11~i~~ de h progenle de Ia autopolinizacioa dela f~ donde hay qllJe rene ren ClJeW~1. dos ~ipo~ de mel osis: LlliI~. (Ie parental masculine yolra del lernenino, Losresultados de M'lI' gan, donde los atdos arut;lld()~ por I a hembra de h:~ F, d ct(::nJIi· nan 1m rennH~"}IJ~ de I~ descend e;1ci~ del cruzamicuto depmah:;' fueron 1(* $ i guieutes:

rr~ . ~K ,Plc-l'g W· - L'g pJ'. ~'J?:I.

J339 .1195 1.51 154 2839

Dhv'a.llle~le,e;s:[()s n~imerm se desv.fan espectaculannenre ,d~ prediccion mendel iana de un R proporclon I: 1 : L 1, t inuICllll iii aG{~plmll iento de gel'leb. Las dos clases In:i~ numerosas, pI" . ~~ y l~r - !.'g~ son ~'1·~ ({'I III bin ac:iolle~intmd~c i das ori gi llill~ I ~leilte I!U 1M mOSC!lS homccigoticas parentales. Como puede ob~n.ll1: d cruzarnicnto de pruebn c 1~1'j Ci.c~ IH ~il~at!i6.n, Rc ~··el,a d.iroolt mente la~ enrlli'lillaci,ones ,Lldic:a& J:~ lUi; gamewlo; de I1fjO ~ lit' ~exm de la !PI> de fumiI1 (~(Ie _~I; IDll£stra con daridadeJ ~c~l~ miento q ue EllI'.~:~lJn )' P~lHllett pudteroll Tall. s610 .iuJ~.lif [[,1' l ,<H.H,upol.i nl:£;!ld6n de 111. F!, A~enHb, e1 cnIZ~lil]emU U£' pru~1Io n::vl':l1a a~gn nlleVQ: 11(1 ~olQ &e obs{'Tc'.!a una. pr()l?olri6n apl'Cl:tfIf.. ,d;!l, ~: 'I entre 'lOll dos I.l.po~ pa.rentli.l es. 'i.i!1O t.1.IHhien el1tf~ I~ ~l~ r:ipo<, nO p~(c:rUHks.

VCalTIOS a!h.or'd! ~oque ~ plted;; aprr.mck:r I"!::pitiendn los qrn mlen10N, p.el'O C{lf! eL ca:mhio i.k: hl~, cornhjTlac'i'[]Tle.~ di::. aJeJos rre\fl(. te" en 1015 gamcloo dc 1m homoci gmos parent<'il 1'.5 riel pruner Ct1.).;. llli.~:mlo, Ell C~~ ~i ttlacioW!, c11da p,'lrei1~;al em .horr.LlJcigotiw p~~ _ alclu d~)mi n aI~t~ y otN'I ret3es'l VO, Al i gUal que ':11 el C1I30 a:n(~ :

I as hemi;N'8 s de ,18 FI !i<t:: sumeti.c:ro.ll a u:n ~lu7.:.amie)"110 dt:" p[JJ~

P pI' ipr! , I'X1vX x ['r!p,- , vg~/t,!f,-

l

F, pr'1w ' 1.'"(? .Jvf.{

pr i/pr . V~+/L<M 'i x pFlp'r ''I!'g!lI[; 0'

p~ vq p'r' "'g'
F' X
pr 119 pt" vg \

I

pr ~'g
F ,pa ~
,
Pl" v,&l fi~ura 5·1. Herencia 5 ill! pj~ ~ dos f'!I"~j;IS d;: dIdos. ~'IIl;T(la" ;: 11 el 1~ll r1, cromosomlcn

prl' , ~'g+ pr i vg prot - 1".'1: tv: ~'i<+

~57 146 %'l 1067

233.'i

;A:';i1l""~. estes resultados no se accrcan siquiera a una proper liFiI1I[I~mle1i:ulii I ~ I:~: I, ,,,"hum. sin embargo. las clases 1mb nuf1l!~.I,a. .. son lal> que reciben uno u otro alclo dnutinenrc de Ia rr:l1r~, en Lwgal' de till;!; s:Jeh1~' druninan res 0 dos reccstvos, como til ~I ~aNO anresior, No obstante, hay Gillie lkl>II.Hcar Lilla. vel. mil:;; flJf 1i1~ ~nM1jJW.ciones de alc:los que I ~L~ hemhras de 1<1 F I recit,:cm rle :'11:-' parentales con stirn yen I as c I ases m:is Jrceuenres eft: uec<;;;emlr.1<l1cia del eruzarnieruo oLl@pnliebn-. Ell SlL rrrahnjo pioncOI'fl'JI~ cl acoplamiento, l3ak~on y 1'1lImeU acu'ilarolll ellirmino fll]~Wm paradescribir ~'~~i siruaclon, ya que les [J,NTCl.'io que, " ..... 1~ Law, ] D~ i1lellj~ dom i It:lHI;eS de ann bos gent:s se «repe I i~.tl» :In {! - ]~1 contrario alo que ocurna en e I acoplam ie IUD, donde i:nek\o dominarues parecfan estar ,·<pegados»-. i;Cll~1 cs Ia ~\r.h.~·;.'jo"n INmLlns fenom'ljno~ de acoplamieum Y repulsion?

~1'l~Qii ~ugiri6 qlK! ~os genes jmp~icartCl" en ambos fenomet~~IJb:m ~jnlfldu~ sobre d mismo IN]" d{! crmno,Witlm twme,'ogi/,.\, De esta manera, cuando PI' Y ).",1{ ~L: transfl ereu desde un ~1I):U13f, ,e encueurran Iisicamente localizados eu.el rnismo ere-

I

pr II vg' I

I

Lug-ar de ,efi.tf\lj~.NJZ8 [mt8 nto

, \

'.. V~I

,1(

~umo~it.:l~ pare ntalaa

Meiosis

:ig u ra 5-2. l:.m:n:c-nmtm il'.llLIJ UlIrnmlc la r"elU~1 s. Un i ildH' i;i'Utl recibe [JiI1,\'l!:o..:f1l~ WrntJlOl{n iI", ~,W'~ 'Pmg~n'iu).j, LJ tntercamh.o d. purses de lOfi grf;:~)l!ilt;F.1r r:!1T(i~a.:~uwmienr(J i"')~~ j;enl':ir&r ":::I\JtnI,.lSU:ni!.;l!:i J.!:am(~ri{:Q ...

. .c,l" 'mhhudulLitj; ii: 91~1~ dlfkl:'~ d~ la, '~<.'l11bmoc'iol!{:,<' p;!o.!1t.utw~s.

mosoma, mientras que pr: ;' v g. se trml'>.IIri~c[il en el CfOl.nosoma llOmo~ugu proeedente del otra lJaret:il~t.I (Mg, 5- 0, : n .... llipotesls explica l.amhie:n]a repwKieJIDI, En ~.e case, UI) Cyo.mosama 1l"211\~.iDmllkva pr y IIg- Y c:1 otrn pr" y "'g. POI' 'In tanto, ]a repll'lsWn I10 es m ~.~ que 0 tro casn de accplamiento: en e~ l::l. xituacjon, cl ale I (I deminantc de un gel'l ;;:fihi <[{'olillado lllI alclo recesivo dd etro gen. L~ hi¢t~i ~ ex plica por q!Je tas cu.mbil~aGione.fl de alelos de lo~ p:J!l"entales permanecen 'll1ll~ls. No obstante, i,como explicar la apari ~j6[1. de: ] a~ combinaci m~es no parentales?

MOTb'3ll propuso ql.l~, cuando 10:; cromosomas I'mm6logo5. i'IV<1- [1.;~1l1 durante Ia meiosis, se produce ocasioualmense un intercernhlo de partes de esos erornosomas en un preceso deueminado CIltrccrusamlente. La Fig(,il"ll. 5-2 Ilustra este intorcarnhio fil;.uco de segmcntos crcmesomlcos. Las do!> combjllaci(me~ nuevas reciben I a denorumaciou de ptlnducios del en trllt:lt'l17.oWLi:e.ofo ..

Esta lJiporesfs puedc parecer un pom rebuscada . .:,.Exisle algun proccse observable cito logicamcme que permi ta ex p I icar la cxisrencin del entrecruzamienjo? Vimes ell el C~.pfhdo 3 ;que durante la meiosw, cuando lo~ cromcsomas homologos duplicados aparean entre si, a met1I1lU;U Jus c·mmtit,id.i& .mJ hermanas apa rcccn entrecruzadas, tal y como se representa en la Figura 5-J_ ReCUeidL: que la estructuru cruejforme rnlIlt:lJm~ recibe el nombre de" quiasne». Para Morgan, la existencia de Jos quiasmas era la (:lJrrml:l()raL~'OJ1 vi sual del C091~cP.t() de entrecruearrriento. rr~;rng.llsc ell enema t]ue lnsquiasmas pareeen itidicaT que el entrccruz;a111letllO ucnc Jugar entre nmmhidas y [[0 entre crosnesomas no du p I ic ado;;. Volverenios 1:1 t.OIlt'III' es te p unto mas tarde). Obse rve qut' Morgan 110 Ueg6 a esta intcrprer.:l!Cj6n particrtdo dela nada, sino qlle buscaha una expJit'aci(~tl.lfsic(1 pam, SUi'\; resultados fVneth·os, S u ex itn. en relucicner los r esultudos de los cruzam ie.l]n'08 gt:n~Lico~ C.()H los leuomenos citok)gicm; le da allil m~ eiua:;is a la imporrancia de Ill. teona c]"nlil1C1$f!mka de la hereucia como UIlA herrasnienta poderusa de investigacion.

COIRO'LARIO " - , . - •...

Los qUiasiFh!lS son las ,lUI f'i Ife~t;a do rras '11Mb r~:i de ios en1:rt!'uuz;! mientos.

D,~tQ5 como In~ preseutudos, en los que se rnuesrra cl acnrlamlento >' la repulsion ell los CL'llL .. amientos de prueba y en laS" 'mJl:op{}lill"i'-'l':I(lnt'~ de [a F" se obsen v an con trecuencia ell Itl~ estudlos ge:ne.'lio[;()~. Resultados de este tiro represerrran una obVI!II nesviacidn de 1<1 !'cgtcgaci:6n iudepcndieme .. De hecho, tales excepcienes cousthaycn un ru1al.iido hnportante a la Yd~'01l de Mendel del mundn de la Gcnctica.

Fi gura. 5-3, R:qlrcSCI1I.KlUn sElnp'lifi(:ii>:ia U~ II)l; (J~ia:;;m3~, (I~I runte Jd 'I!~i lJSI~. t:adu HJI'~;~ ~].'re;e1L1a uma oCm·] !;1liJIL tie UIII8 Pi' e iii (;rorilQsorniea 1~li 'l[I~p~ts.

144

COROLARIO " .

'Cos genes situados terca una del otre ~ ef mi5mO par de crblnOSbmas l1om6logos, no segl1!!gan lflqepeJXIientemente. _L

I d' ,itua{"ion ell la 'qUI;; d'u,~ brcne~ res iden t~TI cl m i ~mll pat de L'I;OIIH)~(}maFl homologos rccibe 1<1 denominacjen de ligall!ut'liltu. Dos gene;) ,1()cali.i.aJu~ ell el rnisrno par de cromosomas bomol n ... gos decimos (jue e!-.r::Jn tiMados. Es proplo lambicn refenrse <.11 Hgiliuicnw de alelos espccificos: POI' e;ie.mp1~). en uu individuo A.la . Bib, A pL)drlfi estar ligndo a b. de lonna que u estaria uecesariamente Iigado a R f ~t0S te.nrLl'1]()s aluden grAhL:-r-Lmente ,I 101 existeucia de una emidad Ihica que IjJ?,oil los genes, esro t~~. ('1. prupiu CTOmOS()ma. Cualquiera podriu prcguntarse por tjJHC uos rclerirnos (.J los ,gencs como "'ljg"ll..Io~" ell III gal' de: «acopl ados»:

I u res puesta es q lJ!C 1 (I.~ rem~inu~ .tLCDpJ am ientu yrcpuls il\11 se '~IIiplean en ill actualidad pm-a indlcar dos tipos diterenres de situacienes de tlgamiernc ;:u un doble heterocigoto, a sl~bt:r;

Fase (Ie ~copi~.un ie-nl{) pr v«
pr' 1JM ....
h'lst' de repulsirin }Jr i:..I{~
pt" V;l1 Ell otras p,~[atml~, cl acoplamientu hace menciou ill Ilg'amiend'O entre d01; alelos domlnantes Ij (1m, receslvos, nueruras que til ITpulsivn indica que los alelos domiuautes C~lJll ligados a los alelos rcccsivos. Para averiguae si un doble heterccigoto est:; ell [asc de a';(lfll .. uiiento 0 en ra de repubiolll, i:J.1Y Clue tener en cuenta los genou pos tie suo; pareo tales (') rea 1 i Z;'Lr (.;UIl tl un c ruxamientn de prucba.

FIgura 5-5. D..~:;I.'IOO de

I" reC'nll1i1jf13doll ~[I los urgun i~ mos Ji!l'Io:)k[';:~. Oh~.r~e que la H;.:il,." :.~ I~'" Ilrld p;:.l~(, de CS[i~

dm.!,'Tl'Imu. L, m~j')1 It!<ltl<'rl.1 ill! dctcct-!r 11'8" pruJUCLW metal i~II'

I ~~'L>lllhi nuntes en lo~ Cti'gwil ~DJ~~ Impl~)'id>:j; es J. Ii I'lr U IL cruzum iCI1ff> ~n~ un mdivid IJ>I.) It~temd:<:n~n

~' I ~lr1'!. hurncclgoro r'CC(':~ lvo rcruzamicnrn ~ J.lrL!e.b.!J.~.

Gameto

ti pn pa 11fI.nt~1

GamB~Q

tlpo pi:lTel'1f~ I

Gameto rccornbtn .. nte

GRmeta

rE~I~O rnlJln.:mts

FI u{;omt1i n ~ nte

Fi 9 ur", 5-4. lill recomb '11~!I'1 ~", WJL uq LltI1(~ pr..,dlt~1 U~ de J,I meimi~

L'0n ~'''Irl>\i ~i'~H'D~~ Ik akJII, dl~llDI~~ de las pl)2~~nl~~, ~J1o las ~eh.113" h,~pl("d"':qi' ;.i'':1 I!,,! I L'!'!I.U' a.I di "'I....ill. 'Jl~J6ticlJ.

,"

Recombinacion

En e~ au ~Jusi:s gcncti C("I HI OO~[f:1U, la prueba priucipalpara de[tJminar si dos gen.e:;; t!~Hin hgados se basa en iE'1 concepro U(l.:l recombinacl 011. I.a 11'l .... 'Cum bin ac i6~1 se puedc observer ~m u ni!! all" plia variedad de si'lu::Jelones. si bien vamos a defiuirla de mrmento en relacien a III meiosis. L~ l~e>:oIlm hlnaelein meiotk.a ~.

2fi

lf1di .... id u u dH rlpo pi'lM·1.,!1

1 nd hmJu(J ~ tiIJ() parenral

IrldlvidLiIl

. 81,;U mDIrn;rw;

!nflividUQ r!l'combin~f!II

1ij~lqu:>e r proc~n meionco 'q lIC du 1 ugar II !H~ p.wu ucto har~oi(1Je ;:."',]lfJ ~elll~ti po (I ifiill\! del u!> dos gel1nl i pOl> haploi des q LIe for-ma'[1111 1& dl ela me16~ h.::a d iploi d.e. :1-<: I f1rij~JLI.(!LO iJJ~i generado se U.: nonuoa reoombillante. El puma uuportanee de esiu ddil11ci6n ~n.'itabIM enla ,£ie[e~ciufl tit la recnmbinaeion rnedlante !R ~~rr'~,m.cion de los genotipos de Jtdi(/(·I tit: la m~i{)Sis COli II)S de 1?I~I:ud~ raT~ntal'es tF:ig. 54}_ Ln~ genotipos deetrrrada s~n I;(j~ j;r... ;;!en"tipus l:i<J!pl[)ides que se comhlnaron para dar lug:kT ;I la l(Jlh-Lluri61l gener.ica del mciocito, Ii! ~6luJlI diploide que ~H.1.irr la'llciul;i8,

(QROi.ARlO .- .•.••...•.••..•••. - - ...•• , "., , •..•

D1a~nt~ 1;1 meiosis. ja recornbinacion g'~n~r .. genQtip.o5 baplCid~ djfellmtil$ de 11$5 genoti~s'p.ar,entale5'

ei.

~~ m'oo)hiuil.{;i{ln meintjc.~ forma parte tanto ue los ciclos de \'l:lahrt~idl:g como de los diploidc»: sln etubargo, lu detecclrin oj: 1(1$ rotom~ill<lll~e'oS en. ]O~ ciclos haploide« e~ mas directu, J:Oi~r;tr.1s que en Ju~ cielos diploLdl~~ requiere mas lrab~jo. Ell io~ cici'.ls 1~r[lJiub, h)~ genf)~ipu~ de entrada y J(' salida von los JII~<lS (.jlr~ ]U~ de los- individuos, de manera l]L1C pueden lrrferir~ II r,ll<III~l11e de Ius Ienotipo», l.a Hgll.lra 54 resume la sirnpli~ir,lu (I. l'llirlete(.;~ici.[J de' I'n~ recnmbinurncs ell 10$ c1dU\.~ de vida JJ~'~'idt~. Los tipnfi de entrada y '-.Ie ~f.Ilida ~Ori JQ~ gametes. Pu~~tn ~IU{i patu detectar Iusrecomhlnanres l"11 un clclo diploiut: cd~ti» conoeer los gametes de en4.r.uj1a, e~ pr~F8r\hle quelos Tifgx.li t(~n~nc<U1 .If:n{'a~ puras, Ad~m::i~; no podem (~S reconoccr de _LI[IY-'! direL'l:.I lmgilroe(ll> recombinantes ljllt: salcn: tenemos I.JIf ,;nlllt'lL:t al 1 ndi vid L!li diploide :;I U n cruzamiento de prueba y .]n-Ji1.:!r~tl I'I'Ogeni,i' fFig. :1-5.1, Si 'L111 dcscendlente Jt: Uri I.:L1.17.a.:JItJ\(1 ~fiTlJilNl -e h a fennado u p.:;1JJ1.i r de uu V.ro.hli: to recombl":nlli~ jl~ 1.~ mdm.is, tambif,n TCC ibe la ~ lenumi.l1tLci("m de rccombi"UI,,'~', [lb;-C.lli'f de u,u t"VO que el cruz, uuiento de prueba penni tc Ill!'IIU~[~nceulrcr[lo~ en 1~1I(J sola meiosis, evlracdo a~1 ambigued:.J.~. En 1~1 Figura 5-5, nor cjt'ill!pl o, _~ t fuera fHt' Lib! c au mfH1 I nnl",ria "I,JlO pLmiamm dlstingulr a un des\~'rldicnle recomblnarue JJA ' filii UC una AlA . RlB _~ln rC;IJi zar cruzamicorov adici unalcs .. r p.~m:.'l)mbi !ltLrrl~~ NC producen uied i7lll te do", pruccsos cclu ~ ares ~~.i:1[,;»7: ¢,gn~gac;lln indept'nJicnte y entrecruzamicnro.

i1.e<omolnacio!'l mediante s~gregad6n i1~ependiente

I", jiWJrii.:5"-b mucstra e] conceptu de rec()~tlbirmd(;n med i ante &.'J~];¥:.un i ndepend i.~nlt:, En LIn cru zamiento de prueba, las :b, ~,:I.'w;:,r~lnhimmw~ representan siernpre el 5U (jl. de la des _en~l~i:(; E!If('l e~.IMY un 25 % de 4,;<lUi.I tipo recombinantc entre I; u~;oa:tl'.d~11ci;L.

~i~nlmHulllmiel'lto de prueba observarnos una frecuencia de n:.\ll'1liiIliUlt~~del 50 %, podemos lnterir que los dn~ genes irnplkailillr ~g~e}liLn independrentementc. 1,::1 lnterprctacion rna" 1I1r.;!le 6~ este reSouhado es LJ ue I n~ dos g{';u:l.c~. sc <:nCUelh'ltlt',;;lFl t:.b! ]Ilf~. u;: l!Wlllflo,5QIlIJS ho;m(,)log'oh dbtiIlLO~_ Sin emhargul genc~ '11t:~lk"'~f1 Ji'o~IGiut1i'~ lej;10ti~ en d mrsmu p;Jr (Ie CWmD~()l1IaS Mb oompurtaT,:.:e- oomo ~i IlU e!'ltLlVierllllligadl.l~ y dar ei mj~,),1 'tS.ult<tOO d~ la ~gITgac i 611 I ncl'tpt'lldicnt8_

-:;:~B

-

-<

)\ _,ij J

,.

x

~,

mprolole meiatlco IF,)

ediV1ti1Utl d~ prueba ~ I tiro parel)tal

D~sGendenr.ia del

cruza rnl anto .ele pru{l1kl

b

1-

4" .

tI

~ ]1iPO parental tI

1 4

1 ;j."

Figura. 5-6. Lt. ~egre~~iM 1I1d"I~'lllh~I]L'e lilwduc~ 5icmJ1F~ uuu

'frcciJ~l;;-b de 'l:e{'U1llhi'Racinm de] ~I) %, iI\te d~iI,I\li3t11~ .n.1J~~[r~ OO~ pares oc <:rIJ!I]U~WIIa~ h<IIIM\lfJg,.,;; Lie un [jf!lani~mrt diI'11-';,j~, ,l'n los :Elkios)1 Y " Fl' 11(10) de 1.':0.;, pares £"1TII1''''''.(''111iw~ y los a1dn, R ~I I, ~D ~L [Jf1I"{). Qb'>CTI.'~ qll~ i"t'ijrf~lII~ haher r~f'!~'~J'lLiiilil J~ ~itu3C i(i.~ el~ IUt urgunism« h3I1,1"id!J eluamnode

"I l ruzumicnro j"I;,[""1 ill (l') ~. ~I de prucha.

Recombi naclon mediante entrecruzarniento

I ~ 1~ recombinantes tarubien pueden aparecer a travcs de I ~11 rrecruzamicnto. Esro orurre entre cualquiera U.t' do .... cmmaridas no hcrmanas (~~ presc ntar:ln prue bas de ell ('I en el Cap. 6). N 0 se rral'fI (II': tILl(, haya IJn enrrecruaamiento entre dos genes e~rocrtiCIJS e n lOda~ la ~ mei osis; pcm cnn ndo 10 hay, como se mnestra en la FiglUll 5-7; Ia mirad de los productos de esa meiosis son recombinantcs, Lax meiosis sin erurecruxamlerno entre dos l("I.el genicos producen on~c.f.lmt'nk gcmotir(ls parentales para los gemilS (:'~~ud i ario$,

Para los genes siruados muy ccrca en. el mismo par crornosomieo, cl hgamiento ff~ii~o de las coinbil1ac:iullt·~ de los alelos ~'dr'~!Il!lks haec imlXlsibJc la segregacioa indepcndiente, ~aildo ~L1.gar, por tnnro, <L Iecneucias de ,(t:iL:'O[9~b"iT'i!lllte,~ slgniflcauvamente menorcs del 50 f¥,; (Fig. 5-8.). Observamos 1111 l·jcm:pln de esra sihlat'.~on en .10<' datos de 1\,.'fnrgafl (pagina '142)_ donde 1:4 lrccucncia de los recornbinantes era (151 + [54): 1839 = 10.7%_ Obvi~meULc. ~:Trc51.1]cado es IU~I,ch(~ 1TI~IIOI' L'ld 50 % que c.~pcmrf<uIlo:,; uc la segregaC'ion 'independieme, La i'r':{>LJcnci:l de I .. :, ft:t::ombi nall re ~ que apattcell pel-!' eiillH.'>(·nl ~Hm i ento e:IUre gr.Il~t. Ii gUfli"lS f1ii-cda dc~JIt 0 11 50 tft, dt:. pcmlicndo df'_ I a <Ii stancia entre lo~ gef.l.c~_ {Que r!Gtlrr-e COIJil la~ frecllenl"ln.~ lit' T~:ombi.lni!lfll~~

Parental

A ~ A
Meiosis sin A B A
enttecruza- - I -
I'fj ierno erutrf.! F,J .b
los g>P.ll~S -,
{J b -
-
Ii e A
.., -
Meiogj~. CiOn A B A
un entrecmza- _. -
m ientD Britr!'! ~
IDS g8rl8~ -
.11 b ~
... - B

Parental

Paronl'il!

Parental

fig u ra 5"7. l.~ reumabilL<l.dhn.~· pru(liJ0~ ~TI .~~l~l]as fi1~;osi~ en I~~qoc.lu! entrecruzumlento tie GrJjIlL~'IIMilb II,' 11~~!nliL13S entre k>;< ~Come;;; en esunlio,

B

___ .

b

B

b

mll},Ol(!: .. <.; del 50 % ';' La respuesra ~sq lie tales frecucncia», como ~e,I't,11KI!-< en el Capflulo 6, mUlal S~ observau.

ULY*r ve ~~1 I aFi gu ra 5·7 ttL1C cada ~·:nttl.~l"i'lizamle!Tw genera dns productos recrprocos, 10 que explicu que las l:recll't~cjJiJS de las clases de wcuHlb~na[l~e.s se.1IH ap.rQximadam.cl.1te iguales,

COROLARIO' •• '.' •• , .•. , , , .. , .•

Ulla frecuend a de re.comb i~ .. t'ltes ~r9nHiGltivi:imente menllr del 51'1" % indlroa q ue 105: genes e'5.cin Ilg.tu;kI5. Una 'he-wen tia d!t! retQ!IrI-b.j na n"fi!5: d@1 50 % sig n ific:..

.genel<l ~mente ~ .... e las; genes no Ktin ligadO!ii 'i se h [lH~n en crpmD~ dM'im45 .

." .. -¥'.'!v.IIL..,. .... ~, ~~-.~ __ ~~.,....

El n;~tu (]I{l (:.$I P il1;ld(l U'a~~. fundamentalrneme tie gem~~ 11 gJdo~ y de. ~1)N recombinames que a[,}£1 recen fI'C' r entreeruzarrecnto,

Sim bolismo dell igamientc

Con ta [ntrodncclnn dd hg:::rmicnL'tl. nuestros ~J.Ubo.~os p<l!lli I~, descri pcion tk los IT'i.1 zamieatos se hacen 'Un ~all;tn .j lJ]c(n:n(Mj(~&, Eu el C ruzam Lento de Dm:wphifa. podemos repl'eo;.e.nt.~y la consti nwi6n gtn8il:a de cada crmnosoma como en el ejcmpJ(1 s:~gui~;nH~:

r« -«

PI' ~'g~

don de t'.atla I f ~]C<I represema l! 11 em mO'<l(lHl a; ~OS" ulelos J~~mL~lI e.~t:i! ~ ell b1H C 1·(KI1(l~omiJ. y lo~ de ab'1j() en el oWn CH'lflwwma ~'lmlL()]ogu, U~J ~mtecru;>;Jmient;() se represtmtacr.~locrmth) U,IHIX CnlTl'~('~ d(J·~ CH.:'JIl080l1illil~" de rnanera que

e~ i.() mi Sn10 que

rr VM

=X=-.

pr' l'M

if '1

-

1 >4

DBS>C8mJ8.nGia d!,!11 Gruz;imiE!fiW U8 pf!.J1:lIM

c

...;1 4

Fii gu ra 5-8. flJ""Oli\li, n~,:iClil1 pm L~I1UDCruJnrnicJ1 to.' Otw;'r~'r' ~I~!~ 1;1 <luna de lus 1't"C(,LI~rut:(l8~ ~!t: lill I"(IWUlbJmllll~"" d'f U no ~~,I,,! iJl,ibriCtl ail 50 %.

P(~di:Tr~u,S ~i:rnp~iJlc!l'r .~a deslgnacion genntJipk,,"1i ~fe TIm, g~r~~ J~t dm dl buj a:ndo una uniealjnea, siendo I.GS genes que estan ~ CIJ~ lado tOI'> 'C'i I re se encuentran en cl rnismo crnmosoma: ahOln n~ W) sinntH)~O es

fH

Non nh~r.~nte. todavta res ulta 1l1cu.modu !::~~rib~[ ,t:~I{J.j) 1n8f1P. c. rnflqlJ f.nia. de mauera que pooemos i n:dln,ar 1:<1 Hne!l! p:.tr1LldbiJ;:l~ IN' 1.' fl,fpr' 1'£ I. man It'llJ'cniJo ailn .I 05 nJdos die un (''riJI1]:n~nm~ ~ ~

iJiI!le h lruea )' Jo~ del hml1oi(}gO ~I otro lado, Sielllpl\e auoraT~·~t::em:~ L'n cl misrno orden a cada uno de los IfIO!]:;, habla T:t sirllJprc de a bla b. m.H1Cti a r~/b a .. r.1I norma de que lo ~~~ escriban ~ lempee en cl rni ~1l10 orde n penuite II .illS gene~Ill. ':1l.1pie.@J' nll"l notacion abrev i srta, en la -l.l ue se representa el J:D~~I. ... estrc soJ.o C~)IJ U n ffl gno mJ:s, En esta anm::tC.i 6n, d geno fI"~Hglpr- 11,1('"10 escrihimos pr vgl+ +. Esta :.u1ma;.;ioll puede ,~ru:';:11 onos I [bros y 811 urnculos de il1vo::s.Ligu~'i'm,

CellO lie 1M visto en ]0:<; C<,) pI[lJlu~ arncriores, J 0<; genes si tua .}6 <~ p<IfEjtIL'>' crorno~(jmi'cil:S dis lJ ntas se sep~!'~ n con un punto }' 1'Li, AllJ ; Wh. por ejcmplo. Ell este llbro. los gC'I'!l~S que 110 ,1eIHf.t~ ~i estan 0 no Iiga,df1S se separan por uu puntn .4ia ' .BIb.

~i ~l'!ahlLi:eT<l.mos de nuevo los n:~"l1[iado,; ohre.t\,icIOt; POI" Bate~)I ~ Pnonett, podernos expl it: ar Iuc i I mente e I j'enUa:I1~no de aco~J:nCtito uri H2:~ ndo el eon cepto de I igamieute. S 1I~ resu I tados ~~[nmple:io~, dcbido aque d'!ll lricieron cl cruzamieutu Jt' prucfl ~ill tUlhlirgo, m,~~ tarde Vt'IXIlW~ que tamhlen se pucdcn cal- 1Ij]' hj~ nu.meros de los lipos recornbi l'!iJmlt'"~ y parcn tal es en un tJ,~llli~nto dihfbrido,

jramiento de genes en el crornosoma X

Ill" ~11lJi:a, hcmos tenldo en cuerua lu reeomhlnaclen entre get! ~J10 . ..urn ices, (', C!itales son I a~ consecuencins de un ennecru,TmlJl de c-romtitJ!t1as no he rrnanss entre d~)~ geues de imeres ~ {li}m{1~1i1;j X7 Recuerde que r-Il lu cspecie humaaa () (:t;l "",\~yIMa.l.as hembras tlenen deseendencla mnseulinu hemici~12:~ para lo~ genes del crorn osn III if X, de Iorma que 1m fenoH~j,tk n~ hi.ios dependen ransolo de ]m gcnotipns de 1(i8 g<llllt' ~~~PQrmd,oo por la madre, Considarenms un ejemp,lo en clque r ~~[0 se (1h'~el"'!ia ita dcsccndenein F I de II n eruzam ienH'! earre ~;q;'l:ISG:t~ Vro:J/(JphiitT)" IDCln 1)(·h'n~l'iorjJ:1d. unaltzamos J" Jl:~t!.r~;mei'l F~ procl!lK"i.da al cnmu cIi!rlt:if los imii\11duo:> de III Fl' fI,~~ ~j'tmplo. urillzaremos l(}~ siguienres ~lllob111ol>; J e V I r't'J,J~~l~l(1~ que determinan cuerpo amarillo l' ClU(:lrpo marren, li:roL~.am~~k; fA; )' lt' para los alclos lJW~ df:'Ee!11'li'!1sn LJj~) t:JICrY flYJ roju; t' Y par::) el crotnosema Y,

p

F,

v '4' 43 meum hi nantes
y' W 2146 parentales
v Lirl'" 2302 r-~ renta I es
l' . w·~ 'I,) recolJlbinanres
45]3 U,,1ll tJlIC los m.achot. de I a F .. nOli ellen de ~os Il:u:l.ch(},~ de 1 a. F I IL.iI1I~TI1e. el ~wm{)\. .. uma y. estm c.l~set. rell!~an II J a pc.rlecJrdt~vmJw;.:I.U~ de la" lueiosi:s 1.Ie la~ Ihemhras j','I' ObS,tll>'C qu.e 1=1ld.Jrnna'l.a necesldad de rcali.zar un cmzam.icmu die pmcba: !\:J:mn, ~egllU' J.ft rJ'Icio~is de uln 010 progenitor, r'le 1111 mlSlIlfL ::ITll qlle Jj) ~J;:Iriamn.']. en Uti lnu:::rmiciltn ,de [m 1~1tm. La Ih::cl1.cncla

147

total de los l'e'wmhlmuIWs en este ejemplo I.:'~ (43 + 221 : 45U = = 1.4%.

Mapas de ] igamientn

La frecuencia de recomhlnantes para lu~ genes "ntm,('lmico~ tic iJroIophiltl que liemos estudiadn (PI' Y Q:J' correspondfa al m.7 'X .. de la dcscendcncia -una frecuencia macho menor que [a de Ius gt'.nc~ ]jgado~ del cromosoma X an~[i7.ad(JS .er~ d Hp~lJWdo ilmkr~ior-. Apnl'etltewcnll.:, hal' diferenclas tJl el nflmcm de entrecruzarnlentos "lUI: se dan entre disti.ntu~ genes ·Ijgad(l~. Clertamente, no hay ["..iZ~l'll J)llrll esperar '~Jue Ias cromtitidas se entrecrucen 'lIYII 111 misrna frecuencia entre diferenres geues Iigados, Conforme Morgan estudi 6 mas gene~, observe. quc I a pm1:KI'b'C i ri 11 de ~a d~'>Cend\::n{' ia rccom h i 11.~ rite v uri aha considerah Ierne. I ~t, dependiendo de que genes ligl:lJu'o estuvicra snalizando, }' pctlSU qlle est!!:;! variackines ell Ia trceueneia de entrecruzamicetos pndfan indicar de alguna mancra 'las, disrancias rcale,s que separaban ~mos genes d~ otrcs ell los Cfl)n:!O.wml:l~, Mnrg;m ot~igJl6 el eSUI.dio 'ue este preblema 8 uno de M~~ c;~l~rn.a'l'lte~, Alfred SrurtcV~JlL.. que (aligual que Bridges) xc eonvertirfa eu nn gran genetista, MmgM'l pidid a.S.tmtevMl, lUI cstudianre de licenciatura pnreetonces, que die I'll aiguII scnado a Ins dams de emrecruzsml ento entre diflln::nres genes Ilg:ldo8. Ell una neche, Sturt.ev·anr desarrollo un mctedn ~lllC lm:La.vi,~~e miJ.i.'ul h.oy para d.e;;;C'r~bjl' las [~:hr(:ionc,~ de l,ig.1- mi ento entre g.fmc::s. C()IJ I a~ p ropias [J'iIlabruK de Sturtevant, «,I fi Il11Jes de I 91 l. en una conversncie» con Ivlorga'll. me ,Ii i ('uen! a de repcnte que las variaciones eli! 1::.1. ma~nJ1l1d delligtl!micnlO, ya mrihu-idas per .\1L1rgan a diferencias en J8I separacien espacial de los genes, Llfrudan. Ta rosibi lidfld de detcrminar ~~ ardell deles genes t~ n Ia di mel] sion lineal de 111l'i. cromosoma, Me fui a 1:.i.1l.<;.a y pa~c tnda 1.1 uoche (en perjuicio de rnis luborcs de estudiante) da.boramlo el primer mapa cromosomico».

MeiosiS T !J II w
1{ J.
z~·


3{


4{

51

sf Fig IJ r,a ,5·9. III u[JI,j'JdoJlilljd~.J ootr~ lJm I!t~r.3TI~ m (:rui1Jl(;r~6;[1J~ y ja.~ 'tb;::m~I\,;k1~ d~ m:mll.bi ma.citHn. 1-1~ L'j'ft~'(,1 'rla:,:>. !rll ~Dlre=zlliA de IlliIJl.;rB ~k"~I\II';' ~. ju i'u['~n del CI'~II "·I.vlil~ J W::,1Dt~ Lw; meio~i~. La di~1 ac,,"''' "I1.!e ~'ill'u Jo"

.!lcn~" ] ~ j! ~II II 0 C'rOmo~!JliIl~ c. mSplt 'ltH." J~ 'I U~ buy Cli auo cr(~mO~'(l11l3 f!r1rr-~ I~~~ g~1Lt:; ~~ Y lL·: L'3:.~ &:!f1Irl ,;jtidil;) .. !! ~nlJr'!:::('l''lIz.an ~.I'! tr·e: T y f f L.:II

Iol.!"L pl'Oporcil'Hl de l~;; IIlei .. ;",L mil~'OJ ql]~ l~ qrn: ~~,i!1 e 1,1,,; V ~ W, (\.1'1Il,", \'erernlJ;T L'Tl~S a..1r.'kJIII'; l'1J ~S[~ (,'21pitUJD'. UA en[rcl:oTI!I'.1.W,i~'J~V p;~<,Je oc~mr entre Lin.;, L1'(~n,)(kjw, ~() :lM:;rmaA~.>L1~id~'qlll~r~,

148

Como cjcmplo de 13 l6gi a que .siglriO Surrtevant, coli£itkrc~ ruos un crnzarrriento de prueba pam el que se obtienen los resullili.jOt~ ~ igl L ientes:

pr 'CgJpr ~'R 'H' ~'g I ip]' ~'g

.165 l parental es
19 L J
23 ,
I rccom bi n antes
2.1 (
400 PIl esre ejemplo, la descendencia represema -tOU g;1metcl~ terneni- 110S, 44 de los cual e~ .( I I '*') sun rccombinantes. Sturtev a m ~ug! d6 tlll~ f'l porcentnje de reconrbinemes ~e: podiu utilizar coruo indicati \'0 cuan l i L..'tti \'0 tk I~ d istaacla enrre dos genes en lin mU:J~,a g'~n.4. rico, (1 m!lpD deli gami~.nlto, eorno scl e denomina a veces,

Aq II L ~a idea b.~ ~iC.1 es bustante sencillu. lmugi IlC JQS genes eoncrctos ~i.t~I~H]U~ <I una cierra d I sranc i ,I 1J,1l{1 del ntro, Ahoru, imagine que Ios cntrccruzamienros . uceden 81 azar 11 10 laJrgfl tic 1(1~ cromosomas homologo~ apareados. En illgllfias de la~ divl~iIJII~~ IIIe10UCfl'>, se produce, de forma alcatrrriu, 1.1[1 cnrrerruzarnicntu entre las clUJllliliLias r~O hermanas eli la re.gi6n crornosomica que separa los des gem .. ~; en estus md~i:;;i,~ se forman 10'\ fr.~~ombi~-I.~lue~_ En t)tr:.v-; meiosis. no se producen eurrecruzaruieruos entre esros genes: ell estas rnei o~i ~ 110 se u btienen recombinantcs. Sturtevant pj'(JPtJ~o que. con UI!l·a cierta apf(H; .. irna .. cion, se cumplia la ~ig)luenlK relacion: cuamo mayor e~ la tlistancia entre genes liga.do~, rnayores Ia. probabilkku] d't: title icngalugar lin crurecruzamiente entre emm;itidilli no hcrmanns dentro de la n:giun cl.Jmprendid.a eIltre los genes y, pM tanto, mayor 1;1 proporcion die reeombinarucs que lie vbt.ienen. A<;f, se punk obtener una medida rl~ la di~tall1cia entre los gc.rlL::.l. ti"lt.diimitc la determinacion d~ la Ireeuencia de recombinanres (Fig, _"i.9) , De 1l€,Ch(J, podernos dcfinir una t1n.idad dc mallil J);cn{:.tico (m.ll.) L:nmo ~ n d i stancla entre kKS genes pura la qut' uno d t: caoa lOU prud udu~ de hi iiido~'i S es recombm ante, Dicho uc otra rnanera,

B

f·-

----~S m.u, -'------

.04

-i3,m_'I,~

una ~{,,lJl'Jlci8 de ~"(Imbi.nilci6n ((U,") de 0.01 (l %oj se LI~int mrno llILU, ~ la unidad lie mapa recibe tambien hi denomlnaciu; de centhnorgan r c]\o1 J, ell ilurLlYr de lhumas J lunt Morgan)_

Una conseeuenoia di recaa de lu limna en kl q LI~~ se m ide. la dl" Lancia de map~l es q ue si los genes A y B estdn l{lzllar~u.lLJ;S put ~ u nidades tic Tn1l:ptl {S m.u.) )' los genes Aye e<:t~n separados Jl(Ir 1 m-'I L, los gefle~ B}' C dcberl ... n cstar a una (]:i.<;t;:Uild~ n:e 8 61~' u (Fi~, 5-10), S('uffewlillt wmproM q,t:'Ie e~l4) em 10 (,1m: SlICr.:.tJi;;], l!: otras palabras, ;,1.1 ,mali sis apoyaba conrundenrementc la idea de g_ Iff;' p;el1e.<: );1: organizun segan alglin tipo de i ,,'clellamie.nIO li[j~

1.:.1 luga I' qlle O(:l1ra un gen ell el mara .. y eu el cromosem re.:ibt: el nornbre ,i.Je Joens IIenk'il' fen plur,al, Ioei]. Por cjCtnp'l e I locus del gcn para t.:1 color de Io~ ojos y e~ del ge!11 para l longifUd de las alas esnin u una JiWmcia de 11 111,1,1, L .. a f'elaJ: ("' sue 10 rcpreseutarse de la s i gn i ente rnanera:

pr 11..0 ~g

-r

,IIJ)

----lUl----~

l.ocus del t-."t' n paw el co lor de ,1(1,~ o] O~

Locu ~ del _g.e 11 para 1 a forma de 11:l~ ala

Nonnnlruente, I'lO~ referimos 31 locus de estc gL-'1l panl el tiol~1 ~ 1m: ojos, de lonna abreviada, como el ",IOCll~ pr!', ql1~ IK!'C ~ mendon ar e 1 nom bre tiel pri mel' alelo no sil vestre aislado, ~i obstaare. con ello i10S estamos refiricndo al lugar en el cromr• rna. donde podremos encontrar cUf;lllqufer alelo at: eslt' gorn

Dada una dl~t~U1L::ia ge~tftica ell llniJade~ d~ mapa, puuemus 11 d~cill' la frecueneia do la~ difcrenlcs clases de Ia descendenciaj t:jelnptn, eerre 1., descendenci a de un cruzarmesto de prueb.n t~

r

Figura 5·10. 'i)nd(I(\'.J:

Iils di~lillKI,,~ J~ 1l1'1!,'" .>"11 ~UlilU. c;!ik'lll,~ de 'IEI~ dJ~nD.::i~~ ,~,JI , ~ -c "". ,j,. I;; L~l1n 1.% do:< ~l!cT'n:J.[i\'~~ puzu LI J:l,tl!><;i.J ~ qo~ a p>rrceL'11 ~, IJl f1g~r~_

.---~--- 13. fTI,ilI, ------

f}

+

_'_- ~ m,L1."--2iT1.tJ'b 1'l'I,U,

1k'IUOC;:l, b::ll'wcjgfilica pr vglpt ~'X·. sabemos qUt~ habni Lin 11 % .k! [t~()lTlblllal1t~. de los que lIll 5,5 (,i ,st;ran til! .10. clase pJ' U!?' 'tiT IUllttn~.5 % dt' I~ pr" vgipr Vf;'; Y entre la descendencia de Uri mmrnitmo a,;:: prueba de Una hcrnbra Ilf~tetDClg6tdca IW }'.K-lJ1r~ I~, UJI,s,1% senipi' l'l!tp!' ~g Y OHO 5,5 C'.-i, ,pr+l!g+/pr vg.

La l'l1DcJll~h'il1 d i recta de I odo t)~W C~ ol] UI.: la "d~~t:m cia >:> en el "l!;'1~ de. 1 i gamielltll se eorrespoodc eon la d i..,tlll1.C'I ,1 fi ic a 1 (I ilrgJ.I J.;:] cmrnosoms y _ ..0.111 real i d a d. es ta tue laidea a la que J.'_;:;JI[111 .r",]llrgmj, 'J S ttl rrev HlIL En lodo c ~L~O.., hay que destacar e 1 -l!"'J1n de qu~ el rrl"'p~ gt:ncti en eonxti ruye un e] emp 1 0 mas de ~m ,h.l <1lliLrtlC La ccnstruida exc 111.<: i vam erne <~ [JiW it del anal isis ~l'Jlfri~.D_ EIITl:lJ11l de ligamiento ~t" podrta nuher ohtenkln lnclu:,c, ,jl:. tl ~M(lriULielllo previo de la cxistencia de In;; eromosom~ ·\;;teMi::.. Jlegatl.~.ls a csle punto de nuesrra pre~etU,[I';'itlH, ILl} rl!rll'llIV~ drcir ~L 1 as ~ distanc i a~ gene ticas» calrul ... dus p( ,rrne-

I~

I

Figura 5·11. lal A.;Illlto ~i"'o::,m'

1,( ll'::,!.i::.r~ Cr.r:·hl'W(I,,'J(r l'/IJI"TJ/111.(.I"i~.rr 't ~~I.!I1J~D~'" ~~"'.<1 .x~rol r.' \I~ UJlI~ ~~llil] .tn(lIil",l~ fUJ 11[0;:010:> & ~ci.~ Md

lIJI~IIn:.C"I~ 1ii~i nrns, i'h) .\iwpiL de

4w ~n' 11,. lv~ seis L-l)::J !?'ira d cnlor

t~ Ii"! I~jt"') IJ~!f.tm.::!d(Hi en (\"')I!Or) y de II(I'~)';; : I i.l.Itl,."1 de I:::: JI).')Jo:CJI. 'I_;I.., lli~~i'I~I-":'"L::o entre ~"',:ci 'f,'!:ll I L"" [I ~l'L>t'.[Id.iI" JL' 1.,,,<:lni.:.l'in. W. B_ Taylor, USD"'.I

y

d t(l de j as f~~c uencias de reco III b i rra cio'l1 represensa n de aJ [!.l[1U1 i111.IWm lrls !.Ii~h[l:DciaN -fl:'>lcns reales ell In~ CEOIllOSOll"lut_ aunqac 10:; am'i.lisi~ citogenencos y moleculares han dcmostrado que las di stanc i a.~ gen ettGa~ ,~~I n, de hecho, aproxi rnadamcmc pmporc i(Jnal es jlI ~a~ dhtanei.as cromosom i'C.a1L _'lJrI obstante, L~O~l"" i·t"~ I~ recalcer lJ U{: la csrructura In i pOle-rica fel m,lp,L d e ilgllllllCnlO) ~. t'labnro teniendo presence UII<I estructuru rnuy real, cl crornosema), all otras palabras, lu [curia cnunosrnuica pmpofciOllo 1<1 base pam d dcsarrolln de la cariograffa por Iigami.t:Hlo,

CQ'ROLARIO ....•••.... " , ••• , .•••••... " ..•..•.•...... ., " •......

La recom binad6n entre' 9 enes Hg<ldos, sintB para C.U'1ografiar fa dlstancle que los separ.a en el (romas-orna. La unidad de mapa (1 ID_u_l!i.e define COma una frecuenrla 'Ii@ rffornbinadim Igual all 1 %.

OJ

--c-w

5.7 2,9

42.

48

5

:3

!El ullali~is que hemos realizado ell 1:1 presentscion queda petfeci.am ente 11111 i strado (Jon ] 00 .. " III 8"pas de: ligamienta en I~' mosca r.m"hrimljJ'ic~ /'wmirri'f,'01'i!U_ P.I estadiuIarvario de este inseeto ---el ~lI~anu- parusita las l1erjdas tic los mamlfero:; ~ causa gra· ves J:ufio~ ul g.l!lado t"'.11 al~ unas pa:rte.~, del I1J lWU,(}, Con fa i nrencion de controlar las poblactones de esre p~lrflSiILo, xe ha propuest11 lin sistema _gI:;IlCt,ico que ha deiuostnul« SI.l exito WII otros j nS.ec~u~" Para. a;]c:mz.ar este ebjcti V{'J, se neees itfl conoccr la gencLica ha~i,t:n del lnsecto. uno de cuyos pmilO:> tundamenrales e:. Ia elilbnl'.xiODi de un mapa de ~QlS crcrnosomas. E~[e nnimst tienc sels pa.n:j"l!01 de cromosomas, wyll cartograffa ge 11elica ya se I'm. iniciudo,

La elaboracion dil: un mapa getletku ~IIT.Lpl.ioD()OI~],kmm por cl ai;o;Iami,eilto y analisis del mayor numero po~ihle de variantes feuutipit'I,I,'. Los genetistas han encomrado dlversas variuntes del L~ Lad i 0 adn Ir(J Jt::: este insccto, I a III osca. tlunm tramn i ndi Yidll{J~ de ,~~i~ colores distintos de Q!OS, todos ellos diferenres del. color !ll~ll'r&IU de Ios ojos de, las I1Ju~ca~ si lvc~tres" como muestra I~. hgunl 5~.Ila. AR·ill1lismo. eneoutrartm Ci11'·O variantes FenotlpJcas de otros caracreres, y d.emo~.tt;:Jron que el total de las once variantes se deb:l.1i d' (,mCe al elos muhl[t~'{'~~, cada uno de I:HI lecus cfOlllUOOmiD() [H,stinto, Cruzaren entre sf lfneas puras de cada LlIlU UC Ios fenotlpos para gl'mlrar FI ctjWIt'I.rkia:s, (rue pn~teriormente :;;e sOlllel~erolJ a cfU:;r.amj'enrO!5· de prueba, U:)S C'J'1I7a'lltientn_" de prueba IT: \'darul'l I a serie de eli utro grlll~o:;; de ligamientlJ que semucstra en 11\ Fig,uf<I 5-] lb. OJ:Ko;.el'Ye qmdUl51,u;J:;] ;'e Y (."Pf' aparccen provislonalmentc lig:3>QOS,fllHlgue .121, frecueneja de recOlllhin<ld6n no es sig111fi,cl.1tivamer.lle mener ,del 50 %,

Un ali~:~bis de ligamie:l'lf,o j,;UIIlU cl precedeme no sir~v(:! para asignar gmpos de Ii gamiento a cromCl~OIIl11a~ conererus: csto delle hacerse rnedi ante Ia u f.ili"l.:.t{, U(:l!;J, de t:ecnic~E de citoglil"ne:tj,Ca. que trataremos en d ClqPltll~{;1 17 .. En el POOSi:-J:Jh: ~j ernplo, mdla.~ Lccnicas l"]lugt'1l6I:icas han p~l'mJi.tiJ(} rclacionar los .grupos J~ ligaruieutoeon ~n~ 'cn:m105()jl:la~ numerado .. ~ fi'revi;aruL.:"~tc, como ;~e muestra en la Figura S·llb_

EI cruzamicnto de [res puntos

Hastu ahnra, hernos estudiado cl ligamr,e.ntf'r en rruzamientns de doblcs i.1etel'odgQlI)S con Gobles homocigotus rcccsivos. F,I slgUliente !~·i.,.el de t'ornplejidlld eb un ~nl~IITLicIdCl emre un lripk beterocigo'Lo )' Hll triple hO]llcl(:.igulo. E:;te tiP<' de cruztL.IIIi!"'~[l'l(}, Ilamadoc.rllZamielltt) de lr~ punh'·~~ illlst_ra oobre cl. t.iprl ('0- mun de e~lrategia 'ljUC ~ uti]i7.[! en e~ amilisi~ de ligam~e.Jltil, Vamo;; a \'1;:£ 1.10:' cj'cmplos, de m1es j.·nl.:t.ami,el!ltos_

1:.:1] primer Lugar. nos cenlmrcmos .en [res geltle~ de Dro,:wphilt~ CH}'L1~ ak:lusno ~Uyest1'es SOIl .~e (sewl.', pel'dida de alt!unas cer· !.Ill:; lOr1ciC;l8), f.·e (ecllinn.'L ~lIrerficie oc Lllar nl,gma~ y l' g (\'e"S~igtal, .. Jas CO![~,a.~). PO'i;Iemos cnll.ar I~)sca~ 'l"rip;le~ homtx':~gO'LlIi~ recesiwl.& SelIC : ec/*~<: . k'g/"'1. j,;unmo~ca!'. fodllve!l;tl'es panl gel1e,,'ar 1;'"1 Lripfc n.eterocigofCl ;,;d~{_'+ . (!("hlc. • vgh!g~, Una vc.z. ob-tenido. ana1izaremoos fa n:o.~:umhinadol1 oomeriendu hcmb:ras rril)le htl't.erocigorw> a cruzamiiemo de pflleba con machos viples llln:lIl1c:igows r.tx:csi vo~_ .A cOlltlllu<lci&1.o. ~e ml!l.e.~trall los re~tilL!I.dno.~ de c~k cru7.am[·enw cleo ·pITI£ba.. La descenJer,K!ill. ~~ indica) con I.os gBnmipos gamflicm que derhanl J~ la~ hcm.hra~ het:erocirg6Li.ca~_

Tencmos ocho tipos de g.mnct.o;<; d i s!·i ntos '1 lit' ,en un a mue,~tr..101 Ifj()_:'i rnoscas, aparcccu en lo~ !l6Im::m~ Si'glL i entes:

Sc ' f'C • I'g 235

se - ec ' .. ~'*. 24.1

sc - er : \'X~ 243

fit: I - f-',;,.01- • ~:'R 2~3

J'(' • ec" . vg 1.2

sc ' , (!r. - Vir' t4

.~r . I?C' • ~'P- 1.4

sc: , ec ' vi§' 16

ioos

l.a forma ~i sternatic<l de .11 nil Lzar estOi~ cruzamieatos consiu, en haccr el calculo de lu;]a~ las p(l:;;ih~es freeucncias de rec0l11t nacidn, Sin enibargu, untes de hacerlo, siernpre rnerece I~ :jXiI lnspeccionar 10); datos para inteniar extraer algnua con~lm,}:t obvia, A primera vista, podemos observar una irnportante ~, >'Iad 611 de 101 p.ropotcion .I:!:.I: I' ~ ~ : 1 : 1: 1, Clue ~edll. la e~perwla'; ne hublera Jigam:iento entre IOf> ires g!..·llCS. Una \'e-7. hlXlKI ",t obsereaclon, ucmsncemos !l Ctlh: ular los valores de las .1'ro(J\k[ cins de ICCUl1I1hillaClun, tomundo carla vez Una pareja de geol" E[i[II]JC.l.l.lndo on 100 loci sc y til" (lguoramos, de 1ll0mentil"" h1CU~ vg), delt'mril1!amos qjUe genotipos ga:medros son ~eciJil}' IlFiifltes para ,K y ec, Puesio 'li ue los h.cter"0~ i gotos 'ej lfimliilll

Oil gam.etos sc • e'~' "f $C~ • ee", ~aI1emos que 'I.l},~ prJdu.ct{J~ rt ~(J\rnhinantes de lamciosis dehen ser sc . ec" y sr: . ec. &: lisra, se obst:.[vU! que !:Jay 12 + 14 + 14 +t.ti = 56 illilliviciw::seaas dLIS~~; ,], per 10 tante, R.F .. (5riIlOOS) x u:m .. 5.5!111l h~ta fl"t~cue!1cill no dil:l.) !..IIU~ lo~ do:'> kid debt;n estar Ii gaOl)« el mismoeromosoma, Uti 1::1 ~i gu i ente .rnlilin.cra:

+----5,) m_ll .,' ---.....,.

Veamos ahora la recombinacion entre 10;;: loci sc l' vg. Ui genoilposparentales de entrada Iueren !,j' Vg Y JC + vg ~, de Frur' qu e debemn,~ cal CIJ 111m: las Irecuencias de los dcscendlentes tipo sc vs' )' IC+ lig {en esta ocasson, ign0r:11110S f~C}. Obsea III (J>S. q lie l111Y 243 + 233 + 14 + 1.6 = 5fl6 recornbinantes; 'j; pu. h' tl tit: 5061 I 008 es un vulor de R F rnu. cercane al "50'1t, W cluimos que loS loci sc y 1.':5: 11(1 estrin ligado~ )', pro bah leffi~1l' H~ h,,~lan en l'romu~oma.~ rl·i,~tint(ls. Pudcmo~ re~umir e'tai> fl~ cio!les de ligafilientttJ de la ~ig;llil~I1LC maner~:

sc

+-----5.5 m.u.---_

lIllil t:undusi6n evideiUe e~ !.jue los loci t'( Y ",I( tampti'c.o &~ c~tar liga.dus. Uecbu que rnd:rf.~ confimJ.atr.c ohteniendo ell !mel't"'1 t be l~-.:urnbil1,ante~ r cakulaudo ia RF (prue!J,e a ,tlll:t,'!:1i' lin .. , '''c .. bcdl.<L" esta~ dedncc:iOllit'S llobre elligamk:.iil'lll, P!J<l.CIF. ree~cribiT elg-enodpo d~ los parellt;;liles de] ~n1zamiellto d8 r ba como 8(;- IJ'{' ·I.~c ec : V{;' h'.1? )( ,It.' ecisc l!{.' ; vglvg,

Un seguIlJ:o ejemplo, ell e~ qli.~ llLili:zarenlos mros Joei de L' sVphfia, T.LO;.~ pel'mj r.i:l'd "inlmduc:i:r al gLlnos c:oncepto~ gcmftic~ gran import<lllcia., En c~te casci, 10& a1dus n.o silvestlres ~r (wmll,ition~ oj O~ ool'm.e 11 ones), cv (c m.>svdt~l~:l"~;, JUat:'riC11L d~

t:l

W:lld\ilt; ~:ruLall e! Olin conecra ndo I a~ venas r~lnl: ~p~lc~) 'i cr \e'w, 'or.td~~ ~ lllli al ~~ dCI1Lado~), E~ta vez, I as esti rpe~ parel1Ir.a~e~ sm~ !l~S~i!i'. dllb~B~ nmlllDcigottls recesivas de geIlo~ip(l L' ! t~," . ('!i/n - , l'rir',1 }.' m()se~l'lh nmoci gotaN rcccsivas 'p~[[a un solo locus de ~rJl'lipo ll/V • G'~. tc» .. , U ~ kt ' , De esre cruzamierun, sc obtie:r1e UIic\~~~nd.crlciillIip.k bd.ctucigO~ic;t, de .'~encx~p('l jilt" - ('I'I.-v'·· . rrIO;'., y bemhnIN de este ~C'Tmli'P0 se ~tll~J~k:H a CflJza~ Vl ieuto de ~~l "1c)flnJn.dJQS u'ip] es hornoclgcros de. ~,,"m~tipo ~ 'Iv ' l'ple}' , '"'1M, U)!i gCnlotiplJ~ ~~u.I1(h,icos de las bembras, que deterrninan la;_oc!H) Cb~eN~IR' [J!pmoc':{';L1 entre la d':':~l:x·mlel1,·j<l de estecruzalI:JrmilL de prueb8, se i ud lean ,I C()n.tt~ll.U:1{:~(m j unto a sus nL~ meros r';;;F'=-d.l~(H. si11'llI]n l448 cl mhue.ro Wt:;;IJ de m{1Q,C<k':

~ • cv ' - (f I 1'+·' (,v· U \.' - ('I' • ct" II I " r"!' I - ct

58() j9'1 45 4H 1':9 W

3 5

1448

I; OJ' <I

\;. - ev" - ~{t r - ,'~ I • ('I r'~·fv'n·

Una ~'~l mas. dcbcraes util i4!lJ' el lVl'ocectim uelltc hahitual para imllJi'<Ir la recornoi nat' i 6n, si bi1.:-11 "tId.l(:'UJ,{jZ:: ser ell L(I:~·dor;;nl; en I fl. '.,I}lI;Ltddon que h aga m!l1~ de las d~.iCR psreruales y T'~'C~)[[:I'bi [IJi1- 11\ Ol:l:,ent: que ~(lsge IIOt i P0:.1 parentales de entrada ~)malt.l~ ttlpl~~, ltr:::L.-ro~igulu~ SiJ'Jl ~, ... , c v ' U j \. - ("I' I ' . .--J I; ydebernoa toner ~ ell euenta ,8 hi hora de decidu emi.ks Sl)[l ,loti recombi nantes.

CoJru~Ii<lll do con lo~ J 012 i i, Y r.l!, vemos tjlK' ID.~ recombineutes !jjIJ J.' g-..:Jmtip~) l' • C v y v ~ , {T •• y que, (je b~to~ hay 4:5 +- 4D + 8~ + + [)4 ... 168. Entre un lUL!tJ. ukll14S mu~C'"<l~, este ~ll:i'lllero !iL~ pone nn(!.l{ I' del 18 j: q;....

fiXd lot; lod ~' Y H. los recombmarues con v : ,d '.i !.-~ • ct" , HHy ~~ .!. '9:;l + :. + 5 '" [9.1 de estosrecom bi l'J.mje~ entre 'I as 144~ mfL'-';:i>-~, a~f que la RF = l3.2 q_

llral';!) ct :'1 _,"" I.oR recemblnantcx Scm. (' L' , ct + Y eli + et. de J os qllt h~]' 45 +- 40 <t- .~ + :'i = 9-3 entre lus 14..:18 moseas, de manern ~ue:ld Rl; = 6,'1, ~ ,

LQS trG~ gcnG~ t'~jla~ hg1ldu~ aimi~m(l cromosom a, puesto que 100 \I:!lrli'e~ de rtf Soon considcrublcmcnte rnenoresdel 5U %, ToeniCili1(l el] CU~~~t8 qUB II a [{F 111:h c lev ada o:::s hi L] LIe ~t', dctecta a1i:!'~ ll:iS: loci I' '! CV. estes deben S1':'I' I fl~ que esu.rrn ma, alcj~illQ~; ~', l);)r1(! tanto, €] .~oc U~ ct de be esiar 1: ntre- ell (k~. F'rndflIl''IO!'l di bujar J map:! iig~11ente:

+----1 j .. :!m .. u.----;..-- 6 ... , m.lI.----

EI ~fu_mm.il'llto de pnlt:Da SC [)l,tdr:: re,e;"c·rli!)II' toomn 1'- Nevil: cr+ ,~ 1 )f; " r.~ nip ,.1 C~'.

Ohsel'~'e VJ rj ni: P~lllH1'~ If]] pmtllnl:es t] ut: bay qlJrL) (:uIlsjd:t':f.'l1'.

En !llim;;r hl~Lr . ..::1 fi}ldt:IJ que- se deduoe para .I D-~ m:~ gc:nt'~ L':.~ ~i;'ewl1,e liel. qUI;] Ne Vcc![I la li ~ta de los gr;::H(JrlipfJ~ en I~ de$cell\"knob. P~e~to qlleel nbjelimdd ej~rc'.i.du t'T& J!ttt:tlrrinllj las rcJ~l'1LJfL~ Je ]j gfLlILJeLllC! e !J[re e~F{1r;. gene'~,I~ h~l.a uri gi.na] 'L~llLa qU~.~f·I1~~~Iri:]JIIlf:ntr.:· a:rb~L1ari.:l" 5:i.Lnp]ement~. n(l COfl(lC!am()N N (\:~lL' de ~CiS .gcl1 el' an't:es: de anahzar 108 tk'IW~,

lS]

ctl

Fi gura 5~·12. E_tGmlptl.i clil' {~illr,:XTU",",1I !HI"III(i ~Enhk, or'!oiCI v c que ~L'

pJ)Jcl LOC~I I ~IV!''''~r;II~~ llohk;: :rt'Ce>rnoi Il~,n1e~ ~ nenen l~~ OOnJbutlilt.·i(>i~~ ~W i~a, p!lR!n:lal~ para JlIIi k';"l "'.\I~IlI~'~'

En ~gLlmiu ]LI~M, hemos establecido t.lcl:lIliL~\!lu_[)ewt' que ('I ~!ML entre ~' )' ~'V, Y las dtsrancias en unidades de mapa entre ct y exto .. dn~ loci, Si n:I embargo, In colocacion de IJ.:1 la LZ4,tl.UcriJa ). CV <I I.~ derecha ha !}kln arl1~rtr;,iJi~L. De igUHJ iuanera, podriarnos habet h:m:nidn el mapa.

l.In L:erClBr .a!'opC(,Ltl "' desracar es que las dos di~nmci~~ de ruapa fl1:!i;~ peq:llei'ilas,. 1.:;.2 m.u, }' 0.4 m.u •• ~llm;l~l~ ~,6 1:l].U-T quees UJ~~I, distancia mayor que ~<l .de I ft.'i m.u, eal:cuJada. entre ~' Y fl'. l.A que xc debe csio? U1 respuesra a esta pl'egLmta c~[a. en come hernes an ulh:lIdn bs do~ d~l:s.e'3 tl18lS raras en nuestra elusi linLcio.n de r-~comhma n.r,e~ entre lOff ] nci 11 )' C v. Anora qu e rellelflfl()R cl mapa. podemos observer que eM3~ des cla .. ~c~ W1t:110t;; lrecuentes cerrcspotrden ~[l reorl.icl<!J(i a l'eCO,!rI hlnantes dobles, q ue procedell die dJo~ cl1trccmz~mielLw~ (trig, ,5 -II ~), S j, n em bargo, no III "1- mos e>1:I cuenta los gen.QlLip~ 11 cr C'l' ~ 'i ~,- d (' L) cuande C-H.iclulamo:> el valor de la RF entre ~' y ('v; ya que, d~splle,~ de todo, csa~ son cernbiuneione; r.ilrel'lt;~les respecto de v y n' (~' C'!J J Y ~ I n"I'. No obstante, ~L Ia Vi~~H JeJ !napijli, obaer .. amos que e~to l.Ieva '@ 1I na ,e~[i111~, ;;I. 1 a hllj a de la~ ·dL~l1tln~i~lJ> ensre 10;;; loc i ~ >' cv. l\:~) ~(~~{J ditb!aJllos· haber CO'T~do la~ dos claaes m{L~ raras, sino q~l"" debcrfamns haberlas courado dDIl veceR,ya quccada u 11.1 de eH:l_f; correspondea una (:]~lli,t: d I.: rooo.tut:!iH"nt.es dnbl es,

c'/'

v cl eli'
';:Y- cv
ev Itt+ cv-

cJ v ~v
e!f' .c .... , ~
at r.v =J ct

v'

P'lgu ra 5-13:. Solu JL<Uy Il'es I)f",itrl~~ i,,<ari~I~~ d~ o(&-n 3iI m2sg(',&1~S.

'p.r"1 1,;i~di'1 I,m;") .cI.:;: dl i'li'. )tri Cl)[fI:cru.1:lmicn1Q Jo:bl~ ge:li..~il uri ~~ !1ri'1!'j pn ru".;hk rC':;oD~bllmDLe LimllU. ::;OLu 11i pn II 1t"1;1 ric, ~a", ·rrf<1r_I~~cinl1cs C~ Cm11![!121!lbl~ ~"1J[j !, "" ti!LI(', rid I~~I"',

19

Pur tunro, podcmos corregil' el valor surnando los uumeros 4:5 +I .;If) + lSI.' + 94 I- J + J + 5 + 5 -= 2x4, Ellln: lLn total de 1.}4.::; moscas. este nurnero corrcspnnde esactamente ill 19_0 %, que t·s idcm:icn a la siuua de las dos dlstancias intermedlas,

UIl:j 'ole;::, hcrnos adqairldo ciertu oxpeuencia con ][1\\ darns de esre cruzamiento, podemll..~ volver a In I isla de lu descendeacia y conrprobar q ue, por lo gene ral, es posiblc tied uc i r et ordeu de I.,)S w.'ne~ medi<inl{: una simple urspeccion de los daros. sin neeesldad (Ie calcular las frecueucias ue recombiuacion. Unicarnent« sun I[H_)~ihk~ tres ordenacione» dis: inras, c~LdJ 1111li'1 eon un g!:.l1 diicrcnte en 1 •• posicidn insermediu. Nrnmalmente. ~c curnple que I(lS clases rnenos nurnerosa» ~nn las de h'b< d~)bk~ recombiIHUlk,,; y tan <;(\10 una de las ordenncicncs debelia ver cornparible COl1 'a formaci 6~! tie estas crases m{l~ :rara.~ DOl' dobl!: entrecruzamiento, Como se mue:>lru en 1.11 Figura :;-'13. s,S]o hay 111'1 or"d t"! I 4tlL.' puede prod ucir rccomhl IKLfHe.s do blcs de ge 11lO"t i,pl) l' N L"V~ 'i v N' ("', Observe, de pa,'j(J, que 1.a capaciriad pam dclcctur un erurecru zurnierue do h le pasa pur disponer de- un gen en heeeI"ocrgmis t:rUrc los dn,;;, "iti(l~ de-entsccruzamieruo; l>i las bel11iJ:rll!) progt'rl"itom~ n(1 t'1llbktan ~~id() -het)erodgt)w~ cur;': nunea habnamos ldenrificado las clases de losrccomhiuanre dobles,

Pi IlJ I mente. dcse cuecta que !O~ mapas de Iiganuento ~ ~tD"m a unos Il)oI."i ell rcl3ociot\ COIl utros, separados pur disiancias en uniLjade~· de rnapa normalizndas. En realidad, rIO sabemos en ,que sitios del cmrnosoma m.;tan ~nli loci, ni ;-;iqlliera en qu~ cromoxoma concrete so cncuenrrau, UI] cscncia, el mapa. de 1 igamiento ~s IIIW abstraccidn que" veruo veremo en cl Capituln li"i', podelIJU~ eorre 1 ac i onar CUII un cromosoaia dcicrmi nade, y con rcgioI1C'~ concretas dentro de e:i, ullkamCtlle cuando apliramos prncedlmienros especiales de aualisis dtogene~k;0.

CDROlARID ....•....... ., , , ..•

los cruzam f@lnt.Q!l de p rlleb:a de tres (0 mas) Iillll flto5 petmit@n averTguat la5'rskl1:iQn~ de IIgami!!nto entre tres [0 mii<,;) gSI'Hi!5 en lin sole e~eritrn~nt.o"

Interfcrencia

La dererviun de las dmie!> de rccembinantes dobles demuestra qlle: 1ij~ t~mrecru7~1mieUilo.~ dollleR liIejbel1 oClIIITir_. S~hie-tl(:Lo e1>to. m.':> poJcmo$ pregwilar ~'i In',.., ellUe,rUZ.,lrni.mt,p;. f:ll I'egiollt'>; emITIOs.-I) III i':<lS ad y~;l.cen.t;e,~ ~0'.Jl i I1de,pcnrJient:e,~ 0 ,~I, pur t:l t(mtmrio, un ell[[e.:ru,zamiento ell IJna r~gi6IlJ afecta a la pwoabilidad de que St: proclU7..cw ntfi, ell UJ1 si tin cerc.a nt', R.es ~Ita que, a 1T1t:llllJU, lo~ Lnlre~ruznlIl:ic:nws IlO ~Ot1 il'ldept';lldicnte;;; uno!'. dt'! O[IO~ Y esta i nrej'~.cci6rl n:cibe el ["10m l"re ~Je .iJ1ltedel."e"ncrn,

Poot:lIltl~ ::uilal'izar este t'ew:'mcllo e-mpi e,UHJ!(j el ~iguif:'nle r.170- IlfUlLi.t'T1w. Si Ins e:run::cmziimientos ell uu~ ~it~n~ disHntos tl.iem[l irluc[lemJiente~. ~k aClIerdo con l~l n:gla del proOOcIO (\"L~~tSe I:) pag_ :n J. 1<1 U.'CCUCIlC ia de rt:(.;QI[] binantes dohol~s dlel!x~ri,a _"'er i gual a I prudllc [(} de las frecill t:ilC i[;l~ de recom b i lUuates cn la, dos regia-lJe~ adyace!lt~s, en J.u~ datn~ de recombinaciOn. COIl V-chell. cl vaT or de 1<1 RF pw-a l'-j'[ e~ 0."1 J 2 Y pam ct-n' es Q,{}64. de fonm que kl~ recomhinant,es d()ble~ dcberian aparecer. en 0'11:'10 de mdt:P'~t~JL~nci a, OOH II n,a m:cuenciil de O. J 32- x CUlM = U, umw (O.R4 %)_ 1Jn, 1a mllestra de 1448 mmiCllii, l'Ie Mpe:mtla la apari-

CiC~ll de O{~184x 1441$'~ 12 recombinantcs dohles, Sill emharIf 1(1"< dl,~roo; ini.limm qLJ<~, en renllued, ulTlIl'c'],ln se Oh5C:i'V<lWli 8_ Slf~ neti('il t:1:J el Tl'limero d~ .r~cCt.l.mbir:lant,e.<; dnl~ies ~ repitiera de ~II COI'ISi~LCIl'i.'c, eoncluinemos que la<, do:. regumes no son !oUCF!<T ilienteJ; y que en la di stri hue i l'vJl Ji!:' It)~ entrecruza rnleulos, h; tt ehos unico~ i'.'t' ven faw'lre-l;]du~ sebrc los dohles, 1.!J1 otras PlIliL'r, se pnxluec algiln ti po de inte Jftf'J.~l!(/~~: ~Ifi emrecruzam ientn r<;1J~ ,111 pmhahi hdad de que tengn hkg1lr utro en '101 r,egjon adyll..«.'r.1.:

La. inrerterencia S~ cuantifica calculandn prlmero un V~lkt("j1: nominado ('ue1Iicientc de ~,ujnddelilc:ia (C.O.C.I. que es el co~,",« re entre l(l~. dnb Ie,> recornbinantes obscrvaJo~ y JOi< es pe-J'fI,x-.;. re-M:in.anle il L cste valor, POI' tanto

lntcrterenclu ~ lJ =. J - c.o .. c, =

[ lrecue ncia 0 numere de 1

rccombinantes t10b 1 cs Oh~ell'VadO~ . .. 1-

frecueucia nmlrnero de

recoiubinuntes dobles eSIK~raJ[)S

en IHJe .. -tru ejcrnplo

I = l - .~ = ~ = :lJ, 33 ~l

En algunn~ I'~~iurnc~, IlUI'1C~ se observan reenl11l;1iLlam~ 11 blcs. Ell e~ro£ casus, c.o.c.e 0 e L '" l , tic rnanera que J a inw'f; ren cia (;:.S curnpleta, L_a mayorta tic la,~ veces .Ios v.31 un:~ de i nil! fe r enci a t] ue se observ an al cartogra fim' loci cremosem ~C(lS es entre () y I ...... TO obstante, 'I:l"fl alguues situacioncs especiales Ilfl.ll cen lnterferenci as 1](' gativas, al obscrvarse Ulli n Ii mere J't' r,,"CI:r hi naure do bles q ue e,:c~d,e ::I] csperado,

Los eruzamientos de prucba de tres puntos, l' otras vel':\i<ll m;i~ cmnplej{l:'i de los rni ~lTI[1S. nenen Mil importancla UIU (LlD1 menta I en d am'ili~is de 1£1 recombiuacion que I}l,ett.'t.'t: la P!t hacer LUI resumen flRSO n P.H.s,.O de esre tlpo tit c~Ludno~; finJllm du nm el dle1110 de la imcrterencia, Utilizaremos ios yah' nurnericos d:el cruzamientc CQJ] los ~~ci .\\ c( 'l n',

1. (:atolilu de las frec].1ellcia~ tIC" rccornbinacion entre cacla.. de gt:.m~~~:

" - (:)'=I! 8,;5 %. ,CT - ,n "" 0,4% ct - r = 1:1_2 !Jr-

2. Rt'PfC~C[ltackm (Ie [as reI ac iones de .ligarr.Licrlh) en 1111 inar

v

~,(

---------;-

cV"

+---13.2 m_ti.---"+_t--o_4 m,U.--·~

ct Detcnnil1acion dt: Ill;:; dt'llieo;. de recmnb.iarlltS dohle.;;_

4. C~ik:ul{j de lil f:l'eA..~uenci~1. 0 mimefQ (;Ie n:~umbimlIue~, dcl.i, eSPb'TaJoS en Gaso cle qlJe no lluhJ.em iJllt;rli;Icneia:

Frecnel1~ii!l cspemda = U,]31 x O_{)M =.o.(J084 Ntimero eilpwtdt'i '" (U)084 x 144!l = J 2

S. Calc 1..11 CI de la ill{t:r(ercncia:

NUH1If:ro ub.s.enaao de recombinHIlLc~ dnhlef, = 8

N omerO c~pcrn.dn de recom bi Eit'mle~ dol:lles = 11 I "" ] -'1 "" ~ = 0.33 0 33 %

l·U:\I.)!ill5,}, 'lialNes de RF (por,entale} corraspondlurrtes a ~a.'om1: selBcclot:l'aiios de z en cnrzarnlentes-de dlhlbrldes en fuse de repul'sl6n

:z

RF

2.~ ·1..9 \l)~

13~ll.

2>1 I

Una pregunta que cabe hacerse es pur que en nucslros ejemr ~~ k lig~erlto en ,Dn)stwhUn url lizamo ~'empre hernbras ~FerO;:igutica5 !til los cruzamieatosde prn eba , Cuando se ern~n ml!"h(1~]Jr t'gipr 1'£' em, hembras pr Fglpr vg, unicamente "He';l1F~flln descendienres PI' I'glpr -e: Y pr I.'gfpr t·g. Estc I"C'UllID.l,) demuesrra que no se prodncen enrrecruzanuentos en 1m; ID':~;()~ de Dmsophila_ Sin embargo. csta ausenoa de entrecruzaCli£DLIl em uno de los s.c:>::os se limila a algumls cspecies, y no es eJ (~'D para los machos de to('lda~ 11I.~ especi es ~ o pam cl scxo heteroy.m.hico), Ell ,(l~1'OS 0rgal1ji~n.ll)$, se producen eurrecruzamieatos >xlo~ machos XY y en las hembras \""l.. La C[lJU1J.<l. de la ausencla d~~'W.n:~cmr.:.lmiE\n:tns en los machos de DJ'O.IOphi1.a esld en su IUum,!1 IlI'l'Ifase 1.';:11 ll'l. qlle n (I se forman complejo» R inaptonemicos ..

C,lbt: decir que [foIlllbien huy UUfl ulferencia entre to!'. sexos ~.IID:ell)S ell cuamo ala rccombinacion. Las mujeres muestran rii~llenC ia,~ de recomhinaci61ll m:i.'i alta" que Ins hombres, [lara {' 1 mb,tll(! loci.

Calculo de Ias frecuencias de recombinacion en aruzarnientos dihfbridos

L~ \'fa m;i~ adecuada para. calcular 111. frecueuci a de recern binacion t.) l<: Jd ~rul<lllI.i~lilo de. pru eba, Si 11 e rnb argo , en La pr.ic.'ica, no ~<rIJlPIl.: c:.lii disponiblc cl homocigoto recesivo upropiadll. Lna ;:;tllIl.oii6it ila~t!lllte COIllI.lIn es Iaidenti ficaciun de. lLl~ nuevo fcnetij'oTo;joo, medi;lme Olnjl15js rnendeliano, se denmestra que eSEa pro ,bJido por cieno genotipo ala, Para locallzar esre nuevo locus en ,;. )11:i~ g~n~tit'o, el individuo a/a ~e (;J:UZtl cull DLrOS genutipo:;. ~WJa ~ 1 bib, ilJ);qdB. ya. ,DO !l(1ceml)~ ~.a rn,~ ici61l en el. m~lPll d~l . .I DC: I.l~ ~,b est~ si('IlJdon • .s.e pUe\1e ob:;.wval' qm~ I~O hmyt'li ngon IWinwdtrifi:l ~e~vo dispoll.ible. De Ilecho, 'el gelflotipon (jt(1' hfb ~e ~;tJ1,'; t~b[l!nt:r tin~CamL':'U1tc IIIcuiantt: cml.'l111.iento~ adkim18le,s.,

Nn nrn..t:.II.lt.B, re~u I ~a po~jb I e calc u 1 ar tal'S f'imcu-.;.nci a:. d~' TOi.DllhUtilC10I'l n partir del cnlzamiellw de lo~ dQhJes hetemdgu 1f6qlll:: ~e ['onnl:lll al Cfllzar .las <los esHllJeS, En est.e ejempl0. los ;«ri:JI[aks seri:m ala. - BiB Y Ah~ . l,lh. Y Ala - BIb td dil:ifhridu, E...1f dr'l[)te heteroC'ig'Oto pmclncira gamern~ parentaleR l1. - R .'l " ~,y g.lllktus !econLbinantes A ' H Y .u ' b, hn el cnn;<tmiento clihi'tIt1CU\ lot'! gamd{j~ ~e fusimtanl:lll al ~l:lar t:. i.raic.ialm~nte. iUG r-uce p"5Ib·le rec(mOCer 1m. r'"~cmnhi nante~ eTltJ'of) ]a de~cendert·

cla, Sin €.mbal:WL la dcsccudencia CD'n ' bib nos ~11·\'e. de. _grim ayuda, ya qLl.e es el rinico genotipo Lp.K' debe haberse hJi'ill,lLkl CUll g.fUlli;';t~)S I-';COlnbirr:llue~ -de hecho, conla tcrtilizaeion ue un gamero /..<' ; b con 0(1.'0 (~ • b. Por lo tanto, $oi 111 frecuescia til" los prtaluctos m€'u6rrims a ' b C:~ p •. ~a Irecaencia de 13 descendencla ot« ' hlb ~em.p~, En crmsecuencia, para hallur 1;;1. ~nlul' Je p slrnplemente tenentes que hacer 18 rafz cuadrada de ] a Irceuenci a de la dcscendcncia ala . hH), PU()~lO que sabemos que 1<1 frecuencia de. (.I " l~ rlehe ;<:~1' i gual a lao de A - B, podcmuad uplicar {I P'(")),A obteuer lit frecuencia de l'ec.nmhinacic1n total,

S. los do~ genes no cstan tigados. sabemos que 13 clase ata - bill se fill' mara con una freeucnciu tit" I~ (cl« I '" dela proporcion SU: 3: I), btl e~ te ColSO, 1'<1 raJ:> cuad rada de " cs 1: l' _ d uplicande estc valur 11. obtendrlamos llt~.1 R'F ~e 2' " _':;:0 %, como sc cspt'nLha_l,Qu~ UCLl.ITC en C1L~O 1.Ie ligamieute? Las sil1l.ackmes de hga miento p.rofiucira,] frecuencias de. ~~ h!ll b Migt'lil~.c • ativamente meuores de 16. Sll'j)OI1gJIIlOS que obtenernns una frCC'II .. LI:mcL;lI de a bta b igua.1 a n.OJ ~ 1 %);la lrecuenriadeIos productos rneloticns 11 h debe huber si do ~l J, 6 I (1 %, I:LJI[l Io 4 lie obteuemos una R F Lie I 20 %, un case evldeure de I igalllrenro. Observe ,tidmbi~il qut:, ~I quisieramo» calcular las frecuenclas de los genotipos p~\Tlmti:l1c~, estas deberfan ser del 4{) 9': panL cada Ul\U de ellos. (I B y ,1 b. calculadas como la mirad de 100 - 20 %,

t:::J metoda que acabamos de ver es t.e.oriC'a:mcrolc conecto. peru {'n I~ pn1cdca. es poco prectso, d;eoido a que se extrapelu Jl; ~6lio uno de l.o~ Ji::nolti]Jo~ de Ia F~ y, ademas, illl eluye una roil Z cuadrada, Se ha ideadn un mctcdo mas 1lJJ.t,'C uado que mcorpora rodos los fenoripos de la F? Se calcula un valor deneminado mz~;n del p roducto (z) 'Y Ia freeuencl il de recnm bell acion se 0 bnene a partir de. UI,ll.1:'l labJa!> de valores de z, En el dihthrido en Iase de relJ\lI'~i6n mendonaclo anteriorrncruc (A. bla 81. Ia razon ud producto se calcu ta como. igue, donne los cuatre componente·s del talculo ~OL1 ~()S U L1alru 11: netipos de Ia f ~ ~

jA/- B/-'~ )(lJir~ b'ib)

i.=· - , " .' ,

(A .... -b/b) x (a/a /3/-)

(!'\ota; para abreviar, Ia ftirmllh1. IIlU lit ha escrlto uti I [zando 10_, srmbolos de Hgarni.enw .. ) H Cuadro 5·2 mucstra lu~ vulores de RF tl ue corresponden a va lores seleccl CIL~ad(l~ de. z. Ex istcn pwgrarnus de ordcnador l!Ue calculan Ios valores de RF :I. partir de, los datos disponiblex d€ la Fl.'

'(ORO LARliO . - - - - - .. - _ .. - - - .. , , , .•... , , , , , . , .. , , '" , , ,.

I!.a fre-cuencti! d~ rE!c-Ofl'Ibinati6n ~Il plJooe I;il~(u!'ar de modo indirecto a partir de I,a descendenda de cr,IDI:zamiento5 dihfbrM:los.

Bjemp[os de mapas de l.igamiento

Lo~ map~l:S de l.igam.ieuto l;oLlJsdtuy,en VI) JspeCEQ e~enciar del e-SfLldjo. g~netico eJ;perimenlal dt). cualquiet organislno. Som el prclLulio de todo ime·mo serio de manirulaciiin _gm.el'ica. ~Por qu~ 1 a carLO~'11!1i1a gell~ti:ca es lun :lmport<lllt:e:! Los [ipm; de genes que un flrgan~ snm pO,~b-C 'I s us p~Jsjdone.~ en .Ios C!I'Omlf)-(lHHlS ~(l1l f!Spt4;WS fl1w:l;unentaJ~ del ammfi~~ genf.lico_ L!I~ ra/AHICS prillcjpttl(~~ p.ara (:onfeccioJi . ..ar los ma:p£l'S g~ne(ic(js taene n qu.e vcr

,

QI,O ... : .. ojG!!; arru g!loiQS

n,2 s- .• van i'l:!> ,r1efect.l.l(l$<lS

• , al a S curvadas

: t venasen ell bita D.©·1\ IJ~ los reQortado~

. I·'it eseutete s ion S'UT'~iJ

\ sin cjios

..... ~ .. ~~.:

., ' ' '''' .. '''' ., -- ,..'-.,..,,'- -_._ -.,. --,,..,_ --_.- ~............. . ..,., ~ _,., , ' ' ' ", ""

- .. ~~.: :

Ii . ,

. .

, i

Ii ~

• •

,

,

• CfJarpD 8 m,Blri!ia

o.o ,~ r.erdas t[)r:3ic~c~s

'.0 .• ojas bl ~ ncos

J.O .\ ujol!; ~ II f<:188'tkl

S5 .', ~'. 0 jaoS eau"t!uS r,~' '. ojolS rub r

1'3.7·'· '. alas sill intervenes

20. [} .' a,1 as cortad as

.£ to" '. 81 as cn8'1T! U8C8d,~3

57.(1

• V~ n 8·S fu~i Dn.,dRs · olos e n ca f'Mdos

• pelos accrehados

- oja\!io an rornhe

3~.O • ()jo~ bermeuenes

36,1 - ~- ~J.as. pBqu8n-3~

43. {J,' cuerpo Gibel~n() 44,0 ,. 'b jO!; Il'r~ natas

~B,7" • ~~rd<:lS [()wr~c:b~

". 1. oiose n 1)8 rra

59.6 62,5 - H6.00

Fig u ra 5·'4. M8J.1a ;;~I>(l iL~~ (foe 1 ~oeDUrlJl~ de [).Qs-op!iila Cj!~::; muestrs ~\gLlD06 ,~e lus b"cj )' 8Im,~ cads uno de ello~. correspnndc it l.m J~r' flr.- <:~\(lrl)("J:;-.L)IIl..j1~ "111) 1b"1(11 ng()Sr

LI~ d·i~I'IJLdil~ ~ dim ~[I umd<~~, ue

mupu u psrtir de I ]leo mils proxlmo ~ un extrema. L~~ \' ~:I<1roC~ In~ s 81 [~" ;,e I1SI.1

~~ l~ul~d" IIl(;<Jh~J:I~~ I~ ~-IJ ma tie Jw 1~~1~1"i1lU!l' r.nfi~ eortos, y~ q ue La .tTcC'IJ~D!:'t~

rlc l'8c~mbi n~iti~ entre tfu~ l.'>!: i nn pllL~d~ ~~liL1I~~)r :ode:l ~O (ii, (fOJJlil,;.lU J~' IE: W

Sol nnon, L, C. D1mn ). T. Dobzhunsky, f'rindljl~R nf G~IJ~rl~,f. 5.' cd, C("fP)'fi ghr 'C I %2 d~ :-'1~c;.la"" HIIP,!

, • val"l<l!i. ru1icuia dll~

c.o @nt0r1iJ 8~li <lriMu

1.3 .f' ejos on estrc lla 4,0 DI\[IS !lnol:Ggidliis

I

13,08 las regmdetas H:I.5 ojes en gorumlos

48..5 501:0 54.'5 ~oi,8 ,J

55,.0 .57.5

'CU~~PQ l1~grO

I cerdes ~ediJ(:ida s of os p(irpu ra cerdes co rtas Qjos.c.l.a ros

DjOg .ci ne brio~

2:&,0" • 0 jo!> de color sspl a 2'5,6' ;. cuerpo pel udo

41. ,(I '. " cerdss ell 105 q uetes 43,.2 .: ·.~ri.sta defe0:'LIOS!!

44AJ • , ~j~~ escerl ata

4E!,O ~ - JljQS rQ~.Fl8. 60.{1 .' • .,1, R S' ri.2ada~

. OJ08 fQ~<l~·

·53,2' .'.ioll.~ ~ rlzadas

r;<>. c .'.I~> c~rd.~s dE.'f8~~U'OS~s

~,,~.,.~ ,I' "ub·1 68.7·' l" ~'r\Jos "" I. QS.

62Al ·r ;. c u erpe bite rae ICO 63 'D' "c·u@rpo. con bandas

. • '. ~}J{lS oJfls'tal 66.2.: . : venasen de~ta 69".5 .': I ': cU1en:lO ,s,i n pel os

] 0, 1 ',' ., 'eu p.rp(l O:P- eol nr ebano 14.1' 'oj'l)~ ~ard~o.~1

106.2· • cerdas d i in if! lI"~S

Fi 9 ura !i'~1.s. ClIJ"iosrafi@ de 10:> (.rtm;lo.o.)I113;',.ML rnmate. ta} \1j;:;r;::>r()l.O~r;)jff~ d~ l!TI~ profase 1 ~e~·Q~kli (p~ll~r~l'iC) ,t" ~Jl1r:I.·>Il5, C~ la q~~

se observu ~L u:qJ!t'ol'IJ l;"'lJD e] 1]lJ=t}jll<J de Jill 12 p;ur{;}. iJ~ g01~1fll;, (bl ~"i~t=u. de numcrac i&~ de lo~ crornosomas que se DuHz.LI en til actuulidsd,

'A \'!~r!i. M lQ~~lnl~n1Q ",,~1 iT~n cn ] g~'_, ell c:.i qll~ sc rnucstran 10.,

grupo~ tie tig(Ljjl£i(!rl~I' ·et'(l}()e;id~,~ emunces, j ~iotU' <I ~;}I:I~ ~1I~l!S ~1';11:t;~..e il,11

dibu] n de la variaute fell(]jlp.lco. que f'=lTAitio la ldentlricacsda de dicho JOtiJ£ ~~G~i(j~ JX!!" prlJl1G'~ ~"C7, I a 1~ qC!rcl:"~.a;' ~ ntro <iilluj~ del fc~otip.;) normal, currespuadlense (-0 ~I 1;&Jurp,'td~) I.M;Jj, ~!.n:Il';;j~~ ~!]~ los .'J~~i S~ .nd i~~.11 ~II llr.i,rnB.o:k~ de nlIHra fl'llTta a y h tnmadas ,i!I~ C. M. Rick. ·,l.'b~ tornuto.» ~frhb j_'J 19'7 ~ rl~ S-cI~!1~inc A".~rl~8n TI1~. R<~$~!·"'.l!il.0~ [ixl\:l~ 10;< ~f·cchru, Parte

c tulI~~dJ!l d~ L. A. Ij·Ulk:t',)

66,7 • ojos escabresos

67,0 a las vestig i ~ les

ra.o ojos en 16btj 10

75.5 a las curvadas

100.3 104,5 107.Q

a laa ell plaxa· pgas IT1,aUtl ne!5 CU8~D rn ol:1o!<.\do

Hofo !R)

a~1f;

Amarillo ~r)

Amilri II~ (L.,ln

n

N"f'"r!~1 (0, Obiongo ~ol

1-~ 16 )1\...

Normal (B~

LBnOSD l\Iorm"L (L1Il)i

l~4'wol

l'I~IL'1r~S~J1-I:ii1 &implillGI

No pi~"de> la_~

It)~

P(X;OO< M"ct.Q,~

loclIJ'P&' rL~ !O!;\Jlo~·

lie)

H.[it~ lipo rMete'~'G)

li;rJ~1:<'frm Inllodo Ii""1\ Datarrni nudo

I.Spi ~_'i'j)1

R~~lste~cIB al t f Sen.gfbLl&d8d a I

m ohn d~ la hoj~, ,"-Wt 1 IT' uhtl de I .. I·HiJ.a

de tipo I (Cfpll , ds tipo 1 (cit> I)

r-

Pu IllU!B !A) ~ B v<'trde ';,;)

r"'-./JiC

~o<m"llo'IIt: \.......l/ Punta en D£z,6n Cn'~

..

~i'l;~ ",,,de Fruto unironnE

(Ui 1[1)

)l'r::)F

Verde l .... iI)

8

EI1BnO .s>:tsn<::ii,Jo IDm~

~

Mod ifi';';tit1t'1l1' (~i~

1 ~ ena np eompacto

Cotilodt>nc~ BI1cho8 (NcI

CQ-tll<?doi'1e~ s~tre~h~ (tiC)

11

12

Normal

M<l~r'(j~Allz Imc:'

:Mc!

COil J.1.1I.mci6n" ,e \.'Ohlf'1jIJ y ni.!1l6lmir:nm de los genes. Se s abe q lie 1.1 posicion de 1111 ge·rI afecta, CIl nuruerosas u,..:a~ione&. a su e ... - pte.~;ion. LlH fenrH'I~t:'1.I0 '{lie suele conoeerse cornn «efecto de posici 611.». Genes con Iu ncio nes 1',el'l(;ioliadas esti~H a J ue nude ,agnl- 1'8 [los eLI los crernosnmas baererianos, normalmentede bide u que '.1.:' transcriben como unu sola unkind, La po~icio'l de llll g.tll eucariouco en una [egi(i.ll de heterorromatina 0 cerru de ella puede afectar su c;>;.pre~h111, Ul conocirnienro deIa posic'i6m de un gen es uti] ern los esrudlos evolutivos, ya 4Ut: podcmos deducir la~ rcorgunizscinnes' erumoNomic01~ qt4t: sc producen ell d curso ilc Ja evolnciou a traves del JnfHisj~ de las posicloues relativas de In~ misinos ~'CncN en organismo» relaclouados, Finulmente, si 51:: prc.knJc ai slar ~11l gen para 1>U amihsis tuolccular, e.~ couocimicn It! de ~ U rm~.i{! ion crorno .... C) IlIl ica I\t':pn:~en La a men ado el corni en- 7U del procedimicnro de rlnsl;ulTicnto (donnci6n posicional, vea.)e. el CJP. 12). SOil .tmidllJ~ los ()rgal')ismo~ en 1mi que di&poncmo~ tit' rnapes gcnencos bastante corn plesos. L()~ ITIap.ai; OhLeni!Jus son cl resulmdo J~ una enorme Jsbor tit: !l1'l,lil'isl;5 gen,tt·lilJLi. 1k~\'J!I.(b n c ... lho mf.Ji<Ulll;' 1<1 col~a.hQracj,on de grupo:-.. de. iU"(i~!igac[on chstrih~) i du~ pOL" tude el 1D1J nci!u. LaN Fi gUlras 5-1·<1 y 5- 15 lllue~H'lm uv~ cjernplos de mapas tk liganriemo: II.1nO de Dro.mphila y d olm del tornate, 1--: I J!t::HlJma de TJro.1 ophit.a (,'~ uno de J0~ q til: S~ ha C<lri 11- gr;]fi.·~dl.l run mayor detalle de iodos los org,mi.smm, I1lflol1eio. ill rlM!)":'1 de lao Figura 5-1'1 tnuextra [an d~lo LJILa I'rnceion de klS loci ronocidos. Observe qur::. In~ loci de Iuncioaes eouocidas (p. ej .. lo~ de I color de I,t~~ ojm} estan d ispersos pm todo el .maP'Ll. Ta mhieli el LOII.lllt: cs lnteresame, tanto desde el puruo de vista de la iJlVe~Ligaci61~ basica como aplicada, y cl genom@, riel urrnatc [)s uno de 10" meier cartografisdo« d~ Ius plantas,

L()~ rl iI ereules pane I es Lie Ia Figura -" - l;i i lu Wan ~.1 gwma.s de las f:;).~t:~ til; t'ml()cimi2111~) ... LraVt~,'i de I<li, cuales 111 invesngacidn Ikg<l a cstablecer un mapa complete, Ell pruner lugnr, uunque los crornn-oruas ~Otl visihles con cl microscopic, iuicislmcnte no hay fOI lila de localizar 10';" genes ~I] ellos, Sin embargo, Jnii CI'(j_· m()~UL[I,L~ pueden ser ideIlti[Dtado~ [Hd·i .... idualrncnte y numeradtIJ;. en base a cierros marcadores propios comn I os patnJn::;!; de: rineion [I 101 r~llsj;;;iul1 del l~el1U6rHell\l. hccho IndktlJu en los paIlele~ (,I) lJ tkla Fig-uri'! 5-15, A c(ll'llinUrlci"(irl, t'l amtli,i~ de las frecH~'uci a~ de recomb.imlCi6n ge ner.~ una ~eriC' de gl'1J fI'O~ ~k ligami.cL1I:O que l~le.bcrL com~~pnndt"!' ;it 105 ClfDm()~mn.!s. si. bier! 1m pl.K'cien esl a hkcCi"M" Ih=Cil~.1 j'g flmenLe ~'{)neJDd(lu I~'~ t:~JtK'cmCUS eOIl C)d,L I,;rulTlu~mna Immt,['luo. En al~rl:i1l lI)ullIcnt.o. como viI1W~ en d ejcmpl.o cI~ hi llIlo~ca ('{It·hl.iomyia hominiJ}or(~x. [u~ ilmll i:{i:; C.il ogelleliCA 1S I_)Cl111 it.ell asjgll ar Ivs 1!HlpO~ de I igam] ~ITI()

a CIHI11 osoma» o.mcretos. La 1 'i~ Ll'ra .5. ~ :1 c tnuestra un map~ dd tomare elaborado en 1952. con las relaciones lie: Iigamieneo ell[J,; gene; q lie se conecian en 1 ~ epn.c:a. C a da loc L1~ e~f,1 repreS~illlidL! pur los des alelos utilizados ell los experimentos de cartografa OL'iginal~. t:uIIfnrme Sf: fuerou i.:onOC"ie'lldo mft..~ loci, se c~rl<1- grathJro[l en l'erac~ol1 fI, 1Q~ loci mOli,tJrildo& cola Figura <1-l:k, d. maneru que e] mapa eontiene en la aetualidad ciemos de 111[[. Algunos de los rnimcros cl'omos6mit,o~ im:i:lciKlo t:,11 la figura ..... n provisiouales >' flO se correspor« . Jen crm e] sistema actual de IlIJl1t·· raclon de :1m, cromosomas. Observe de L1n,~\'U que genes (on Iunclones relaclouadas (p, e]., 1<1 forma del frutn) es(,8.s disperses;

Prueba de l cn e~ anali sis de ligamiento

Cuando Jot. valorcx de 1<1 N.I; estan prrixlmos ul 50%. ~e pu~i'; uri IJUI! lu pruebn de l. -r para cstahlecer ~H existencia de I igamiec 10, Supongamos que hcrnos cruzado des parentales humocig&i cos de genoripos AiA. . R.f8 Y ~~h~ , bib. ()btellie·ndlJ~e cl dihfbrida All - Wb, 4'LI~ hcmns someudo .a un cruzamicrue tIL- prueba cOP ... do ' bib. Se I'lh~leJ1t· uua dcscendencia til:." 500 in[Hvi'~lIl'~ qJ. ht'mo:" da.<;ific4~JU de la ~iguiem;e maneru (illdic:ld .. til fUlilci{'!f de I()~ gametes pmdtLCido~ POl" el dihibrldo):

141Ji i~ B
135 () 'h
~ 10 A 'n
I 15 (I '.8 A part i r ,1<:= e~W5 d aros, obrenentos una freeuencla de n::cmn b\i'ijlcion de 225150rl = 45 (f., bl{) parece un C,lSO de liga1m¢Il~) puesto que la RF- t:~ menor del 50 'k que ~. espera de la seg:rq1it' t:iun independiente. Sin embargu. L::S posible que IllS des cia"""" rccnmbinantes ~cal1 llgeramente rninoritarias debidQI tan solo ~I aza r ; per .Iu tanto. meCesillllrlm realizae la prucba de. y:-.

lil problema estriba {'II detenuinar los v alore. esperados, C, para caJa (l FI'I de I a~ c I a~es, Pam '{;OlTIp'JTll1r! I' I" e.'i..islent:la rlc ligamienio. p'l)(.I:ri;J.mos suponer que I.n~ valoj'~~ ~spc.mdn~ vjel~n dado~ ~impl('mente por ll:lpropordoll 1: 1: ~: I de b~ cuatrn.:l~ lIl1!;:' :.c· ohlienen en cl t'TlI.7.:Jl1lient.o (Ie pmC'ba -Guam'll) h~:- Sl:gr.l' gaciuniudle:pendil:I1Lc. Entre b descendcnda (1<= 50U imri\'id~l~ de. nI."e;<:tro t;jcrn:pl('l. e,'rel'ariumus 500/4 = 125p-ara carla. d:l'~, !'-lo obst~UilC, la prr)'prJ;(:i(ln I: 1:.1: II I"IfJ <.;lIILI>Li.tuye el Irnejof Ii$. para JeLel"miIl(lI' Ilgamjcnl:o,. ya gbl¢ para ublenerl~ rlebl.."tl Y..T

ClADRQ 5-" rabla de wntingeilcia qUg comp:ara 105 resul,tados ob1;ervild0!i y lo~ e.sp.er,ados {i'e ~1t1 nuz.amliento de PFueba

teaijz,3Qo p,ar~ eMfl'Iirrar el Jj9amt~n4.:o entre I~ IQ(;j Ala y tUb

LOCUS 2:

b

o

o

LOCUS 1 A

').>;; :Ii! 25!J','500=:: 11.7: t:s5'~ 250:' ;:;OEl = £;21.:5:

(\ Thltl

t!",rh)~ UU~ aspectos. Debe haher ~egre.gA(;i~)li indepcndientc de ];~)lut:lI~,A. '! B~ y. ademas, del-e Imbt:r igU<ll: probabilidas] (]c que ~ Iormcn !ft" dislillto~ genotipos durante la h:'1tili7.aciol'1 y de 4~~1tkaJ1tell 101. ed.~J '" [" !.iu,c son cxarrrinados, In que generalMellie ~flli 1.1.~:a q UJe ION cuarro g.enoti pas deben tener igual rm.l~~I~m;J;Kl desobrevlvir desde el clgoto ~I la edad udulta. Sin em b~T~'Q, es frecuente que las mllwci.OIl(::~ que se I.u,ilizan eon el ·.iihsh de Iigurniento tengan a'lglJn efecto delettS: r en en d horne.'1Rui(l; d~ rnanera que I(I~ gf',Jlntip'l)~ aia u Mb ticncn una prohu Bi:ia!!.(l de. '5upe ... 'h~~Llcia menor q~c los hckmcjgotn(';,.It/~1 y Nih d~ .:cen'~tipo. ~i~ vestrc, E~llQ [lodd a 1'1 EJ\' amos a rech aza r la iJ.ip6~1.::~ iii> il'p~gr~gac.'~ol1l i ndepenrlie ore, incl LOW cuando fuera correcta, pu(:~ll) que 1.1f> riiferelldali ~I] lu supervivcncia de 10..<; genonpc», 1.\l)J,;,lrlall des" i:I("jOnle~ de I u prLJparc iOIl 1:1: l: I esperaela, l.o IIIIC- ~e requiere es un mctodo de calcu I ar los valores ~petados, ~', '{k n(l dependa de IJ~ ~ifoe'I'eElCi<~~ en I.~ supctvivencia.

~b imp.-, rta con que frecuencia aparezc:J!1l 1m, genodro~ (I/o Y WJ ellU'(' JlI~ aJlIllo~ tid cruzamiento de prueha, sl hay seg:re}!ad~IL ,[Jdr;pr:lldicn re, e~reramo~ que l:l frecuenci H de 1'01> game ~J> [-il gl~l10riro a . b sea el producto de las trecuenctaa fie 1m Jib" ~ r b, .I::m nuestro ej emplo. la pr,tlpo:r,c-i 011 total de alelo ~ a e'-:13S+ U5J/:'iOn, 4[Je es de hecho eI5(J(,~'e&[.lot::nu.lu. pcru la ir:-'Jl'trla del al:elo b es 611icamente UJ5.L I 10)/500 =; 4-9 'k .. De (~I.,: rr.ilJlfnl, esperantos que la_lh:cuen(~a del genotipo Q - h sCJ! ')_-':h;'{).49 = 0.2-45 Y el tuanero de gen:oti.ro!'. a - h ell ~H1f1 nmesJ:lde 500 sea )(J{) X 0.245 = I 22/L Sc pneJe realizsr el mismo :iro1kdkuto para cada uno de ILk, genoupos y. a'.f, obtener los ',Jl.,I~ c:sper:a-rlos. La comparacien se haec gcneralmente ell una I:!briil de (_ol1li1l~e.nC'ja. como scmuestra en, la Cu iUJ!'O 5-:\, 'I ':l\1r esptnulo en una entrada de If.I tabla de contingenciu C~ (11 VrWl.1l':() dela frecuencla ql,le ~I:': of,~~va en su filu, lu frecuencia 411(; apnrece en su columne y d Lam alfi{l total de I:) mues tra, Lrlnn l;~ Irecuunuias de kt~ fiila<; y lns cnlumnas SOil el numere tn, t .tlL hIS fi1all 0 ~n Ill." columnas clivjelitiopuf d lotal. de 1a 1wu:~llli .. €it c:ikulo del VJlol' espenuJu (:ll Cl~di'l Llntrada C{m~i~te~ir.I]1r411~~t,e e,n Inilltj p1i(:m lu~ valorc~ tutllJes de ~1~ fi l!l Y de su ~.:Jul)n1ct 1\I1J'il (li vidi:r d Ii lII1]l:ro obt.enl dt) pur e I WT~ I de If! mu~~Il'l: EI ~W(lr til.:: 'i S.I!! calclJla eonlO" ~igue:

(Iale 0 E 0- E (0 - tY/E"
l!'if 140 12:7.5 J2_5 1_23
,~' ~ lW 122,:\ -12.5 '1.28
" ~ 115 l27.5 -12.5 1.2.3
,; Ii l35 122.5 12,5 U,8
Total i - 5_fl2 fJ d'l'l rJ~ ~2 uM.oll ~do RC con v ie r[o: ell una pl'olml~.ili,r1ac:l empe~ndo \Iml ltLbhl dc X~ (veas(:' el Cl,ltldro '1:-1. 1)(Lg, 120), Para tij.{[J:;,,\o, net~Rit.amn~ det~I'millflr cilll.inlt::ro Ul' gmtl(}~ de liber Il~!dn en b rrlleba, Dk:hCI !lUmeN; como ~ug.iere su pl\!pio tloll1- k, ~5 el numeru til' dc~vial'i.ones 'indepen.d.jenteil d~ .Ios HrJores lI~t'il;tbs rusPOC1!() a. lo~ e~pemd.os, Se ob_<;e'.r.'~l t~lIC, dt:bido a '~a If!Ola dH:\lClllo de l()s. ... alU'tt:s cspL'::radnN en 1 a tan I!l rle, oour 111- .l!n,m<l, partir de los lolak~ de. la., 'i1hLS y las C'".,nILJIlUl<l;'\, i'OO!lS las iBwiaci.0Jlt!l :sun id~n:ti eas. en Ill] mem fl bSQluw, 12.5,)' nllem,'11:1 CJ] ~u ~"11(}, de mal1t!!a que !n ~uma Jit Luda.,> cs cem_ A~ I, eJl reali(lad. 11'>~!n I'LJy LIM dc~,,'i;acion independiente. de m,Jtlrril qlle It::n~!-

IS'7

mo~ Udl rlnico grado de lihertad. Pur lo UIJHU, ~!guicnl{l{) la linea para U n g rado tie libCHriU t'L1 cl Cuadro 4- .1. vemm, q LIe 1 ::I r robabrIload tit" obtt'ner una desviaci.6n tan grande (0 mayor} de 10,," valures esperudos ~dl[l pnl' efectn del azar es 0.025 (2,5 ~1,). PUI;;:,l() tjuc esta pmbab i I kl8d es fnf'IlO[ (leI 5 %. 1::1 hi fl'lJlL:.':,~i~ cit:. ~cgrcg;.~['ion 'i Ildeperu:llente' debe rechazarsc. Una vcz rcch azada J a h ipf,re, i s ('!e ausencie de ligami;::ntu. il1fer9mct,<; qUI1: los ,1(IC"1 dehen e. .. tar ligado~,

Curtograffa genetica con marcadores mo I ecu I are s

Durante los prlmeros 7() :Iiio~ de In cnnogl3titl ge.tl~lit~J" los mare adores dis pon ih].el' e 11 I (IS niapas eraii gene!' (;01] \' arian Lt'~ alelic·as que producian fenoupos di tercrnes, /\ medl da qUE: aume nto n uestre conucimiemu sobrc la geuetlca de los organlsmos, IH.;l.... '0, rue el n tlrnero de genes qu e se p.o~11aJl uti lizar coruo rnurcedores. Sin embargo, incluso (;11 los uj"gani~mml ellyo~ mapus paredan estur «llcnos» de loci de efecto fenntfll'llcr; conocido, 't: t1em{)~tr6 q lie I ()<; in te rV8 ~O~ em musoruicos ~IILt\:" los gCirll'~ til> bnm contener enormes caMidtld~~ Jt' DNA._ Estos intervalos no se pHcJleforl caracterizar mediante al];:ili~is de I ignrnieuto, ya que S~ carecia de marcadorcs en dichas regiClnes_ hll';'! 1::1 (I bc-em-:i6rl tk Imlpa~ de mavor resolucidn. era ,lweci"f1 rh~pllrler dJt. un _gr.:1I1 rnlmere demarcadores gen~t.vJ& adicicnales quepcrrnitieran rcllcn ar lo~ hueeos ex istemes. E~HJ se rcsolvio con el descuhri m ie Ill\) de varies tipus de mareadnres n101eL~l1bJe~_ Un 'nUl~i:,udor 11\ .. lecular es un sitio CI~ ~I que hay heterocigosi» para ill~'lll1 [lpIJ de variac-ion neurra enel D:-.JA: e~tu C1>. qut' rIO C.'>La asociada a ninguIla vanaciou Ierunipica observable, CU:3:nd~\ se encuernra ell hetefO(: ig'!.ls i.~, r.:c~Lc doc u~ de DK r\r;. puede nriJj zarseen I~ C;]l1ogrlllJIa gcnetit::H11 1 gll~ I' que en ~!q nier pal' de flk:lUc"_ cunvencion at Como los 'll1lL1'I; .. adores I eolecularcs !>l~ pucdc n detcctar hie i lrnente }' ~OH muy mlltl..::m~(}~ ell d, .gelloma, rerilliten Illen::lr lo~ hllr;x:('Is trltre lo!> genes tk l'el1Cltipo cOllflcirlo ClIOInclo ~~ si11.l<lrl t.li d UlH~1 rnc~Jial1lc: alll:ili~i~ de ligllmiento, Obst':[lie' ~iU.~> pam 1-1 CtH1ugr-.:I1Ta ge· netill::lI. e.i' ~ig~i'~ka,dn ~~io11'f.@;icu tid mJaR'aJkJt moIocultlJ no es i.mIKlll;) 11I~, e~l sf ttl i::iJ rru ~ c:] I>dio de :hicK: r(1~_' i gC1Ni ~ e.~ ~i mp [eme nfe 1,1 il pliIIIU t"ollvcniemc de refereflcia que· !;-eri uti I para oj'i.;:ulal'~t: t'I1 d genomJi_ EI1 este senti do, I r)~ 1Il;;n"CarJuil!::~ !>L: ..::mplc:an CO[fl{) ]om; l11ojone" nrillmdus p'IJ~·ll.)~ \'h~CTOS en lo~ sig1o~ -anl.erimes_ Via.i ews tJ1u·t: no t'~~ab~m, i.n lcrcsad[l;'; en lo~ rrOp'i os IJ1Qj;one J;, ( luarC<IJJorc:s) j', sin embargo, :re h~I)'I'faH cle-sorielluldu s]n eUos,

Lnfo; do!> t.ip()~ baskos de £n<1]'caJort'!> nlnkCllhLte~ sc runua rnen[<I'1'l en las variaciollcii en 1aN diana!'; de lOll en7.ima:": de I'ewicd6n y err Itt ..::xisL.eTlcia de D7\~r\ re,pendo.

Ulilizaci6n de 105 po!irrwrHmos ern 1.3 long itud de los fragmentos de restricci61l

LliS crl:lima., de l"Cl'li.ricci!in !l;lct.e.mmflrs conal1 el DNA ell Sl'r:LlcJlJcia~ diana e~pf'.:dfjc,~i> gUle flparecen Je mant::11i alt~aLoria en el I )NA de OlIOS org~ni&moll_ Norm,aimente, 1a~ di::ma~ "e ent:lI!Cn'lcID ~n ]11. mi';!oima pmieucJl1 riel f/NA de jo~ doislink).,; ouJi\.ii..Iu{)~ de una pnhla,jt111; ~~IO r;::S, ell d DNA ue' 1u& CT'OTIlo:somas 'Ilomologo,>, Sin emba,rgo, (;'-.111 cit,rta J.'n::.GUcl1t:'ia. una. dian.a cmK're.ta Pllt:OC t.k:~.J1paIDOOr pm cau.,.:.!. de un<l IlUlIOlciClll neulra. LEI HHi~~Icion pbdr( a afecr.~r a IJ1 n get1 (l a una region im~r~~lllica, Si un .iruJi \'i·

X~L10 ~S lleterQcIg6[im rm<t ~,l presencia 0 aLL~.tJlC i a (+1-) de ~fHlial1a, podernos uriliaar e~te focus en la carltng'rana :genehCit Los .~idm +1- xe determ i rm<l con la lCcni,C"ll. Southern, utilizanda una sonda q!,Je deriva del DNA Ji: Taregi6n e:n 1<1 que se encucnrra la dLnlllL A C1iJltj IHI a.ci6Ul. sc deS'tri'he 1) n ejempJo npico:

,

,

HOIllOlogQ 1 ~lnm6Jugo 1 ,- ---,,--

----,,---,---

t kh

Un in hibridacion Southern Jlel. DNA de esee Imli. viduo, I'll. sflin(J,a revcla tre~ fmgmemos cun UI~ ulm.~i'io de 3. 2 Y I. kb, O~ro i ndividuo pod l'jf,1 ser homocigcill;k:o para el frag~lilt:nw d~ mayor tamllno "! inostrar solo b: hOlllda de J kh tras 1 a h ibtrid<li~,oJ] Southern.

11iomOlugo I.

,

J I.;Jy

_______ T _

hi otlli5log(_' 2:

------,---

3kb

La exi venc, a de rnriltiples, rOP'l.l'lS pilw 'L';~t{regio~l es 10 (lLiC ~D" conoce wmo. ~w]irnjj,iismo en La 10ngitud de los fn!gmen~~II' de f'\est.r~N:'Uim (]llF'l,P; del inglif\l, Restriction Fmgmem Lengh~ P(~frmmphism),

En un cruzamientc entre los dos individues anteriormentc descritos, siglliiElndo Ii! ley Jt~ ~\.!gr;:gaci611 rnendetiana como .10 harfaruos para cualqeser gt: 1:1, la mitnd de fu descendenei 11 mostrarta tres fr~gmen lu~ de restr~cd6n }' la etra mi tad nn unico fragmenw. POI[~Hrnto, uu RfU;JpueJc traiarse y s~tuarsc 00 el mapa nI jgual iJue c!U~klujel' ouo s,iUo crumosomicu, El sigulente ejcrnp To ITIlneS[!';1 Ja existenci a de ti g,~;mietUo entre e 1 R FLP de nuestro ':*'~1ll p to y ungcn heterocigotico, CUrl D en fase de acoP I amiento conIa (o:rma de 1.18 hand H~ de I y 2 kb clei. RFLP_

.D

2 .I T 'T

J

-----~-------,,~ -"---

J

F~9!Jr a 5~11Ii. M~].liJ de J.igill1JkntQ 001 ~r<:J.iTiQm!w I. humano, 'iO':rei.aci,"w!,) ~UiL su P"ltt0:ru de ~1<'I1~d~~. t,] 1i.i"Lo§rmn~ rn.ue~~~. la di"irlbudoQ ,~ ~, 1Ul' JiW["{:~OL~~ (Ii:~pnnibks J~~ comosomu I. A l~,I~lltIS lfl<m;~o~~ .>0J~ ~e.n~ de fc:mm'lln ~L)~(wjd~l, p~IJ'.i8 mayortn ~fl,1 ilillll1l:mlmu de

DNA b~~~dzy;. ~1\ ~~ia{·lU!I.C~fr.;~.trlL~ de I •. -, secuencius.

E~ I~ [.",I~e=1raJ de I~. fig~lta, ~.P=:~ U!Q mapa de li@~lui<:"JLLo OO~:!tdD

~D ~L :1J1<tiJl~is <& las f;~"I~ru~liJli I:I~ =~)mhin~i6Ii Jd IlpCl que ac ha d'~~~lli\1 ~D ~~li'~ ~8pi[~tn. Blril~p''' mucstra ·~Olo 3.lt!l"I~fJ 'k 101.' 11[11rrc\J.d;pte~ (!i~pt:111ib]~~.

Lm; c\i~l3nd8~ 4<1 liWIP'2! ¥kDI:'~ jl,ld'ic~Jr"!s t'"-J1 [;(JnIlmm~~I.ls (= .r1_1J,l

I.~ k~lgi IIIJ totul 061 maf.<ll M I ~tQI1L<)50!1J2i 1. C,; de 3~(,; ~\oJ; ~. lnua de;1 I",~'" I.<lrgu de ](:;; cromosomas hUIDIIJI1Ol<.

L@~ pn~ IL~I ones J~ pl~~I.l(}:i, 1li!~f·C"lIcI()re.s S~ b~1J ribd0ma;t{) L~I~" el di<lg!lLl.H~~

de la_ ~ti~I~giolJe~ del: CIX'.Jl.1L'l.';'<J.i"i!1i [ b;1~~iIb en ~l pa:tr1'i~ <!~.~tHJ~r ue bsmdas (dk:110 dm,!,'ilIm!J. red~ 1~ deJlomin;l!:tiO:ru.~e ·irlk'8.ml~l<it. G;;!~ Li,p;) de c(1lj"roW"I~ru;~n . ~~ puede ~~mh~e(;~r ~uk~ mediante hihr[JllclOm ~~ .fin. ~C~.f1. F~)·o "'Juili~l!; c!10!!.~n.ilim (t8p_ :17)_L8 r~~vurb de, lo~ m~adl)n;:s lM~w:mlq~. en

el m~ scm lTIol~~.l~n.~s. P<'w se .irujluYi!>~ t~Hlbi!!i; ~liL[]t$ ~~~ f@2~~~~~

en E;mll~ AI'OA~. <I,,-.;ilip!lpmb2,'I'l~: A(,T''f2, protefnn actina, f'Rr', ptU[tJJJw. rc~C'I~,,~ C; SP'A'U. PIDtt:'I1~ ~~p"'-1~jJmL. (l'o:m~do dl:\!'I, [i;, bSll.e~ etal., Sd""e~1 10 ~~ ~P[i';::11~I~., I ~g4J

[ii5trlb:ueioln .de-Ioci

Di!h"rids (aM)

MW-il dJ#

I igamltmm,

~ • •

PO~UI'fIO rfismoa 8[1 J\!l r~f'!gi~ud de secuenclas qCJ!las,

otros po~i rMrlislrT1!(ls deClr:-.JA

} """"",,. d. ONO

Genes

d

ESl.e .1 (lUciia una ;[I Jluelle locus I

a !J C G
J X
y L_____:)
I G Ci
C
(l
~ X
Co g I C G

Oescendc 1101 ~ ma~GU I i na

C t3 G G
P~rllnt~t
If y
,C G C .s
--..-
]~~rnh.JO.iif!18
y 'r' Figu~a S-,17. T .~' r!%>]~.II~rt"'~~ rt",JI iJtfpi[ilJ; entre los tli.i~ varnnus cI~

II!j n:nj;r bl~itci,gOtkm pura dos §~Q~, lil!::io:fns 81 ~r[f.m();',0m~ X;,.<~ pueden utilizur r~~ c ,I~k ~ lret~u~ci~. de r<lC,)m~i.!I.I!~i (in. 1\$', ~ jJL!'-",k reuhzur Jd

'FU"Jl!ff:f ,j~1 ~rQm~mH~ X ';'(lIJllJl[I~mjv 'p~dl:;r{t'~ 00 estc [JpO.

LI!.f~[lrrocrmtmrueJlLO& etHIC estossi ties darfan Iugar a productCl~ ltCr.mb1l'l:<' etes tll'Tel: tabk~ como D-3 y rl-2-] , De esra lonna, ~l [PL.P ~. pLlcdt· siruar en e 1 FHa~,," en relacion a la posi cj6n de ~hr! n '®e' ('itrn.~ rnarcaderes rneleeulares.

U:lllzad6n de los po limo rf srn os en VN'TR ~n la~~ rtograff aqsnetica

Il diJlf'm de tlllidfujt;~ repcn.ida~ en ~aii secue ncias ol'gruuurd<ls raJI'Klem es ~'ariablt1, Si nentrar, de momento, en ~l mucM:[Ii srne ~U~ ~~ ,e~t1t variabi ~ i dad, ~o rnrroMnent1l1 para 10 {!J LIe ues Q;,I.:qnc P'! e.<i~ fema es C111e-poJeHlu~ detectar alos jndlvl duos jJ.:"I.l~rgMtC05 )1t1[a elmnnero de rcpeticjones en. un determiaa;c Vffllt y 1,;~tC !";it:io en hetemc:igosis se puede lltiJizH:r como rr'JM.dur en cartogiraf( a genetica, Para ello, se neccsita una son:;l,SU~ Dli:tri(te con 'eJ DNA repcrido, En el ejernplo q1le se muesIr.t;fCAl1t] IlllaciOn. se utilizan di atm" para en 7,i m ElS de re &lri~ciun ~lkeman fl,jeJ:,t de 111 unidad que se rer1te. L~, unload basica mp~' li~3 It indica con una flecha

fl

, ,

. l'_

___ T~ _

llij;.~us (i~ VNTR formara dn~ bandas, una grande y otra pcqlI!nn,lID~ \o'E:7. gl!le se :aflliq ne 1£1 tecuica Soathern y se obten ga 'In.J uli'J]jlladinw.1fr~., U lin vezItltli~. d sitio heterocig6ti co se :~J$~dl±~1r en la ,,;.anl~gnjJf(a. tal] C01UO sucedla con c.:uall.Julur

IX:A d~RELP.. .

159

M apas de ligamiento par recomhinacion en la e specie human a

LD~ M:::re~ l:IiUIlJll:ll.tos, tienen miles de fenotipes que W.IJlt's~:ran hercncia ;mtosnm lea" y. aparen temente, PHK:OCl: rel all v arncute M)I]ci no ~1t11:l1' en e] mapa geuetice los loci de los gelle~ respousables de estos [enotipos, trl1lrl~eaJldoli'l s recnicas que se ha n {]e~af:mna.do en este ·cap:iwJo, Sit] etnbargo, poor variasrazones. e I progreso en la cal10graHa .Ie (:l~Hl;<; loei I'ue lento en ~11 ln lei (I, En pr.i,mcr Iugar, no resulta posible r,e~llhm!' cruzamlemos 'COIitrelados en lof.; • .;.ere~ humauos, de forma que lu~ gtn{:LilSu,l.~ DC, btan intenrarel calculo de las Jreeucneius de recerubinaclen a partir de lo ~ dilnbridos qUIt·. tJ", vel'. en cuan do. se f'i'mdr.ld " ~l de maaera uleatoria en lo~ aparearnientos, Adem{I:~. Jus t'.ruz~nl~entn~ eqL]~V<lktM:es a lets de prueb ... eran cxtraordinariamcrue r;J.I·()~., En segundo Iugar, la dcscendcncia de una pareja es norm<llt:rlen~l~ esrasa. 10 qu~ dificuha la ohrenclon de los ~<lI,O$ min unos impresci Tl'di'h~es. p'-l ra calcular d~Mneia~ de rmrplt fiable ~_ En tercer Iugar, el genoma human 0 e~ i n rneaxo, I (I que implic;r que JI:1 media de .laK distancias entre genes conocides !out! grande».

L_(J~ marcadores de [) NA h au ~·](Iu de gr<lIl ay uda l'lTl Ia cano gm:fr a ge [1 eri,ca de los ere r.nu's'lHina~ hum aUWN; 'i 1.:1 Fl gur;! :'i - Hi muesrra IlI.I ejcinplo.

A.lbinismo ecular

CElr1trdmem

QuinS8B· "Pol f'Qs,foqli()er'Bto \emima) ~-Ga I<;lGtosiQJ~s~ I ef'l~i fl'l81

- XJ'n fp rotema de b sa:r1Ig Fe)

Hi PO)(S nti n'a-g u a ni na hsforribosi I ~ra(1si'er~~iI 0 H G PRT (emim a ~

tip- u~oenmopia (u n tipode ceguera a I os COIOF~S rujo"lfElr,tl9)

. G~!f' D (etl7h'f! a ~

P'rot<lnQpi~, (u n tipo de ~eQ~Jo8 rlil ~ I ns ~(ll o.re~ ro jO-\/f!,rr;l~) ~+emaHII a A (f,a II~ 08 n Is GOBg"LH1'IIc:iC\ n(le I a $8 n me:'

Fig u ra .5·18.. M<lp<l W.e bi!-:umWl1lLl J~I crornosoma X hllinitl~_ uno "'"'

h'~ prn~J~I~ 'In!Ipa~ ~mm(!o:'{!Ilii,!cy.; ~ 'L,n,'il<;l~. ;~~ hi'~" ~'rn~itI1l~UI~ en ~I aJHiJI~U~ de Lu~ HF cl'iI;r~ l(l~ loc, 'I II oz,Ir,Wlli. ~JD J~ ullb~ .. itH;:kio (k murcudores 1]o10Icc~~JJI:,<

n r{,c" i(.~, d" 1w.!mulILi6:r. ue v6.l1ll1.rn; illl.n:H:iC~ (;iJF'- (}). Sc ir4i~ai!

I.,~ ~WQ=~. de mapa pl.'l)'J:[~L.'-!ilal~~ pa~. ro~ 1r..~j_ (']:"",,;;,11},J~ W F BuJ mer y L.I-. C~vaffi -S~(lf?~. ~lmf{llc:t, I!ll{llrl~irm, 1foOi Men, COIJ'YTI~lll ~~, 19m de

W, I!. F~~ntlhl.l lIOJ COLl.lliKlnJ. ~

un 1.1:1",10. ~ (:r.1 la de U III g~.do concreto tlie 1 igl:lmiC:fllu. pm' [lim, A cnflti,Ttuackltl se ealcuili . .:1 eocienre de la~ dos, pmrulhl~i(l1.¢1 ~iel[ldn 1.11 plultt1adon 1 ,M el logantmo de este valcr.Paesre g{g II> log.1fiJmj)~ rieoen UJ1.~ base exponenctal •. Ia jru nlLlfl:ci(irl Lad tiew; Ir Mi.t caractcrfsticadc q uc sc puedcn surnat las puntuaciones de ~~ rios eruzamlentos 6r1 b\,que sehayan util izado '1 os 11ti~mQS m~~J doses: y, rnr lo tallf(), suminisrran un OO1~~lHll'(l acunarlativc 'de da~ qnt: Hpuy~ltful 0 nu uti dctetminado y1L1u:r ~·1igamit:nLo. VU~'lJnus WI ejsmplo senci ~ ~n sobre como se h ace e] dlcu]o.

S~lpoflgWHl{lS que tenernos un t~mm<l equivaleme ~I CJlI& mieuto de prueba de un dih{brido,'Y qLK pDdeEno~ dcduclr rl ~pmlipn de I,,)~ g~JTle~Q(; pr1'l~!l1Cidos rc'T este individuo, de f\)l'lr~ q ue re~u I ra poslble detenmnru:: '8 i los gametes SOJJ 0 t1!O L'ecamtJiIl antes. £1 d i:ha'bl'io0 es tletemCigotlo,) para un alelo dOiLlUa,1L1te que dl~lt!ffi1ill;l eicrta en fcnnmutd nercJi tari a (Dh:.fll' ]1lU':l. TIQ mal'C~ld{)r 11Ilokcula'l' (Il.~I/M2). As umamos t£lmNel) que es un ~ar6~1 y que I os h:mmews que se fnsimlilwn pam rormar este ir.l.dhiduo eran dit:' 'ge.rwLipo D . M1 :t d - 11.12_ S~l ni.ujer esdid- M21)12 La Pi 1!1.Lr~. 5·19 muestra un pedi gr. em cl quese pUOOe1l1 ot~,ar 1 C1l': ~ei;!; h iJ().~ de 1,11 p1l'r.;.j'8 .• caracterl mdl"~ en fnllcl<1f1 del gen(ltt,., de lOll gameros proveniemes Jd padre. De los seis, dos SUIl 1'0;:(;£11- hilll[[i1t~~. 10 ql:lIE d1liria tina R.F del 33 % _ Sin CLTID.nrgl). es pl.Miol. q Lie los dos rnarcadores ~gre:gL1.e Ili i'll dependieneemente y q,ue I~ hi.i\t<) represerttert una. desvlaclen alearorla del !'es[Jlwdo esp:!';lt· para esra CDmlk;~6n. C<ilcuJe.rnooia probabiHd ad de es~~ I~&HJ~ . conai deranda ~'ar~:.iS .hLp[ltesh. A eonrin uacinn, 1'IC mdicun ].lI;S pm p('Il'Cio..nes esper~ de los gcnotilJ'O~ p~ renrales (P) y !re.oom hj!13n~ tes (R) con tre~ \'(ill.ores de ru i y CQIl segregaeien iHdep-e~djf)jl1e,

D'M1 I:IM.2_

~ 7'

• 0 D,,'d M 1,'tol ~ t dI.d M2/M2

[5



o

1 {),M1 P

3 ()·Ml F'

s D·M2

®

d·M1

®

d,M2 P

Fi 9 u ra 5,·19. P,;.:t;if,1'f ~gi!h~I~!I i~ I!. nn 1~~ijtA!(n ;"1;[(1 M I'!l!~ijli. INri

SI '" ~ 1"1,,, de Ull g~J1 ~!~(lU1LllJQ eou UJJI( ~]Lr~:rHIDljll.J; .\1.J y .\\12 ~l>il «uletusrnulec J I"res_ pur ejernptu dill Inrtnns de un ItPLP_ p. parcnrel lno recombi r,:I1I11X:):

R IJC"~Cl"lt:-ill!!t1~J_

Cartograffa gerHl1tica del cromosema X

PI G.IY'I1r!(1S0!nJ' X hl.~m~110 ha ~klo siempre ~YI.(iS accesible que II~ aulosornas ula ;';flI~ugmHH geuetica mediante Hm:lli~~.~ tic n::CUH~binacion: y_ de hecho, cl primer mapa que se ohruvo de un cromosorna humane fue el del X. PSlO ... edebe J que Josvarones SOil hem fcig!jt.icos 1)8 ra los genes Ii gados al X. As!. al igu"lI. Clue \,1- mos para Drosophila, CIIUIlLlo obser v amos l!Ilicamcnle 1.:1 JCKccndencia ma<icuHna de una rnujer dHlihrid:l estamos 'lllaJi7.1:n(io 1<'1 constlmelon geneticil de los gametos de dicha m~jer. En otras palabrus, Lcnl:HiU~ una s it~L:;lt·i6n rn u)' aprt)x i mudu a 1::1 tlcl cruzamicuto de pHI.LOa_ Considere la ~ittiac~()ll ~igI!lLe;n~e,cn 101 que estan rm fl·1 i c:a rlos dosalelos reces lvos l'aJ'(}~ Hgilclo~ al cmm()~oma X. W10 provoca incapacidad pera metaoulizar azucar {M) }' ~J etro, en unlecus distinto, ccglLC:rEl 11 1D,~ celores ~cr Un va[{)[1 afecradr. ~ie Olin bas c(lnd:idfm,e~ t c glY]. se C~8~; CO~l u na mu je I' normal (que es. casi COn wall cerreza, C Gle G), Las ltij.l'l:& de C':i[4\ purcju ~O[! hck~:ljdgulit!~~ en tasc de acoplamicntu, 'i ]u~ hijos varones de e~t;l.' mujeres ofreceran 0). ~'n~ g~net.i~t.1L~ la npor· tmli~1ad die medlr la frecnencia de recomciaaclon en ·I.'li! meiosis de las ruadres (Fig. 5-1'7);. UI. Fig-urn 5-18 muestra un lIIl21Pfl de al gmlo~ geue~ J.t' 1 crnmosoma X humane qU[l son responsubles de fenotl POK Ii gil.d.o$ 01, dlkbocf"Ort'!0l':Olt!a. Tenga en cuentaq ue los marcadores moleculares tam bien se pueden uti liM I' eli III COifLog:mha gl:lIlil i.CH del crum~)SQJ~Ul X.

Puntuadon Lod en el analisis de liqa rniento en pedlqrfes

HI e~C<)!':O ~time:r() de. de~ce:l1,diel1~:;' en 1,1$ famm~", hll.ma:na~ lle\~8 i, In P I k ii() 'l~ j m ro~ ibif i d~,d de de~rm i 11;8 r~a exi s~enc.i 8J da H,Q:amie!1lO ~u om.!; a ~In Li.nicu ap<trearui~ll(), Par~ ubtener ,\!.'llo.res Jl abl~· de RF, ~e nlilce~ita!n LamaJiM~ de mue~hl! gmndc~. NI) obs,,:mte., ~i ;;.e cnmhi 11a1l lo~ re~lll:taoo~ de !lIlucho~ ~plll'eami.~~t(} .. \ IdentKos. se plJecle obrener nna ..;'>.>tUHiI masflable.La fCIJ.'I'Il<lusual tie fll{>t:(:1 csw t~ mcdifdlnLc ~] d.l>.:: LI]O de la~ plIIillruad:o:nes lAId. m termino Loci es u.n_ lIc~lnim() de ~a expre~i!~ln. ilmg]esa Log Of Odds (Jogfll'itmo de his prdb<lhilld~des). E~. I.t\ewdo cakulOl simpl.e:lllent-e la prooohilidad de ~lfui[Ciit:r H[J c(JrnjUlito de rcRu~ [!lum en li rl!l fa[llllI ~:l. h:L~~ind(1~fl en hi ex ~~'t,enci a de '<egregac i n~ i ndependi l~ntc, Imr

lRF 0_5 0-4 0_3 0.2
p (US ~3 0.35 UA
P 0_25 03 0.35 OA
g 0.25 0 .. 2 001:5 0.1
R 1.),25 0.2 OJ:)' 0.1 La VTI'obahilfd;ad de ~h,t.eller lo~ !;e~l]ltadn~ hahi'en.dn se.g:rr~j cioll indepe.lldie~te tRill del 50 %) semi gUl'l1 a:

0,2,5 x 0.25 x 0.2.5 x 0.25 x 0.15 x (U.5x B = 0.00024 Y!l

doude B ""nt'hllero W~I tfe poslh I es C()m hi n at: iones p~1l. cl ~;:{i mieuto de cuarro it'lslj vid1i.lliJsp.11'ellt<1J es y do~ recom hi nantes_ Para HM. RF de CU. la prebabilidsd es:

{)A _x: 0_1 x 0.4 x 0-4 x D. I. x 0.4 x B = CUJOOQ6 x B

POf ranto, e~ coclense de lo~ des e~ O,0002()PO,OO(B4 -::; I, ,OS ({~ serve que H se el:im.hla). As.1. elll fllllc:i6.iJI de-elitos dams. 111, lJipfre. .~is de una RF de 0.1 ~s I. .08 \'tx~~ IIias prolKlbk lj ~ ~.'I da L ~'Cgregaekjn. i.llu"'pendi.ootx:~. A contillul~lci6n. &1: IIlJU;Cl:>:IJ!1HI 011. cooiem~~ y ~U~ YalC![r-e.~Lod:

.R.F 0.5 0.4 0.3 02
Pi'l:lb,~bil idflfl. 0_00024- O.{l{)(l32 O.{)()([l4 CI.UP.{Q'
C04:.i.emc l.0 1.3'3 1041 1.08
Loti 0 0.12 0.15r 0.03 IF,M.o~mjm~!w~ r;:Oinfj f!11l am in u.e;<;;tra~ ~n~pech a~ inic~al,e~d€ tjlJ.; L &l:f' es Iii arL[rt: 30 y 40 %c. Shl (:m.bargo, 105 d:a[Qs LlO propmdDw Ull arwyncm'lvlllCenle de hi exiMeneia de ligam~cruU!i. Puf~'('

ren.it'll. S~ eonsidera q ne b ~ ~nlHtlWt!io.rJcs Led de :I dererm ~ nan Jlla[:l:~'I1 corrvmcenre ;J~ ligamicrun enn un valfH' de R I~ C.aUCfCJ:'. !)b~,'e (]ue· lmfj pumuacirin Loll de :1 repre ... enta uu valor de ~F;ID~ es WOO veces [estn es, IIY veces) mas probable que la 11\l~~ib oc l:ilJI~el1d~ rle ligamieruo,

Namr-aleza del entrecruzamienro

L,'liib::al:le q,ue lo~ reconrbinantesintracrcmcsrnruccs se vril:(ina:i:ln po:'.ra~llll 'lip~J Lie iruercumbi« fie marerint entre cromosomas l.'l:IiJlllg_oS eru convincenre, No obsumte, cstaidea debra ser ~;.lIiIgrul'lliclll con al~tin experimcruo. Uue de IO:-l primerm pasos ~:Iflqqie en correlaciunar la apari eiCl1l de recombinamcs genetiM~nli Im~rtaltibio~ entre part<"$ de cromosemas. Varjn~ inves:1[;i.l:i1J,il';'> riguiemn I~ misma ~'~LrnlJ:.'giH para abordar este pmbk:;..1, IEIl I~ll, ltarrter Creigluon y Barbara .McCHntoc.k eslaban 11I1ldbf1d~) dos loci del cromosoma '9 del matz: !WIIO IIfccll:lba al ,~Jc'deJ..i semilla tC', coloreado: (", [10 coloreado) _¥ el otro afer.Jb~ a b bhmr(""J,o; le i Cit! tie.l endospcrmn t W~1[, cerosn; fi~. feculenk,~denlj8. el ~rllIrJ'osO(ml que 11~\'·ab'l (' y ~b era ii:lU~UllL yu -f.t '8illt'nill UI1~ protuheranc i:. _~]'a 11J(: y leiiida intc nsam ente en ~I tXII~rnu cercano a. C Y WI .. I regiun crornosornica mas larga en ·1 nl""lTIp. donde se encontraba Wy: as], ('I hie[erOCJgom era

c

w.r

c

rrnt1,) liil.lmpw·amn los em moso In3 ~ de Jes recombi nan te <;, gef1iJiq,l~ CUll ~q~ de I a descendeuc ia de ti po parental, C I'elgh W n y M~CFIlt.')ck eacenrraron tjUi: ioda la prngerue de tipo parental 1l1111i:lJ,"O la o.rg,lJ1iLa·l'i61!l de los eromoso mas pare;Htak s, mien It.!,; q~ ~k'UIl~ ]~J;'; rceombirmnres fueron

~esltlH:lJ.lem, n:lai.:iou:ln:m el feoomeno genetico de la recorn'in~~jL.i.DLrd.crmnlO"ii'i'llIlk·a con el c.iw:[6gicu. Los {lll·ia~ma;:: pa~JO [t_p.D.:,StnUlir ·1 cs ~ i uos (jel lntercambio, pcro 1 a prueba fi 11 a I ::~~ on Ileg" O'IM<I W7K

.: I:uil ~ el mecanisme de i ntercambto cromosomi ell en (' I. en~al1ll~t1lu?· Concisamente, I~ respuesta es 4llIC d entJ'OCI11- 1lJHicli~) resulta de un proceso de rotura 'jj un ion. de los cromo~Ill~', 1~ crOdllo~U"mtls parental es se rom-pen Cil Ia misma ~;kiNI ~' vlld\,Cil a uno rse fie IIllle,vo ell dus cUlIibiIladO"ne~ no ~1aJL'::l. En el Capitulo ] ~ t:sHldi<lrumos modelmc'le lQi': pro::sil' 'i'T:Il'llwlJj"res L]ll(: perm iten al lD}J,<\ rompel'M~ y lI.11.ir~ JLl flllllll [no proci:sa.

COROLARIO· ...•........ -. -- - .. - .

los crornosomas :re entr(!Cri.lzan rompi8ndos~ e,rI 13 mlsma posicion Y uilhliooose en las dos ccmbtnecienas ne Pilrentale:s rec:ipnK<ls.

Oh5e"\'~m que, en nuestra rcprcseneacron esq!le.II16tic~ del elltrecmzamiemn en este en ptrulo, bernos Jll ustJ:'dt.iU que f~I..OS nell rren en ~a fase tie custro crornatidas Je: la meiosis, Sin ern bargu , no es p()81b~e distinguir est", posibilidad del entrecruzanuento en 1;1 b:1~~ tk' d(,)~ crotnosomas tan ~olo l)ledifW~~ e.~ e~ll.ldiu de Ius _[JWULaClOS alearorios de recuiubinacion de III mcio~i~, como en un cruzarulenru de prueba, E:'LC asunto quedo resuelro ;J lI<1~'e7 I.Jd ,;'llliili!;I" geueliw de organismo« ~LJy(l.'i productos nll~iuljco~ permil Ue0t"[l ju n LIJ~ en grupos de cuauo llamados h~t.i'llI-dru. •. L1; los (11' g:m isrnos SOil fu Ilfhi I nentahnenie hn; Imn;go); yl as algas u nicelu I,~res, En el 1 L)~. Sc: puedr [I aislur ·10:;; c uarro productos m~~6{icVS t"u lll'~l ~ilJlil1~I.:radtl, to que es equivalente a aislar las cualru rrcrruitidas resultanres de una unica meiosis, El ami]i~i~ de tetradas en los cruznmlentos con geoes ligaJ!J~ rnuestra con C I aridad fill e. t"fl muchos C·[[6'D8. La~ L.clrm .. b:-; contienen lo~ cuarro genotipos d:DkD:nL.e~ r elacionadnx con estes ~oc,; per ejemplo, en cl CTlJZ"IIIT~CIll'O

Algunas LdmJa~ presontan los ~l]'111'('1 genotipos

An Ail IIR ·0 b

Esre resu uauo puedc cxpli curse tinica mente rorqlJ e haya ~t·lDid{J 11.lgar un cmrceruzamiento en la fase ue cuatro nom:iJ.ii.la~_ ya q~le. si el emrecruzamiento hubiera ecurrido en III. fasede des cromosomas, tan 56k' deberfa habcr dos genotipoa diferellre·<;, en una meiosls ulldj\'idu~l. eomo se muesrra en Ii:! llgura 5-ZU,

EI fltn'Ui5i s de tetradas fad I: i rm 1" ex p lQr:1l;iurl de muchos otrm aspee lo~ de l a recem bill aci611 intracrorucsomjca, que sc t,....n~ l-an

Fase de dos fi lamcntos
A b
A. B .oil b A b
....J ._ - - e
a

{J b F,J B 1'1
Faso do cuatro fiiamantQ~
A e A e
,..1 8 A b
____.........I\....~~ - B
a

8 b <'I b
., b F·igura S~20. r,,1 !lm1tt~il; de UltmulI£ proporciona ~ .. i;1rn"ja.l qll<! 1'£~I'l)Il~ll ~ l,u~ genetiytll~ decldir sj cl Cnrr)2l:'ru7;'llli~111\) ~11~~,d~'I'JL Iu.l~l~~ ue

OO~ lil.!lnlentos rdo:<. i:<mm"'''1IH,.,j'' ",) 0:1 de ~WiIru J1lilmenio;; ~(,U:lltro crlli!l.l§ttdn..<;'j. P'tJc5La 'Ill>: ell ;l·II_\llIlh I<!-Il ~ >~. pl!~en ot>.eJryar· m.!i~ de OO~ pmdlL.::[c~

c\.i .. ti'liO, plA..>;:~deDl~~ d.~ un.~ win. ~1;:},<1~. ~I ~1~1"I'eC"JJjZi.lil.lLeIlIO 11.0 'pLI"'tl~

:s.IJn,d~ ThL1tliJU. Ctl I D, fao.c d~. ·011; irl ~ Ill~flll)t, ..

lQ

c
b
_.....
i'l
,I ts ~ I,J tJ L' Fi.g u ra 5,21. TJIl" Ii<: lns vari ns li(!'C!~ Ill}:<i~I~~ de 1':;1I'8i1~~ il(1)I.;,!l recumbmanles que se ~F~·il.Il wD~gubriJiIJ, Ob.o.i:'r~'~· ~JU~ -~.iilrl i lil]J!k",dil.~ Illii~ de ~< ~mrn~.tida~ i RtDO'C'ambiando ~~gm~ti)s_

en dctalle en el Capitulo 6. No obstante, de memento, utilizarem0S I a ~ rerr<'ldas para re~ponder dn~ cuestiorres fundamentales sohre t'l entrecrn Mill ie]1lr{'l" La prl !nei'1\, (, puede hailer mti Ir-i rl:e~ L:IlLr<:CTl1Z~lT[iC[l[U~ entre trlrL~ de dos cronultidas? Para responder a esm, tenemos que ohservar los entrecruzamiensns dobles: 3" rlClq estudkldos, nec_e~i~amo~ rres genes Hg<ld.(I~.,. ~or ejenrplo, en d {: ruvum ~cn:w

ABC x: a r, c

pu edeudarse muchas t:etmdllS disti ntax, pew algunas dc cllas Lirl-i!.:arrLL-111LC ~c pueden explicar con eutrecruza mi enlm doblex, Cousidere la ~Ctrada siguiente como un I'ljemplo:

ABc A._ll C alJ(_' abc-

F~t.~i t.etm.:la se puede cxplicar por do~ heehos de cnuecmzamienro que impli.C11111 a dos croruatides .. como ~ mucstra en hi

l'lLgm<l 5-21. Otros tipos (Ie te~md.@s indrcanqueel tot<'ll de I~$ cuutro CrOLT.~(LL_rJlH~ puedcn partieipar en emrecnrzamienres e~ I. rnjsm amcicsss. Per ] o tanto, las crorml1:i das (P_W tomen pm-rc tJ e I. fenomene de entrecruza III ienro en un a ~lIi~a me i osi ~ pueden ~ J os, tres -0 ~ uatro,

Si tedaslas cromahJ~I~ participan en t:.~ procese, podemes pLI>' guntarnos ~ i, hay al gun ripe deinterferenela dt' c:rom~lidlls_ ~~ orrss IMl~bfa~. tel q:~le se PH:.dl-lZC'1 [In el~tfecn_17;;lln!eIlW eI1r:n; d:o~L'I--oI:I]atiuilS no liermanas ulecta a la probHO]licl,\ld de q lie ~~w cln~ cJ:[lIn(fL;idtls i ntervengan en olt)'o emrucruzaJl:n~cmo tic I.!! [r..i" 1118 mei(l,~:i~?' EI an.Hi~i~ de te~r~:la~ puede reNpcmder Ii estu W gl1U'l1'l Y Uiem.LLtstl.'.'l. qllt~ ]8 disrrib LJd6f1 deentrecruzamieutes ~I]treIas CTmntlt~Jla~ W:::UI:lrC nermelmente HI HL.ar;C~ d-L:.o"t:il', Ilol),;y i nterferenci a, de crorn utidas,

Anl:e~ de fl nailzar nuestro antillsi s sobre l<l impl ic<lci 611 de fJ' crtnn {ltid.a-!> en los cntrceruzamientos, rnereee la peua 4 ue. nu' p 1 urnecmos mra pmg-lJIllt.'1.: iJ'ucdc haber cmQ,x:tLlzlllL1i~!:Ito rlf..~ Cl'O(n<itijilS h ermanas? En <I'lg un 0'3; ()i'f!:<1:nil srnos, ,"C ha demosnsl, (]U~ sf se produce cntrccruzasnicnto ·!;)ulre ~-wul;Uid~lS her1TI~ln1;, si~ embargo, puesroque es:tJ) flO da ~ugar a roc~.mibi:illLnl~~, adcmi'is, i10 esta clare Glue: ocu rra eU'l todos k ... ~ ~}rg[:Hili::sr.ntl~,. ~I normal e~ noi I~Wt~m t'~L,t' tipe d~ i_rllie'tcamb lo el.l IQ~ e:'lq!J~ITI~ q ue 'I'epre.senW:rI entrecruzarn i(::nk4." __

EI eurrecrezamleme esun feJlb!iIlienO e"r:ra(Jl]:lil:!~ri_mU61j_1\.:-PE ci so. La sinepsis y el interc arnbjo cromosom ic~~ -oclJ;tren de ~ manera ·que- I'm se pi erde ni se gana j11llgUIl segmeneo, y api$~(1;1 cu aIm G I'Qmo~ol,na~ cnmpletos en ] a 'ldr.1lcla. Se hJaprelld.mucho sobre I=~ mlnH~ leza de los rrnde.NO~ rnoleeulares que Iii ncn lugar ~11 los ~lli:O~ ut entrecruzcmie !lIro y <: n SIl ~ [lTOximiJ~ des, )'101': trararcmos en cl CapHlllu ~D.

R.ESC\lEN

'.,,~_L! !. "!! I. ~" !!' "II! "'. ~" !!- .. ~ .. !! III" ill " .. i [Ii .. , Ii, Ii i .. i iii i ii iii Ii Ii ill"', iii Ii .. I .tii Idl • Ii '. iii • Ii ' .II .. ,Ij .' 'II • ,.. • !Ij I '" 'I! II !I! ' ! I! ! !II ! !!o'l! !J"II! ! !! ! "" ! !' ! .. '!! !" J <II ., Ii i." III

\Villiam U."'k~Uit Y R, Co Punnert descubrleron deaviaciones de la pruporcion JeIluL-ipic<1 esp~~ [.IeI'l, St::\: 3: 1, ell I <J g:<: neracion F z de crnzamieruos dihfbridos con eicrtavuriedad de gliis~nte. Los t i P-I:I& g<:l'Ine1j,ooi5 parental es .;lparec r an err Ul.1 n(lmem rnucho rnafor qll'c- las otras dos .::1 8 ses, Con pnm.erioridad, Thnma~ Hunt :vlorgall ennmuv LLlHL de s y lacion si m i ~ a L' de lil ley de la ~eg.reg;a-'I.: i.6n independierue. de Mendel. ('~l ~ll l<fil.udilJ sobre des gen es ;mtmO!ll!cQ~ rtN·unk.l~ de Drosophila. M.argan propuso quelos dos gear ~ esrabau sJW ados en el mi ~!llf! pm' de cromnsom us no IJL6'iogo~, E.~L't: li:H6mcno recibe la deuominacion de Hgamielw)..

1-: I II g~ III 1<: mo. ex plti ca hI =6n pOT 1:1< ,que lll~mmbi[tn~;{Jnes f1,;i l'er'11ilJe-o;· <Je ,11 e I ('I:': pe'rmllll'0ClI311 jun ta:,.. pEru no ex pli:[;~I, n1m~) np~jr{'JCell l,\~ COHl b i n,1C i on.::~ n11 parentale;o;,rvl org1li1l prnpu~o que, durantc h mei~(j,is., pUlccJe- habe't uu ttUe.rcIUllhio n~fc() de fr!ig. melltos cmrno~omico~. mediililte lLl'l proces1) qllt' en ~m acn:aali(I ¥l se def1('i ill ~ 11 11 entrecrnzam i ento. P!)r tmllO. exi~ten dos. Iipo~ de r,ocoIllbin8c.ioll mei6t.i:L;I .. La l"ec()mihl n;lc i ('j,n debi·tla a la >,>cgh> gat: i ()~ 'i ntlcpcmli.cll.Le nn:m.Jejit[u'8. l.L~lk WI.HO re-suUfldo Hna frecuenc i ~ de recomhimm Le~ del St) %, El .t::L~ lrodct~I.!.m~_Li.;':'l]tu d.a IlJ-gar a 1m3 fi:·e-cuf:ncia [k r-ecf)~nbi!l~nte~ meJlOT del 50 ,%,

COil fmcmc t'S-tudi,Q W~iI genf';~ Ugaclo~. Morgan d)escu brn6 1m [Ie na va 1'1 nci6n ell 1 a~frecUiLmCta~ de rt-\t~ornbinantes y 5t~ pl'egnnt6

si esta v~iti~ci6rn tell ejaba In €Ii sra cc i ~ reOlI entre [O~ 1l~1l,e~ de II cromosome, Allred Sturtevant, llf! ~,sm(lj8n~-e de Morgall. d;:t fin I J ou n mewdo ba~adn ctI c J porcerusje derecombinames ptr:! determi nar ~a dhta nci 8- entre get1le~ en un mapa dc~:Lg~md.i.1 en IIW.p.U de .ILgami~llltl es t'I:ro ejer,Hplo de tnr eute hiprn_&'f: q LIe ~ 1I1-gC tid !In~Lli~i~ _ genetico,

til forma ma,-.; ~ell'l(''i.lla. de dererminar lss fn::~ ueucias dr I. l1'Clrnb~llilidCin 1;8 mediante el eruzarniento de prucba de uu p bride 0 de lin ~rih(b.l!'ido>, r.. 0 obstante, Ills fr~cu£'·neia~ -di~rcl~ bill1tcl{in se pueden dcclu.I!ir rambien a partll' de q ruzam i;mL dilubrldos,

All.lique ~a f)l:lleba M$Jk~ de Jigan1i1i.elll'{) ~~ ] a. ,dG~ ~-ia'l'i!5[1_ J~ ~ p:roiJ'!J[''!..-i.6n ~: J: 1 : 1. emrel fL descende Ilda die IJI~ CI1l;!':1tYlj'.J.LlllfJ, r~lJ~ha, tal dc~wi.:!L[_':iorq!ucdl: E10 ~t:r 18111 ob'"ia, La :pflleb~df;{ Cflle 11m dke C-QI1 que-frecll~ncia ~nK vlI]·mc!' ub~l':J.-V!:ld{)~ ~ il!' ... ian de los eSI)era(lfI,~, ~i11lrl'e1l1ent;e pm :lz,al'. 'p~D& a)'_ljcl~WJI"" tiecidir sirn1Y ligil1lillemo (I nlO,

Se hlln IlIj}Lmtadn valFi~'L.' t{'''-or.l::l5 pam l~)tpIi(ilr c6rn]CI se gene:z 10& cro mosbmw; I'eC(lIl~~hE llIal1t\3.,_ Hay _ ~~bCl1.m~ que d em:fWI!I :";;'UIlICLUlu rl"~ll!ta dot 1<'1 rofl!ra y uni61l fffrka~ de fragni1ei'ltiilS ~7l m (k'f'lj.,-nicm: y tie1:1c I L!g::fr u'uralllJk Ill! hL-~e Jt: C lIillro CilCln'l~tid:ll ~ 101 Illeio~is,

Ln.~ varial:iunes neurras en el DNA :iC ulHizal'l ell I", actualidad :r.<Ylu .I.lJtlllE de marcadores para lot cartf.:lgmfl,~ cnunosumjca.

Eo dam] 1 j_~i~ dil:!· pt:digrit';s., cl tama no de la mucslm e~ dcrna-

MAPA IlE CONCEPTOS

"rano P~(IH~ill_l como para permirlr [:'I caJ'log_ralTa gClleLicll_ No ()l1<qWII.~. In acumulacion de datos, expresada como punruacloues Loci, puede demostrar k~ existcnciu de ligamien(('!,

II -1;'. ~';I,. ~ • tJi!!li .... ,. •• Ii '!!' "'! ! !!! ! I!' • I' 'I! II 'I! ~ it! I' oil Ii ill i ill •••• , •• .,. ..... ~ .. 'I! 11'1; I !II! • ~ • it. ~ iI-,jj i iii .,. ~ "" ! • I •• I I' • I'll .... It! ... ~ 1I.t! ..... Ii I iii i • Ii ill •• ,. ~ !I!" II!" Ii ' ... ,I 10 -Ii .. ill • .Ii ~ ... II Ii .. ri iI ill Ii ••• ! ..

[race: U~ I\)klpa ue concepros, relacieuundo entre si tanros de Ju~ ~ig\!ltnq~ L~rmir](l~ C(HllO le ~t;<l plJ sible, Observe que. los conrettc1n.u cl'=tam en J.li:Llg L111 LlrJCL1 concreto,

V&OOf.!EMA DE INTEGRACION DE CAPITULOS

entrecruzanuento i \:roma~idas / cartng.I'<II'(a I cromosomas / cruzamiento de prueba l tlgamiento I unidades tie mapa I reccrnbinecidn ! ,adfrivld,ld l seg[cg:::K~i6u indepeudiente

Ifo .. i •• "'II."'I .... 11i ill • .,., ••••• ,., I!'.''''.'' ilhlli II iii Ii ........ III! .'."'! ••• 't;! §-'" .' •• of; .. -tlI.,.,. ••••• I'.! •• '!'. I II,.. I- 1·1;. "',. ill i iIIiiii iII •••••• -I!.'I!" "'1' .... ,I •• ill 11 ........ i .. i .... , ••••

:!il [)m.~orlll1'h~.los :ilG1D .. s P Y r detenninan ejos tie color m_lo y pilf-lnl, re ;ptlcli \' arnente _ 1,0£ alelos B y b deterrni nan cuerpo tie l'11~' ImsrLm }' negro. respecu V[,:I110cnle_ Un genetista cruza un a Iltlllhr.l de 11Ili'lllm:a pura de nJos ptirpufOls )' CUL'IpO marron con In :m~b.o, tambicn de un 0 HIl~J pura, de ojos H1jo~ y cuerpo :C~11.1, 1ijd,a.~ LIs mn">C~~ de la rl ~icnen ojos rojos y cuerpe tria, '1>in, tmwndo entre s1 b~ rnoscas de 1:<1 Pl,tl gt~llclishl obtiene LI I'!' E,<ta J ~ CC1llrPOr.J.t'Tl: ] os mdiv lduos siguicntes:

marroues. rojos 6R4

1l'~IIl'QIl.eS.. piirpuras J43

.[l;:;gmii. roJO:': 34 f

uss

\Ou~ O:Illi .. 1e11l estes dateseobre la localin~ci6n cromosomica :k Irt,t gl~~~?

* SQlud 6n t,

1\lte..~):i!lnQnobfe la localizacirin cl'omu:,um-ic..'i, lPIli:iIe.~ de los t~II&nefl{J1qliel:u.:::rLl()S e~tud18d() h"~ll;l ahora vienen a ]UJt":~m~ m::11If'{ f.t1. pillIlU luga r, ei)t:i. la dis lim:i.ull entre herenc ia illlllJ~(jmrc~ 't JJgili±:r.lIll'lexo. Luego c~hliiJllll~ poslbilidadcs alLermttiv;3:;' ~ 1~~lTIlento Q segrcgaci 6ni edepeedieate .. f'\() tenernosmucho lIla; ~Ilt nm; preocupe en. este mome [Ito, ~ C6rno podemos des pe':1f~t'l~ ]ru;0gnita~? Empeceuios COIl el ligJ'lmie.tlto al sexo, La I;'lr~l!t'riHt~r:Jj distiutiva dulligarnienro <II sexo es la aparici6n de 1111 rutrDllIk hereneia que ,!i.e relaclona dt: alglllla manera con el )rJJ.l, ~~iTe:-lj~tCtL Pfll<!!fJ<lS de ello en auestros datos, Asl 'q lIC, t1l:ldo qUI' RQ rec ihLmo~ i nformacion en otro sent ida. debemos S uponcr qn: se ubtuvfI ,e! inismn ruimero de machos )' hcmbras de cada rl~_ S~n enluargu. hay un hecho l'ehK:iUllatio con e:1 sexo que deJo...':If"~ "t~n::I ell cuenta siempre tlLUC tratemos con I~ g~1l6tira ikDi'f)1~lphllat como hemes visto en este caprrulo. no se produ~n ecrtr~:mU:a.rIl:UeJlt()S en Ias meiosis de los machos de Drosop,\i/I,l DD rnomemo, mantendremos esl:e datu en la l"ecamalJ"J.: )' m ~U~ pocIrtl :>tm(]~ 6tH ITh1.~ HLfdc_

AfJ qUI: Ml1i.OS lI~gado a la GnllcluSi.Oll dt: t)1lL:' ambm; gene;;; ~ 3uul~r'imitoo. AhOTa poodenws tew~~ tm:ir ~()~ em;>;" m ie.llws ~lIir.1".:;o[';OO dmbnior.; gelletkos, para H~lvcr ::I, plallt:eal' to qUI:: ya tIIh>cfl};l.j_ pem ell Un [orm .. do liger,amente ,distimo,

r

!JIB ' pip x Mh - PiP 1

F, NIb NIl

BIb ' Pip x Btb - PI!,

~

F'~ R!- ' pJ_.

m-: pip blb : P!-

Ahora vemes con mas clarldad que liencn deinusual e~tos rlatos. El cruzamientn entre las iil0SUL~ de h FI es un cruzarnieuro dibfbride 'j, de acuerdo con b Reg~rllda 1 ey de Mendel. espcrernos que dos genes que xcgrrcgan lndependientemente den cuatro clases l'el1utfp'nca.~ en. ] n F2 con prcporcion 9:3: 3: l. Aq u.( teneruus ~lJ.lo tres clases fenNipka:;. y en proporcirin 2: 1: 1. Ciertamcnte, algo Hlli.~ ~~hl ocurricndn, La clase que ralls, cs la de genotipo bill ' ~llJ.l, .Hsi que una COM chrn es ljLJC ocurre !lIt-'ll que irnpide III aprrn.ci6r1 de esa ctase

Recordando 1 <I exposici on xohre Jet.') I i dQ,d del Capliulo 4, podr! amos es pecular con la idea de que el ¥t:!lU>L~PO bt b . plp e~ I,etal. Sin embargo, eso e,o;, poco probable, porque nom deiar~<I' con la rnll.~iGiIl 9:3:3 de 101 propereion espcrada, que puede rechucir~crc: a lIIlLIL pmrmci6n J: I : t 110..2: I: I. :\in ol't.,~t."!-Id·e, acabarnos de considerar en este capftulo etro fcn61l"!~110 'que impide que Ios gene,,, <egreguen Iibrernente a TO! flora de produeir los genoti pos, esro es, cl I igamiento. 1'V-[ que cxplorcmo .... esta po;<;ihilidad. ,st':giin esta hi~te~js, deseribirfamos el cruzarnjento (0'[[.10 ~iguc;

T'

B f1 h r

~x-

e {P b P

{

Bp

hP

Si estu ~.~ eerrccto, {..que proporciones cspcramos en Ia F) PotkilMIDO~ ren~!u qu e, pam responder a esta pn.~guma< debertames conncer el umnero de unidades de mapa que separa ambos loci, Ello es rn.zon.abk" pem hay que empeaLI po. ~lg:rJn s.itio, asK que sl1pung-d:IHO:>. que. los loci t;5,l.m scparadn~ [lor 2.0 lHlLidades de mapR y yeamoJ;. qu,e Pl1f;f1. {RcclLcrdc ellllli'lIisLs tnedjllilt~pnleba y error; los cietMficus 10 u~an [I1ucho ell sus c,U(;"1I1u:,; .. } TenicnrlG eli C LJ:eJlta qut: riO hay cntrecm7'lmiellto~ en kt'i machos, la~ predkdones para la f'. pOdriM ~~L~libk('er;se: de la maner-a $iguieil~(:;

Esto nO~ du prccisumcnle ] a proporc ion 2: 1:1 q ue t'le~~FtiltJj mos. i;Comu ha oeurrido? (,FlIimjJ~ a[Ol1.m~.lido~ a.l esroger lIrf 1'I'e(;u;ench de rec()mhlf1acic'in del. 20 %'! La rt'Sptl~~lH <L'~ no, :\I,l i ~ npll[~ a q Lit Jrec ueuria hub leramos e~crJgkln, se hahrl a maD~ilf d(~ c.' mi~n1.{l re~ultllilu ~ cmnpfuE;belu).

Por t~I'tf1, CC"!'rTlO conclusion, (}!).dCItJU~ dccir qlli: lHprUpW'J,:~'· ferwdpka o!JtenkJil se debe a [a hereneia de gene~liMtld.lli ~II, e~ Laban C[I lase JIt" !"CpLl L~i.6n en los parentales, pero no lJ'1.1dem,: dm'BLTliltnar a 4 ue di.N1:and~l. NC enc uen uun. l':na t'arlng,nltla ml' preci sa requeri r 181m ceuzam i ento de prueba,

AllrJ4Hl~ ~~k haya sido U11 problema con 1111 pequerio nuce.: camino ~egl]idn para Te~olvcrli~) dernncstru la varicdad de ,cr' ceptos rl i sji nm~ qu B pueden emp learse fHlra dcxcartar cierrss ~ s.lbili clades y He~iI], a 1~:n_8 soluc i Oil que fun do ne,

ESPF.H:MA
()VI.IlDS B I~ 50<;:"1, /j f' SO%
R P IWX. B PUJ P 5·% if Pj/! J' $');-
Ii fJ H\''if h plBr S% /J rib p 5%
B P 40 s:;; if {!IN IJ :W% B 11{h ? 20%
h P 40(k h PIlJ {J 20Q- b ''fb l> 10% H /J B {i bl? hp

;'i + _1' + 20 + '2.n = 50 % :5 + 20 = 25 %

:~ + 20

= 25% =Oc.T

1. Ln nmSfJphd'f, el .11c:lnh'lmxlL1c.:! cuerponcgtu ':!' (!"1 fJ- ~·W!I.·, pu rnanun, (:1 fenotlp« silvestre. Elal e I rIC ~v.~, f]e un gen drsti nto, produce alus ceres as )' d. ~n· - tlh]::;HU' H:tl~~HS, e 1 lenot.i pn ~i I vesrre _ EI al elf) {.'I[ de ~1.Il tercer gcn pnxluce i~jOS L'inJI)I"jh~ y e] .r"!'.-' ()j('lR rojos, el ftn()tipu xilvestrc, Una hemhr.) heh~I,(Kj~ollc 8 para ,e~i'OI1 nes gene~ se u~a en un CI1H;a·rniente de pruebu y xc clasjfican LOU(} desceudierues de I:. F. i guient.e III anerar _'1 Nl 1 vestres: 6 rn:gms, cerosas l' cinabrins ~ M cerosa ~,c inabrios: 67 negros ~ _;:12 c:inahri 0<:; .,N ne.~ru~. ccrosas ~ 4cX ~~1·0~a~; y 41 rnc.g_f(lS:, dnabfios.

a. E·N plique eshV" n timeros.

h. Dibujc ION· ulelos en sus pU~i~'.LOI'I{,~ correctas sobre los ~'rolno~()!n,1;;· de, I;) hemlrra triple heterocigtiticu.

c. Si fueru posi blc, de acucnlo ~ on S,j.J t:x.pli.e<lc.i on, calcute \~. i.m erferencsa,

d. Si cruzamesdes hctcrucigutos t.ri:ptes como el anterior, rq!J.e pmpo[ci:c'in de b. descendenciu seria de cucrpo Ilq!:flI )' ~!las cerosas? (Recuer(l!e CIne en l'tif; machos de DrOf!(JjJTjUI1 Imn,ay cntrecruzatnientos.)

.. Solucion t

a. th.! cousc i u de carSck:r gellt;=l'itl q I)L~ heruos dado con anterioridad C~ el deser rnt:H)di(:u~ ,AquL l.IIl<Q buena S userencia e~ e~l~dhil'lo~ gen(}liro~ que puedun inferirxc de ]oll" .lCTwL~pO~. El expt~~i.mer.liw e~ Illi cruzamiento de pfll:eh!:l del~iguiel*~ tip~)~

b ! tb ' wx' h~x - en"! en x b/f) , ll-'.dwx ' cnicr:

Db~cr\'e q!Jc ha~' {cli~1in'll~~ lJ(ll.lt;~;l~ tk clases, Seg-li II su frecue IlC i D. entre In~ descendicntes. Ya podemos adiviuw: q ue ] a~ dUl> clases m ii s freeuenrea rerre~ellt;m cro rnosorn as parcutale-, l;l~ do~ (JIa.· W~ de t! 110~ 68 i ndiv i (I [lOS representan en.tn:~crU!y.ami en t.OI'l uniCON ell una regi6m. lus des cluses tit' unos 45 indivudllos represeruan e Illre9ruzanmie.mnl-i uni (:;(N> en 1 a otra [cgi.(m y las des t~ asesdc l!11I}~ ;) i.lKlh: lduos represenm ndobies entrecruzami entos. Pede 1ll0~ dcsc rifuir los tkSberllj,~cn(~ ~ como f.:~9sefj d erivadas de Inil galll~t.o~ del a. hernbra, j' ugruparlos [;(H:trm iooigm:;

fJ- , 11')"- , ell h' wx : envL , \\IX' (."fIo b . I';) .... + ciT ~ h+ - WJ: - cn~ b - ~'iX I - en b \A.'!' {"It

b" . w;r~ , en"

j~'.l. 379 h9 67

E~c,r.ihiet1{to a~( l;aR·,di~tintfl:t elases, coniinnames t]llt: JQS [%II~,' de clases ~nH, ell realidad, genotipos rcciproeos que n::~IIIlLaLlr c"Q.';J. uno u des enu ecruzain j1e n.\(I;<;.

/\ ])rim'era vistu, pareec Clue no pudierarnos apiicar la d.::f111 cion de recomhlnaclon. en hi ~lUt' ]O~ gl:!lOL~PO~ gJLlII~li~ot wtnp aran co n ~QS des gtn,rH L po~ de el'ltl'a.c\~~ q uepariicipusn la constitucidn de unindividuo, puesro que no conoc~mfl;, , parentales die b hernbra 'l.ripk ·.netcrm.:igultt"lI. Peru, retlll~l, nando nn FOCi"'I, los li'll.ico~ g~nmiro~ pareutales que l!!::n~~~ tide, ~C:fun los dlllOHlitiponihles, SOli b 'Ib i-Wi! Iw)[" • ,!.i!' Y hfb ' wxA"l-'x - C~j +h_'1.r-" pucs ccnsutuyen las clases 1lI,i-; r1 cuentes,

A horn porl emos ca lei L 1 a f I as frecneneias de mc.ombin.r~ PHm

69 + 67 + 4~ -t44

h-« w:'., 1::1 RF == _._. . • = 22,8 (,l(-

. 1 (100

;IX +4Ll + 6 +.5 t! -TIl, la H:J-' =

U}(lO

69+67+6+5

~~',~ - c·rl, la Ji1F =. . '" ] 4._7 %

.. lOUt)

b

pJ

~ I 0,;1 iTI_Il_-~!----14.7 IJU.L~

ih, l.(Is Cl\Of!lOSOma8 WIJ'enHI.le~ dt; la h~mhru t['j fl'c~e ht!krucig{~,i~WIH

{)~ ,11: ~L-"". +
-I -
t-
; (~1·
J.J I,;;X c; E.llllh[1l~ro de rccornbinarues d(Jhl~~ esperados es ~ll 03 x ~ 0,A7 x 1000 = t'L 14 L EI r!li!ll'~Ht) ohservado es (i i 5= 11. de m;JfIQr~ qu~, I~, ,j aterferencia ~.Jem(ls calcu ~'I1'I<I co mo 1 '" I - - '1IJ3, loll '= l - O,,7~6 =; rU74 = 27.4 %_

d., En estc upartado sc nos pi de qUE: llt~ I ic:em()~ IO~~(1lmc!n'~I',t'l~ Jdqui,idos~flbfe d ligamj~wo de 'eqns gl'nt:.~ para prededI ~1 ~~~lihlldo de un cruzarniento, D6e cuenta 4l1!:: no sc 'lmlll Jl' UJi (~rulil1mfC:l,l~[1. de ]I!nl£ha, :>J()~ preguntan pm- la proprtrC'iOIlI prer.rt!a de nKI{';Cll~ de !::U)erpo negro Y alas eerosas entre 1.1 dest;\m;leoda de dos hel(',l'(.lcigulu;. triples, P!JC!::il0 que (:1. macho UlU ~fka~l][r,Jjcmt1:u:ui~'Oll)~, la rnitad de ~1J;'j garnetos seran f) .. ·n' w_~ : I~ etru mitad! h I en wx " au nq ue estosul tl mos nCi eontrl buye n ;11 7eoottp'o de cuerpo negro ~ alas cerosas, Bfl la bembra, des nlJl1' ~Jmfjicos dlsrinros pu eue n cemribuir a qee la dc&ct' nden~ia W~ de cuerpo negro y alas (;CW;;;H,~: b cn"t w.:t' y hen w.:c En ,~"m~, k,s p(l!'ill1k,<~ cmuri,tmciolle~ de I (t~, parem~ les a estelenone<1 SPIl iII;<; s:i gil lentes:

~

~,:une'tl)~,~ b ('II" wX l b ("fl~ w.,~b Ui'.,.w~ b (:1'1+ wxlbcn w_~

I'J ~l0lILO~um~1 tl'!; In hembra tllic lleva b en + ~'"x cs un tipoparemal tu r!Xll;1T11bill;mlG_ H~\y ,UIl total de (:1;R2 + ::Irq)/! 000 = 7C'l_1 % rle rJ\1!l)IlSOllla:~ ]XIrelltah~!> no feCll1 mbi Ita I]le~. J.~ IT'I~tad de los: j3U<i Ie,>, es dt~il' .. eJ jB.Q5. %. llevan b en + wx. La lrecuencia de b en wx C~ 111'l1nOO() "" 055lJt_ POI" trml{). en loral, 38j)_'j: + '0_55:::;: 38_fl% de :u, ~a!IliMtm femeninos pueden d8F lu_gar8 descendencia de cuerr'" n~g_m y ~ las cerosa ~~ 13 III ilad de ellos seran fecund adu~ per WIIJIe:lgs b {'n + 1M .. del macho, lI~f (1 uc ] l1. lrccueneia cspcrada dB ~_~ililJi.l':"~~ de .:::uerpo negro y alas cemsas es Jo"c)i2 = 'I L).J %,

Ei.l r~'lu:nleJ1, I a~ fusioues gilH1etlc~ S ap.ropi,1d;lis puedendescri~o.t\~ ~ estn manera:

1 2
D •
iii ~'i~ I
,d
:il 4 5!
.-0 ~
f'W ~I (9f.i
3
r',
\,..J
1"'/1' rt fAr!
. ,'1 II

WI

desceridencia de I;nerrn negro y a~A~ cerosas

::1:, E, pedi g 1'( humane doe ~~1~ pjgln <I muesrrn U I] 1;;1S0 de he rencia d d ruro ~f ndrorne II fl~-n~llula ~ LnlA~ 'trol ulas tit: ItJru radas), .M uestru tambi ~L~ el gUl{)il ipo de eada i ndi v i dun para el gl"Upll ~8il,gHI"ll~O AHO, Amoo~ ~~xl implicndos t:OH am(ls()mic(l~-

a. Indique si el s(~)dr(~me ui)a-n'i~l,!laes dominante o recesiYo. 0 trezca 1::J.~ razoncs ~ Lie flr.oy~u'l s LL re spuesia.

1:1.. Ajuzgar poDH~~Lt pcJ]gTLGcxi-~kn prucbas tic ligamiento entre e 1 gel1 ufill-1'6tul a y e~ gen para el gmr'L'1' sangufnco ABO; (',Porque ~ 0 por q,tJreno-?

c. S i. existeu pruebas de ligCi1trienru, dii:lujc Io~ aJt-l.us so bre lo~ crumosornus h(JLWJit)gp::o; implicados del os ub ucl us, Si no hay pruebas de ligamilellto., dsbnje la configuracion de las dos pa(f.j,IIS d~ hemelogos.

d. So:giJt~ SiL I nodeln , r,~lli& d~";,;endie.rl1e~ corl't"~pliilll.erl ;! n;:(.;olliliin<lme ~'f

c. ~Cutll esla TIII!'1iUf estimucion Jl: It! RF'!

[. Si cl vanin Ill- I '~c caxa t:on una rnujcr nunnal de grupu .~ang:udln<'!o 0, /,~ud~ e" la 11Ir(1bahilidJ~.t.de qile xu primer hijo $ea de tipe JIj 'j tenga el :dndrome l!:fia-r&lLlla','

"So.iucI6n t

a, Lo m.i.~ proh,aNe eli que 131 sfndmme ui~a·r(itl!.lTa sea dominanle. S~ 1101; di ce que es Il na anomalfa rara, de manera que eo; PlUS in') pmbabrJe que, 18S Pe'JSO! L.11:; nu af ectadas que st': J r,[rodHG~H eLI JiI r<HIIl i 1.1 a, por L:illiamicUlO. ilcVL::1'I lIT] ~tlptle~to ulelo I!1.:lI;c~~iYO pura el slndrO[lW, U"memll;l;. N ,al alelo uausantc del. 1>fnJ:rnrm::_ Tmla~ Ja:.- p!:r:;omi~ afB::lad!a.~. ~(1Jl l\eten:K'igritic<l~ ,'\lJn, ya que Ind;!L:~ ellas (ind.~lymdj) rambler, probahle.tuem~, .:l 1<1 abUf']111 proceden de LUI rnatrimomo L)l)[I un individlln normal nln. Observe qtle el Nfndwmc ~LlxLJ\:0L: en 1J'e~ g...~].€;nlciones ~L'Cest\'.'a.~.ntl'a 1 ndfcae ion de herencia dorni t~antt"

h. Haypruebas de 1,11 existencia de 1.fg:.uTfllento_ Observe que: III maynrifl de ~os srmbotos negrofi.--;iqlJellos que llevan <:1 ale!o N---- tambien l~cval1 el aldQ fI~: rnliy probablemcnse. <lmL~05 8!~~- 105 cs Lin li i:!.~uJu~ a] m i~mo CnHl1~)~Om~1.

'10: 1~ 1~ 13 1~ 1:1) ]ti
G ~' .... • •
'J
ill I~!j fJi ill iii !e!1 f!i n .~ J\i lJ
x
fJ Ii c, Lo~ ah(ldo~ deben ser: C\lo incluimos €I ma~rbn()flto [1-6 x 1[-7. pCl.l()l!e [iO podernl~~ di sri n gu1 r entre sus desceudierues a IIJc~ 'L] L.1t: S,{JJ] ~~ norecemai ltHmeS,)

(Laabuela debe Jk .... ar ambos alelos recesrvos para produci. t1csc(~ntli,cl.~t()~ de genotipo iii y nIH L

d. OO-YBrve que e lca sa rn lento entre I os .atJH~J 0, equi .... ule ~ un cruzeruiemo 1I~ prueba, asf !.:lUll [os recombinan t<:..~ de Ia genera~i.6n 11 Se(m:

mieusras que e] reste no SOil reccmbinantes, sicndo bien il,' JOIn i (1 n. iln i.

e. Observe tllLe el masri rnon in d~ r os almeh)s _Ii I O~ dn;<; P i'jmeros masrirnonlos de 1,;1 gel1emd61~ lLl SOH iderulicus yeqtrivaI ente ~ :l cruza mienw.<, (J<! prueba, HfL)' un hmtl de 16 Jl)~C:;: I.~ cii'eukS; tn·~ de cllu~ (Il-S.IHly .111·3) S,(Hl recerubinantes. F-m:o no;> Ll~ una frecucncia de recombiH~ci6n de IU~ = V. = IB,~q"

PaOBI.El\{AS

f. {UI-I j') !'0,1 ifJ (nmrn41
n
x b'Tilpn O~
11 n Uamdo~

J'. J IV l~ HL% l '

11 I

40.6 % .,....__ una-rotu [8.

4(H~ % S1LIl.gre de grtlPtl .B

IKH% J N i l n (I

9.4% 9.4%

La~ dos clascs parcntales aparecen con i'grJ!)I pr1!'ctrlt8Je. 'i I,j mismc ocurre con Ill;'; dos recombiuanres. POf tanro, I~ proh~bili·· dad de que el pr lrner hijo !enga eJ sindrouae 'Llll~L-rotlLb. 'I ~l'!a~. g!'tl po B es dt~ 40,6 (;:(.,

.. ,~ ill iJ" Ii ill I. II i ill •• , •• ill ••• iii • "'. "' •• II! '! "" • !!' ! !' L! !, II! !' II! ! • ! !I! '! .. + • .., • ~ Ii ~ iii Ii o!i i .. i .. '" ill iii ., Ii ~ iii Ii ,. i .. i • L~ • r. II!''' !!' ! .. L! •• ! • !' 'I! !' "I! !J"" ! II! ! !I! "I! II.! ~ I!.

1. l, na pl;1IHJ de genotipo,

a

se ut i H7.lI ell I.m ~ii'lI7;~ mientc de prueba ClYU una pl~mlH

a h

J,

r euiende ell t' Lien L~. qWle 1 (}~ do~ loci ~~rr~l.u a. 111m. LJ. de distancia, t'.qLl~ t"i,lnp0rcifin d~ I'a descendencia sera A lJI~" b?

Los Joel .4 Y D estan Iig!ld os tan estrechamcnrc q LlC[lO S'L: ha ebserv ad~) nunc a reeembinaeien entre elloa, Si -se cruza A (lIA d pm (.~ D1t.1 f), y lm: indiv~d~lf)~ de 111i F, se p!'m~.n entre sL (,que feE'!(!I(ipns se ob:;.er-\'ar.~n eft .18 [11 '/ en que proporciones"

Los loci R l' S se CIH.: ueurran 01 J 5 m, u. dc distancia, S~ sc iUI tnpoliuiza un a pbma de r"rennti po

l? S
r \' 9, 4.

~'L] uc fenrrLirtl~ se nb."~I,"Var{i:n ,eL1 1 a descendencia y e.TI qlle prop0rc i onea?

Se real iza el cruzarni enro BE ' FI F x ell? ",f/ly la F I se usa en un cruzam i enm de pll!leba co 11 el paremal recesl Yo. Los genorlpes de los desceudlentes se imreron de sus Ienotipos, Dichos gcrmdipo,'!. dcscri [01-: ~egLlnla C(}I~hi bucion g'1llmellca dd pmgCllilo:)f" IretrBTOC Igcltic~~, lIpm'ece!1 en las ~ig!:l~ente_~ J?llrporcioll~~:

E'F R . .r

e ' i' e 'f

I ~

I

ii

:5. SeCI1J7.:l una eS~I!"pe. iJe j"{eur~IJPVn:l de gt::rwup(j .Ii ' I (lm. orra de ~,noliru h . i. La ruitud de los dcsecndientes sn~ j!j' 1 Y hi otra ruilad h -I, Expliquc .:orno e~ posible e:'ia,e ha!)h,1

6.

Se cruea un an hnal herebra de. .genoti po Ala ' Bib 001 U III acho doh~e reces i YO (ufo . bib), Los dcscendieraes ~mJu, yen 442 .4la . !Jib. 45~ ala .. bib" 46 A.la . hlb .'i 54 ala ·It',1 Dc una cxplicaei 6n a ie~rlf1<: resultadcs.

Si se eruzan iVA .. WB 'if aft). • bib, 'y la :1"1 se 8,omet~ a I cruzamiento de prueba, LilliE: JjJ()[ccf.IH~ie d.e ~os JC9L't:ntii.,.:.rics ~[;.L-an aitl ' bib. xi hlN dns g~ne~ cNtan (a) TW ~ug8d~,m; It I completarnente ligados (no se produce rl~nglifl entrecl\l~" rn i etllo) ~ (~l separados pur W unidades Jl~ mapa; (ol) sep rados pOl' 24 .unitladcs de- mapa?

En un organ] srno hapl oide, los loci C 'j' D se el:lCl!llerltr~m l una distancla de il 11UL En 11 ~ CJU7i1m,[emo C d x: (i fJ. ind ique bll pwPOrciO[l. CII .~;.1 (]{)&{:eutlciifua cit: ~LJa UJl3: ill las clases siguierues: (a) CD. (h) c d, 'k 1 C d, (d)t()(]~~ k.~ recomhiuantes,

7.

Lnarnosca de geFT!mlpo # ,~jb ~ se utiHzII en un CflJlJmr~r, ~0 de prueba con una mosca b rib ,~, 1]11 el ~4 % de laos [m:i~ ~j~ IilU st: prod uccn q LJia~tnt:l~ cIILTc los dos gCil(',8 ligallo,," 1r el I. 6 % de I:J,~ me i o~l s se produce un qui .. ~sma e'ntl,e' Ii" ~t:nes, i. Que propotcicn tie les deseendierues sera lJ ritd

lW. Se lIa n~flJ.iz~J:o 1I1:ii cruzamicntode trcs PUiItlO:; con msil,.1:J fC~ lJIJ:tlWO Y U.I1 }lnii.l isis de- recombmacion parei at Hpicr> _ u n cnazam lento de p"~leta de rres IJ!I mLu~, se mu e.~tI'an ~ e.:lI liul!t~c.i6EJ tp = hojas purpuras, +.0:::. hojru; HwJe~; II =ptdw~ lasrcsi SLtTHe~ II virus, -+ _. pll1intub~ ~CrJ;~ ibles; b = s(.."f.!I·il~ de borde marmn. i"= '>ern I II ~s rle CO[(lI' H~(I). A m~f ice el ® jW!~O d~ IloS lesu!la(lo~ y tespt::)Ildm 8 b~ apa.r[ado~ 21. l'I.Y ~

P +1+ . +/+' +1+. x pIp' V/.'· bib

Ga.ll1etns +-..".. 'r fJ' . ,.. • [,

.1;;1 +I{) • +1 ... +1fJ' x pIp' vJv . Mb tpruebaj

Fel1rotipos {Ia~ d eS·(~ti! d ientes

Gameto5 IF,

Recomb i na ntes entre INumems, p-ts p-'V It-:O

\'';::1' sen li~ ~" ., 1- + :l>2JO
~ ~l!T i1.':'~ OOi[ P \' -b 3222
't~r res lis T v '+ :I (~2A R R
l pur gel~ hnr I' + -h 1044 R R
pur res lis p v '+ 6t.HJ R I~
t, vcr sen bor .+ + 1) 678 R R
1 Ner re;:; bor + V b n R R
~ p~r sen lis p _. + 00 R R
Tnr.11 worm l500 nOOJ4J6 a Determ j ue q u<i gen~N esnin Ii g;.ulns_

jh, lDlhl1jeu n Irrar:m. q ue muestre 'I a~ d i ~t1n1C i.ll~ en unidawes de ~l1ilpa,

£, En caso de que sea '1pmpia<io, calculel a Inrerterenc ia,

Er problema PQ.W) apaso

!1.~.i •• ~I.!I!I"'I!II~". 1I!l1!. 1!~'I!!lI!!ilI!.!I!!i."II! Ii"!! II.!!. ill ilil ~t ",!!!il!li!!!!!!!!! ~

1 HagaJib!Jjo~ de l~~ plllHlaS d e Hhl.fz p<lre[11<lle~lP). I'! Y de I~ uLiltwa 0CJmo lndividuo tk pruoba, y LHi~ ~CC: Iloch[~$ pam 11,j')I131 exactarnente como rea] izan a este experimcnro. lmltq uc ~ dr.r.-;Ie 5~ recogen j a S se 1111 II (IS.

1. ~~rt)I' que se emple .. a repel idas \'~'C~'S cl ~hnhol.o + _ i ncluso [I1~¥-otntS dblin'lus'{ (~Por que 110 H~vJ a confusion?

~, i!(:(I'1K~ pu<::de U II fenon po ser pqrpn",. y III ~rl'6n (~'k1r ejem ~"J\ Rl tnistao licmpo'

4 J',Es slgilhkativo que los g~nes se descrtban en el prohlema :11 d lJl'uen p-v-b'!

5 ,',Que es un undivi:(['u{'J de. prueha Y ])(,If,' (juf se uri liza enesee ItpHk ~!I~]i3i~~'!

~_ i Que representa 1::1 co I um nil rnarcada C(lH10 «fenoti po!> deslf111~i~nlJ.:sr'r En ta elase 1. purtl.jernp~o, &t]!Je ~igrl.i.l](;a «versen ;i~(r~

~, ~QuC rcprcscrua la Urrea marcada ,«,Gameru~)' y en tiu~ S~ W~r~~da (Ie 1,1 co I umna 111 arcadu «Garnetos F I"'? ,f, P(ll- que es i'}Nrn:im~ ¢i1 ~I aJ18.Hsis de la recomhi rmc16n ~ ~ '0('1 Inp.1 racion ~,[~ l'!S~S !.]D~ tip'p~ de gamelo~?

s. ~ Qtl~ mciosi ~ es 13 Ijul:i H1PQctu ~'II CS te C'Ij,Llldjo'! SejlJi~eb \!II NU. uibuj(l.

,) u¥or q llt: 110 se muestra fI ICIS gametes del i ndi vi duCi de rrudj~?

]~ (.Pur I,ILH:: hay solo 8 chtsc'i It:n()~~pij.;a~'t iY~I.lita alglI!'L,~7 IQue clascs (y eu qLl~ proporciones) se esperarran SiC8~jOl !):n !~4U\~llm en Lin crornosom a disrinte?

I!, ;,;.., que corrcspondee Ioscuauo mmfl:lios de J as clases rdos mu~' n~mem~a.~, om interrneui I:l~. otras c1o~ imemrcdias y do,'; mil}' i'fqo eil~" J' I

167

l3, ~ Que r~lt:de dec IT sebre ..:1 unlcn de los genes, rras LIM si r n-

ric insl[leocion de I!j.~ clases fmmtlpi cas y ~ U~ frr()u~nt'i~~?

~ <I. (~Que 1:iPf) de distribucicn til: elascs l.~nut.fpi(;l:l~, se es penlifa si solo des de los g.;me~ estuvieran I~gad{),~?

15, l.Qu~ sl,gl1rflca la palabra «punto» en un ';<CI'l17.aI.Thiento de pmeltHl.de [t{,~ P~lIlto5»'rGlmpHC£i esa palnbra 1<1 exrssencin de ligallliem:o? 'I~C_.ti111o~m~rf:lun cruzamicntn de t'Lml:rO puntos?

'16, J,Cufi.1 es I:) detlnicion de 1'ii'{'o1'!Jbil'umt(! .'I G(]:m,(} sc aplic.a aquf?

J 7. i,Qut: ~i~nifil;Hdo tienen hIS columeas de «l{ec0!nbinante~ crure»?

:I ~_ i,Por {Ju;6hay ~()lu tr.::v columnas del tipo ilRecombill<lt'lkS entre»?

l~). i,Qtlt' ~jg.fll f"ucan las Ry pm' qu.f se han situedo dendc csLan"

10_ i,Que si gnificen ](]~ tetalcs de hIS colurunas )' CD mo u:: 1I,q II e ~ este ;:lI1ij1i,~i~?

21. (,eu.il e~ el crlrerio para djagTlo~tucal[" la C;x,i~Lcncia tk ligamiitulo't

22, i~Qli~ ~s una unidad de n~ap~.? /,F.;;; 10 11lismo {]ue un ccntimorgan';"

:LI" E,f] uu L'flllifll nieme de Wlleb.1 de tres puntos eomn {\:t.:! r a"or tjJ.u0nu 80n considerados los indi-iduos d~ ~il F] y ~1'~ de prueba Wino I{)~ pa.renta]e~ en el dkuin JI;) l!inx:ornbu!L~Kiofl:! (1~"'1 cierte senddo, SOli parentales, l

24, {,ellal e~ I~ f6rmui;a de 'I~ 1filerferenc:iat (,C6rrw sc calculan I i1~ lrccuencias «esperadas» de Ia forml! IJli del coef ciente de coinci denci a'!

2S, l,P.or que dice la rregIJlIbL ,r del problema "ell CilSO <Ie que sea .~pr{)pi udo» ?

26. (,CUi3n~1:) lrnb1~o CtJli~SL11 obtcner lin rlU mero tan grande de descendlentes en l11afz:' (,CllantO\ dt~~'(:t:~Jic:rnlCl>, aprcxtmauarl1enlf.,rel)1·esent:l nn a nna7.or~a~

m I. L!1 indrvlduo hereroclgorlco riJrl,I cuatro ·6oeneN. Ahl - Blh . , Ctc ' DId. se eruza con ~~l. individlJorl/o ' bin" d.' ·.rI1J j' se obticncn Hmo d~:;'(·CI:H.heTitcs, que ~e disrrsbuyen como slgue:

a' B - ('. D 42

A ' h - f . (} 41)

A 'E C·cJ l4{)

a·{J"c·D a-B-r'D A'b'C'(1 A' B· c' J a-n-C-[)

l45 6 9 JO;;; 310

a, .:,Que genes esr.inl.ig.tdos?

b" sl se uriltzaron dJo~ h]l~a~ pUF~:; p!llnt obteuer el indlviduo hererocigorlcc, [,cu:iles eran IQs. geTIoti'Il'U.~ ck diehas I fnea.~ puras?

Capitulo 5 P'rincipios de C=:u:-lugmfi~' cromoscmiea en I.os eucarfotas

e, Dibujc UJI mapa de Ii ~ami enio (};l' los gene~ i mTl,1 i'::~I~ Ius, inc it.l}'el1do su orden y di ~lanci~L~ COl un ldades de mapa,

d. Si Iuera posihle, cnicule I~I iuterterencia,

1.2. ]f;,11 el Jl~I)j II iUJ ud di ab 1 o, Edl!Ui! t tum tsfm erium. ex ixten cI,,~ SeA.OI, scparadc», (es una rblllll dieica) que no estau U eternri n udos por c roruoson m:; SA.: x uales h etc vornortos, sino porgene» especffico-. Lm ulelos de leis dos lociirnpHc-arlos til lu tiClErrn i llaC~(111 sex ual son M I}erti I i dad rnasculina). m (e,;;tel'ilidad tuascutinu), F (e~terillklad r~:men i n:ll 'Y _r [fel'rilid;,lul lcrncni naj. Lc~~ i nd i v i (I uos de pub laciones nat ural es de esta pi anta puedeu ser rI1 asc Lli i no~ lap'tL)ximatlamcme ~I ~fI q) 0 lemenmos (apnn::nmaufImente el 50 '1'(.), i\delllil~, S~ cncucntran il1rli\'i<lllu~ herIII !'lfm(\I tas, peru ron una frccuencia rnuy baju, EI hcrrnalrudita ticnc "I'ganm ~xll,~k~ rnasrul'iru» J' ferrterunos eJI Itt rnixma planra.

:I. ;,nJ{!J debe ~r cl gcnotipo complem de uuu plantu In::J;',rul [U<:I''? (lmJoq]uc la re,lI)Ci(in de rd~<L[nit:IlJlo entre In~ gene~, )

b. (,ellal debe ser el gl:"[lulipD cornpleso de una ~~hUlHI. Ierneruna? (Im1iLIUt' larclacirm de l'igJmiemo en lire 1.01> gen~~. ~

c. i.Como sc rnantlene eu la [111bia<..:io[l la mrsma propnr'... i6n, apmKimaclmneme. tic 10): dos ~C)«('lI,'!

d. ~', CllJl e.~ el origcTh de I o~ e~L~<lSO~ lterrnalrodi tax?

*J3.

En I a e~pt"C1e hu mana, c 1 alclo antosorn i co N pro V(JI:U ~IIK)ml ai idudes en uri as '! rol1ll a, I';~ ~p.o\S (;lJiuocjdo~ cnmo sl'IJrom<:! uiia··'{'(JIUilJ" CunsideH~ JusrnaLTim,ollio~ en los que LIllO de l,()~ individuospadece el smdrorne ufiu-nrtula y es de gnipo ~!mguine{) A, y el o~rn i nd i \'ll.Ill U C~ de grupo ~,mt!.uIllc{) 0 '!i no padece el srndrornc. Esto~ matrlmnnios rienen.alguuos IlUOS qll~ SUIl de grupo r;:.mgllflleu A y pudecen el ~Iml rome II il~l-n')lLlIa. SI! fion g.l que h.ijo~ de esc gr1J.fiC' fenmipi'l'u. proeedentes de fanti I.i~s DIU relacionadas, rnadurun, se casan enrre Sd )' tiencn bUn~. Entre It ... dcsceudientes que forman t'>!.a scgunda generacion se observan cuarro Ienetipo« di ~lintC'!~. en I A S proporciones siguienle~:

:SlnJromc uiia-mt!.lla, gn.ipo ~angtllnc() ,\ 6{1 (Ji.

Nill ~rndr[lme Ilfi"j~r6tLlJ;!, g:rupo ~ang~{ne-o U J6 %

~jl1 ~iI1drorm: lllb;,) rritu'la. gl'l,upo SaHglJ.LLl~ll A C) %

:-;fLlurDI.m: lIf'ia-nirl,lla, gwpo :>Wlg_UlnDtl 0 ~'Gi{.'

i\ t) ~.Ilce ~S1..(1~ d atu,~ ttm lmln detail e, ()fr~cie rlJ\) Ulna ex pHCacjf'l!l ,J~ Ja~ ('rccllcncia~ relariva~ de h,~ cuatro fenn II po:; ,

IS.

L'liIi zall d.o Jf\~ limos ubR'TIiJ.(~~ pOl' F! at.;soll Y .PUIl ndL 1'f'Il:ldITI .,:-1). ca'lcuk' Ja di~t::Lnci:k de ft"lfllx!. (":1'1 m.lol.) que ..e.pararl [l'S g~ne); del cfllor y 1ft i"uflll!l.

Di~rnllem(l~ die lHl~ hm::a de fJm.wlJhi(a homocigo'lica pam los aklu~ tww~()mlco~ I'BCf:SIV'OS t!. b )! c. li.giJ.do~ ~Il eSt: mdl"[l. Cmzamo:,: hemb·fa$ de t'~lta linea COil rilach!..l~ homncngotk~)~ pnr~ Io~ ald{)(~ ~ilve~tre;<; ~~tn't:spumh~nl.C'~.

luego, CnJ7.3mOS los machos hererocigcticos F cnn .w, hermanas h~Wr(K':-If,'/~tica~ y obrenemos 1()~ siguicatcs F~mli pos (:11 I a F 1 (, I 0<; fenoti po;, reccsi VO~ sc indican .00 11 lerrll ylns Ieuotipos silvestres con el ~igno +): I ~M + + +, J65~ f1' (, fJ.7 (.l: b +, ~~. T + ~:. 47 a + +. -44 UI (', :) a + r: y 4 +n 'r

~'H., ;;Cu41 es la fr~(;tltm:iil tic re.:.:ombfnr.td6n euire 1lI~' Ii j,Y entre h 'I c?

h. :.CtI~iJ C~ cl cocticiente de eoinculencia?

.lfi. H. A. Lmerson cruzo dos llneas P!.III"lb distinue, de Illart, \ obtuvo una F, fellofl'PK4Iilt'rn1t: silvestre y Ileterocig(i[j(',1 r'H:rilln~ nes alelos recesivos ~iguiente~: an delennjll;LI[lI~, 1"11: lu i!lekJ'JIlim! braquuico, ylderemlill<l ddgudu. Hizu tJl cruzamiento de prueha entre !a 1'1 Y una variedad IKlTI'II"l;I' gOlica rcccsiva para lo!.- tres gcn~~. obtenlendo estes fenDli· pm; riescendientes: 355 ant-eras; .139 braqufucos '.I IklV do1;,: SS rumplcturnente silvextres: 55 antcras. braqujtices. deJgadu~; 2IlJelg:lJos' '17 .. uterus )' bruqufticos: 2. hraqilir cos: 1 tlntel'il'i y ndgaulJ~.

h, Dibujc c] mapa de ligamiellilu de hh trcs gCJlCI' (il1cl~ ~'I [a~ di stanc i tl~ de 1'1'8 pa),

c. Colculc cl valor de la tnrerterencia,

17.

El cromnsoma JUt' I Ibl ltlZ contiene rres loci, con Ius 1Il.cJ..~ I y b-, v 'i t,t, 'I ~g y 19 I. L(l,~ fenotipos rcccsivos I)[)ITCS(lI:!l di,t:lltc~,;;c abrevlan como b (color tie la planta mfells.itic;~": v i plflt'\ta verdosa) 'i 19 (planta sin I(glll:l): los 1e'nuljp~ s, vestres se i ndican con el ~!gllo T, UI ~ cru zamiento dt' prut'h. entre planLa,~ de ia F, heserocigoucus para los tres ge~es Jl]anlli.~ homocIg6ricasILipie rccesivas produ]o TO-Ii l'ljgUi~llItl 11:n ntip()~ deStotUlJit~l1l.c~: 305 +- v lr,; 27.:; [I + +; 121!: b T",IJ, 11'2. + \. +i 74 ++ir:; Mb .... +: 22 ++ +. )'18 b v {g. incl:iq~. el unlen de los, gene ... en el cnnuosoma, I'lL" di!ita'lld4~ i~ mapa encre ellos y el cocficientc de coincldencia,

IS.

los grudis snn organ IS Il1GS h J ploitles uti les (aunque il1lagl narios), que se udlizan c;omoiIlsl:rumeluOs.£ellerico~ t gmdi ~ilve:stre eS gordo. de cola laf£~ y ~uu flagdw S. ()('fI'lOl'C[~ ][nca~ mutante~ deIJgad<tS. IJ ~in cola, n que tare n'rn de I1ttgel(lc~. T.os grudis t.t: ap.::truan cmre s( (1II1Il{l:lI~"f Ui.n ~·e q:;flIU.O 'IJ~ lJUG rw ~,abem()~ como k~ lw.cenl }' ()IJiJ. cel~ r't:cUJ'nbinanlC~, Un g nul I ~'II vestre ~l:: tl'I.Z1l l'tlli U~I del~Htlu )' !.ju~ l~al'eCe t;wto de CUhll:OmD de! F1agelm. L. 1 ODO C'llchorr-o:;; gmd i lj iPl~- <lpMC{:eu <;e di~tll'ih'lye!l se,gi!J1 ml!e~tn en 18 rigllm Ut' la p~;gil~a si.guiellte~ As:ig:lLL.~ N1JTlW '1 ('I~ <l k'ls ~enuLi pos y d ibu.ff; el mapa lk: lus Ilr.:~ gcn,:s_11I'1- bJenuL l ~ p.or ~-nrie~(" de ll11rl~'11 S Gu Ilmam. ~

19.

Sl.ij)lJu~a ~J ue SG idel1t[fic.TIl 'rI'es pares J~ ah::Jm; en .D.rr);~j~ /lila: x~ y x, y' e y, ~ ~ y ~. CumQ indicaiIJ los ~{mboio~,~' llielo no slJv{'mre cs ret.:esivo fre:nte aI ah.:lu :Sllse~IlC, '; cl'nw.tni~nw .entre i-lel1lhn:!oS helero[;igoiic/l.~ p~ra w;:;;-,

370

U1.:S loci 'i machos silvesrres rmdnj 01 R ~ i gbl he nre descendenci a, c lasi fi cada ~~g Lin 1()~ genu tipos de lu~ g .. nietos ~pllf1ado& pOI' ~1i hernbras: 10 I (") hemhras x'" y+ . z": 430 IUflcbos .c : r": z: 4+ 1 machos I ' ". 1'" .': J9111achmuc' y .::;; 3J,mlleh{)~ x+ . r" ':1:; 30 machos.~ ~ 'r- , z ': 27 mnchos ~ , y . ~ ; I macho _~. - . y . c. l' 0 machos x . v • z -.

a" j~En CjJllt: eromosuma de f)wsophiia se encuenrran es t~)~ genea?

b\ Dibujc los c r omosomas de Ia hembrs r<ll'eb'ltal heterocigiitic.a irnplicados en losresultados. mostrutsdo en ellos 10: dlsposiciou de los a le I O~_

e, Calculc las distancias Lit" mapa entre los genie:,> y f'1 coeficiente de coincidencia.

!t it partir de Ins cinco gTUpO~ Lit, r~sUlladlJ~ de 111 tabla 'ld.illfllli, avengue pnr mera 'in~pn:t'iull (CiSLU cs, ~:i.il calrular tre'~~lldilS df' reco III h i nacionl el orden de Jo~ gt:ne~, l.os I~I'iOUPf'l~ recesivos se i nriican con I etras mimisculas, y iON dDmLnanle~ cen ~:1 siguo -i-,

F~flot i P cs obse rva dos
en cru zam ierrtos Conjunto de resultados
de tres ~untos 1 2 :3 4 5
++7- 317 JO 40 :'lOS
++c 5~ 4 6 232 0
-I- t) , 10 ]1 339 M 2~
+ b ( 2: 77 IJ7 :20.1 1m
~It + t- u 77 142- 14,j, 124
(/ + r 21 31 2~J] 77 :1(1
oIh + 72 4 3 235
~ I) t ':W:I .1.4- '-hJ 2b.'i ~l En la t:tbla acljl.l.T1ta apmcDcll d::fltl,'" fcnoupicos J.~ los cruzaniO;!lIms de prneba de W~S pnntos, en In" qlle e,<;.dn irnplica dog (I) a, b y c. 'j 12 ~ b, (. 'j (1. Averigue en qt1e orden estiin Iq~ cuarrn _g€n~s. a, b, c y J, 'j las LIe~ ili~tancifLS de rna PH ~ILtIe ellos, Los fenotipos recesi VOR ~e ind icart ocm ] [!:LIa~ IU[DW~Cljl as. 'i los d~_ml.i,mHlles -COIl el $lgUO +.

P.'obl em 3. .. 16~
'2
1""+ 669 bed 8
ab -t- IW b ~ +- 441
(1 of. + J h +J 90
+ + ~: '[21 ca ;n()
~ he 1 + -i- -i- I,..;J
CI + c 221m -I- -J 13.~
a b (. 65J. f' -i- 65
"h + 221.~ be + 141 22. LII Dro.Yoplti!tl. el alelo up' determinu alas lurg<Ll; y el '1ldo dp ulas s cortus . En Uj[[ IU(;LI~ distiruo, el ,l'Ielr; Po deterrnma cuerpo gris 'I el f-' cuerpo de celor e-bal1o. Ambos loci son aurosrnnicos. Empczando con Unt~a~ puras, se reaUl,1 el ~igu,i'~n'le cruzamien m:

p lJ.qt~1:i, ebano '1" x corras, gris )

1

largas, gri~ ::.: x cortes, cbano J (llllt:~ puruj

!

larg .• ls, chano 54

largol:'>. g:ri.~ 47

52 47

2m)

LJ tiIi~e 1£1. prueba l !}8 ra dlererm I nar xicstos loci l:~Hin li is,<ldos, Pam. enn. (a~ cstablczca una nipotesis, (b) calcute ~I valor de 1-~' (c) e'l va I or de fl, ~. d ) nHdlq:ux:: qut ~iglli I-[c~ esc valor de p, (e) esrahle7ca sus conclusiones, 'i (n inner" la con-ritucicn ewmu~6Gi:1il",L de los parentales, la Ft, l(}~ rna chos de prueba Y ~a descendenciu,

errrtas, grl ~ curias, ebane

23. Para m l-Rli:lJr ell i gamh::!HO entre vari ()'> g!;:lle~. $(' han eruzado dos estirpes de levaduru bap1uiJt~~, purl.aJvfa carla 'Ina fie dos alelos rnulautes, Ll eslirpe A lleva los alclos a,~p y gal, que Imwoc:1Il requeri micnm de. a~p~rttLw ~ :irL1;;4paddad para ~l'ljlj£.lU i.<1 gal<il·I('Isa: la esW'l'I~ f'l II~V.1 los aldo~ rad )' W'O, qlLe p!'()vocan scnsibilidad a lu m.Jiaci~~n y requei imi~fll\) de ami noacidos ill'Om~~titco~. Se fm comprohado cl genoti po ut' los descen dientes haploi des, COl] el resul lad I) sl ~ IIi enre r I (I~ :.I]do~ silvestres ~e- indican :;OJI .;:1 ~ign(J +):

Genotipo rrecl,J~nda

(HI)' gai : -i- . -I- (1.136

av,' -t-> +, fad n,l}!)

IJ \-1'; , 8£11 . ,~ . mel 0.064

a.ip . + +, -t- U:Ii(i4

-+ ' gaj , (H'V . -I- n. U()

+ ' -I- ' (~m ' rad 0, l36

I '~'lt· uro ' nl(l O.HM

'/-'+'mn'+

a I-J} , g()j, ara : t-

asp : +. aro ' rad £(!>p • KtJl ' (lro - rur] a.~p . -i- - am . +

+' gal· + ' +

.j .. + . -i-:': rod

+ ' xai . -i- ' I'ud

l' '·1 ' i 'i

U,U64 O.()34 U,U.34 [1.(116

uuie (J,O.:l4 OJl34 ~).Oj6 fl.Olfi i.ono

.170

24.

II. Ca leu le ~a8 seis frecue ncias die recombi f!;:l.c! Or),

b, Dibuje LlU mapa de ligamienro que muestre las fi1()siciencs J~ loscuatrcloci gt:JLt~tkos,incJlJycndo las distanclas entre ellos, en unidades de 1U1Lpa_

Un ge l)etlsto1 coloca Junr(lS Un::l hembra de rmrin de una lfuca PlIJ:'fL. sitvesrre parael CO I or d~ 10(> oios y e I color del CU~I'P0, y un raton macho de ejos de color ~l.bmi.ru4l~l~ (determ j 11<)(1 0 pm eJ allek~ iI) Y peloaJe grid determi nado per el ~ldu M}, L(J~ rVl-~0!.U apareau )' produces una j 11 silvesrre. lE..~to~ ratencs tie 1~ PI ",pm:cfin entre ~i y producen LIlla F~ cf)l1~dtnida de la ~fguiente forma:

Ilenlt'll'8,S lI.·hl4:hu~

todas .',11 vestres silvestres albarieoque, gris gii'is

albaricoq ue

5~.

45%; 45%

~.

h. lndique In~ t.""I~ntipo.~ de ]O~ parcrualcs ':j de lo~ do~ sexes de- 18 PI j' 101 F~,

25. La rn Hurl:' de una hUHLli H tic d ieL i.tij uS es de grupo sangujnco Rh", Turnbicn m~n.ilit:~~,a Lin;:! ram aleccirin (elipsociro~i ". feuoti poE) que haee q ue In~ hem i:[rrC~ ~can de lonna oval. eu vez iJe. re(lomle1l(k~s, sJl:)h~n nn p[mlvce ningtln etccto c1:uIIi,t'D ~lJ,,'~'r,,;o .. El p~Jro C8 Rh- (car~('b tiel f~[ll:Lge- 1'10 Rh,) Y rose.:: hematf C~ nnrmales (r~_'noL:i:po c L LlY~ hij'o!> ~[m: 1 Rh" e. 4 Rh" F Y .1 Rh e. Tenemos ~Il,formacion ~ubfe JUt> padres de Ia madre, que ~H UI1" Ii Y Jlh- e, Uno 1_k ] U\~ J icz hijus t ue [CIlOtipO ru~ - E) ~c ,C1L~a cor~ U~HL rnujer RI1' e y nenen un hljo Rh' E,.

a, Dibnje d penngrf de I~ familia complete,

h. l. h$~,ti este ~igri de 1lclJerdo 00'11 la h~p6te~~ s de que Rh - es dominsete y Nh- receslvo?

e. rOiiU es La fon:nn de herencia de laellptoelecsls?

"d, (_Podrf .. n estarlos genes para ~os fell(Jtl PO$ E yRh en el mi ~:nlt) cromosomu? Si C~ ~Nf~ haga una estimacion de ~<II dlstancta entre dichol~ gene~. comentando ~u resultadn,

hi padre de Mr, 51'(K;k, pruner oflcial de la nave estelar Enterprise, proredia del pbJ1,e~:8 VUk,.;]!.!lO; su madre. era de la Tierra, U n habitantc tic V ulcano tiene ~ .. s orejas en P1UHti (determi nado pm el al elo P). carecc d.c g!{md~l.ia~ UpF.:Irn> nat {delennhliKlo POL" A) Y riene .. J (')('I ra 7~51l ,1 ~i'I de reGh~j (determinado par En. T odos t:;S.N)~ alelos son uoruinentes ~c.)bl;e~u~ alelos nermales t:'rJ la Tierra, Los genc.s corresp~1Hdieme~ son antosom lcos y .estan I i.g::±do~ segL1H C 1 1:f1 a p.a

p

+--- 1_) nL~L~""'____2:0 m.u,----+

f';j, Mr, Srock s.e c,a,,,al~J..cO]TJ I1na~ni(_'j(:]b }' no nllPie:llIalg(in lipu cle in~(Jmp<ltibi.lidad (gem~fka). /.qL~e, plnpOf("iOll de ;~u~ hijON:

11:. tendnU as urcjas .'1 el eura~u'n como 1m, ¥Iulc'f[o,o!>,~' t gla Ilclu I a ~1.1 pr.11'Fe!l81 como lo~ te'rriCCl~:lS?

d, tell'ldrfi I as meJa~ com (10 I osvulcanes, y el cm-::.I!l:L)D ~ .• glAndula suprarrenalccnio los terrtcelas?

rEL ~I.lbkm.~ 26 .~<J sido tumnuu ~t! D. Hamson, l"yoJ:;oi.~m~ m (irp.l'" A,1d'~o~- Wc~18y .. 1970.)

27. Ell cierta planta haploids, hay tteN loci, A, B Y C, I:I(IC ~ Ii g:;'Nt05 de ~ a sign lente mane ra:

+--- 2:0 rn, u, ---'H--- 30 m .I!I.-------'>'

Tiene una planta t ll am~mns 111. pareetal) de g~'nuiipCi ,{ I d!1/~ C.

a, Supo uierdo ausenc ila. de Ii nierfere ncia, S i la Illano·. 11 L1lo;f6l;U nd ~L, i.tl!Je properc uon d l" la desceadeucia s~ri" g(::'n oti po a b cia 'J c']

b. Ern auseneia de i.nl;;rLt:r~nda. si Ia phmlaparnl1(a: ~ cruzu COil O~r:I. de ge!lGrripo a h cia h c, ,i,que daliC~~ll1lUE picas se encom.t:llrj n entre los descend i eutes? l,cuantfl' r~ di v idlHJ,~ habra dt" L:Hd~L clase en lin liolill de ~ 000 (lemr u:i.t:mt.e~':l

*c. RepHa.eI apartado b, .~upon~C'ndo e~HI vez ul1a.~nLWl 11:11 d,l de I 20 % e n[re" I!ls dos regi ones.

2ft 1'1".1:': Y[;ll'm~ CIU7.lIm i entos del [ipo .MA - BtR 'l<i ala - blb_ UI ind i videos de Ia F I A h~' nib, se 1).,,11 i zaron ell eIT'll7.:Imi~11ln de prueba eon individues aio ' bib, Los resultadus ~'jXi~ E!~lh lllbla:

Cruza m i e nto de prueba con

el cruzemrerrtc

DES(EN[)ENC~A DEL CRUZAM IENTO DE PRUEBA

A/~ Bib a/ill b/b A/a bib iiia 'BM
_no 3J5 2fn 2a~
_le) ]~ n ~]
c,
360 380 ,BO 2~O
74 72: 50 .l4 2

Use d 1lIlliii.li~is de f parn decidir si hay pruebas de IP' mienro en cada caso,

2'. CiCTll:l~ v~[icd aUesd~' hno rnuestran resi stencias disti n!ill esri rpes especffi cas del h[mgo 1 ~amadtJ 1'O}6I dd Jim'!, P e_i'erllipfo, ~~l v <I.r~e(j<l:d de I; ~o 17 OB e~ re~ i,~tente a la e:;tirv, 2A delhongo, Pc."£('1 es s u~certi ble <II JW'lqtJ e rl'i;l b e~ti!'W ~ m~cnl!lll~ que hi vmi.t::,uaJ Bombay de lit:lo ~ .re~i~tenr6';l~ e~ti ~pe ~2 y ~m.ceptib~e al ataqne de 1 a e.>ti.rpc :!A. Cunl,) 8~ (.:wzm:tij[ lu(> YlfIl'iwa,jes nOB 'j Bot)d,ay, el ~lIhnM r~c Le~i~lt:~1lc a I:lm.ba~ t: ~dm:pes llt:l. hunglj, .A1 ,hHOillOl i'll~" I,~ FI, ~e rff)diJjo !<ina E" qu~ ~nclui1:l b." pmJlut1;iu:n~s li_~ pic.~~~ q 1I~ se~n.d~call en la tm bla.

c

RAz....A,.1J DEl HONGO

[{ALA Z4 I R I

DEL.HONGO S

_R L~4 51{

63 1.5

Co ~",

a, J5.5IT1Wezc;a una hip6ll:Siis ql)e de ell emf! de las bases geMlicars de JI:I rcsi stencia dc.lliilllD 11 cstas r::;U:aL!lp'MLi,DLJ.l arc.~ del ~i::)Q~fi" Desarrolle t~ hipotesis de fIllll'le I'~ cl a ra y defin A cualquiet ~[llUol{J ~ent:li{'1J que l:.rnpk:c"t:waiqUJc los genotipos LJUC proponcpara las plantas 77GB, Bombay. F! ':! r;

II.Compl1l00;:: su hipQ~e5iR, I,ltilizando la ~~I'm:ba de Y. •• Indique 10& valares esperados, el 'v alar tie i (COil do~ J!etiuL<l. .I:;~j 'J' d valor currcspondienrcdc la probabilud~l(l, p. Expliq ue ~~:;wtalllellte .!f! que se reflere e~ta prnl.f;jl,i lidad, De acuerdo c,.;,~ el valor olnenido para i, ,(_lloePL3. 0 reclla,z,a su bjpOlf"s~s't

,!U rtublNtlJ 2'HIS NEOQ,ad2lp[WQ ~c M. Strickberger, (ie.nt?!ics. {>,..~2oCrn illan, ]k'i.1

30, I1ll JJ;}.~ dos pt:d:igrre~ (1 ue ~l: mu t:&~H. los sunboles con una I:ramJ, vertical indican chfici encia enlaenzima su Ifa~a,~a de. ~stemitje;s, y ,1que 1108 con Ll11<1 barra h('llri701)t:.~ lind lean ~JeI·id~ncifl. eu In enzj~Ili!I uaascarbanrilasa dot: la ornitina,

1l. GEtl}' ~.glin indici 0 en CSWN poo:irric::t de que Ius gcnt:~ ~'J~ pmvoean l::!L1 defleiencias esten hglldDIi!

II, R'Ilpol'dendo que los ge!le~ el't4nligOld()~. ,~hay a I gUll~! pmeh.;1en los pedigfl~ de que se naya pl'oJuc:klo alg'Ci~l cr,LT!1lcru~;SliTuienLc)?

~ Iudique ~'I ge;no<t.lrJoo de esros i I1dividu()~, con tanro detaII~ eomo sea pos~:bte.

Primer pedigrr S~,undQ ped~grr
EE-() T
.<l
IJ " onS~
2 :il
rII III c5B8
1 ~ J .4 5 6 :2 3
[lJ [JH
2 1'L, EI"'iglliem~ pedigrl' muestra una famula ell 141 que aparece I] ~,fe.l:Lo~ipo:> anormales Ilili£m: C~d0t{J:li~ azul (dd)il:utlad en 1M hi!L(;so~), rcprexentadamcdiantc ,'lhlll,m~o:,-: de horde gruB. ~ l' bemofiil m, represenrada P01' snnbolos con un cuadrado D::gru,etl. el ct:J;I!to, Lcsindi viduos rcpre~l'''Jladu~ pur ~iITU.bOlm ~lif:npktalill,cnte 'negm;r padecen ambas errtermedades.

~, I;QLLe forma de herencia rnlle'>[[OIC;'ld,l urtn de e">(1;5. fe-

~ulipo~ en c:~I.t: pedigri?

PI'QbJemas

,. Iadiq U1C los ,gcrmLipo~ del m~yor numcro posibl t" J,~ rn iem bros de ~ a fa,m in 11_

c. ~ Hay pruehas de .I 11. exjstencia de ligllmiemn ~

11. ~[lued8 corrsiderarse 'a ll~giin lTIim:nbm de 111 tamiliu como !'~(:,oI1l1b]Jllflfi~ (e%f'I es, forrnado 111 r<lJ'ri r de a ~ rnenov un g.l'lmeHl rccurnoimmLc rt

)1' En el siguientc pedigd"Im. nl},Hs verticales lndican ceguem para luI>. O!JJ:u[G~ d,e l 'li:po ~~~nucid:u l"I),[UO p.rm'~IJ.op.'a, j' las myas horilzc:mlales ceguera p~lTa l'os Cl(~I:nre~ del tipo deutc raaopia, Se trata dedes afeceioues dlfereutes que prc)Vocal~ di~hn1()~ tk:f0!5L'U~ en 111 percepcion de los colores, Cad(l una se debea un glln diferentc.

"

)) I C5 ~ 111u
III ~_... HI
2 ;3 4 51'\..., :I.. ()i.~.uewa el pedigrl alguna prueba de que iu~ genes .e~tfi r'I II i g,H~OS?

b. S i hubiera llga rn i ento. J)lOl.y :11 guna: prueba en ~I pedi gri de tlllt: se h.ll)'a pw,juc~c:lo algLin enuecruzamieruo? E:\r~i que M!I~ [l.n~ r.e~pLle~.ta.~ conl 11. lIy uda de dj agrl:lmllN,

t'. i,Pualie L::1kIlI ar un valor para la fr{'>!..: ucncia tie recornhimiJcioll entre estes Rem~8''! ~})e m_na de seweg:aci on iudepeudiente o de CIlLTlX:nI~l~Hr.il·illO'?

1n

"'33. Lu~ ~(!nt':~. bumanos d:t;' la eeguera para l05 colores y de Iii heme I-~ I i a e~ LIm am bos en el (JiW mosom H. X. [HO~lrHlJd(j ~Iure e-II()~ unafrecuencia de recomblnaekm aproximaslu del 10 %, ~J ~ i g~.n) lenro de un geL1 pa~n 16g leo a OW) rel arivarncntc luocuo puede utilizarse en el diaguosrico genN~CO_ A(1U 1. apare(~ parte de (In ped i.gn I n~~ extenso, Los ~In.loolo.o;. en (liVe.UTO indican qu.~ las pen{)llll~ son hi> moltlicas. y [<IS cruces que sutren de ceguern P<ll'a I(I~ colercs, l.QllC intonnacion puede olrecerse a l~IS mujercs m -4 y lJVi respecto a la pm bahil i dad de quc ~IJS hijON VMum.:S se .. n he m('lf~l, lcos? rr"",III",,,~ ~.~ a-iJ~ rl~d" .k J. F. ['r(,~v. (:''::-.I.~di(·~ .~·me~~ . .q~1 ir:.riHJductiml' to Gf:~iedLW. Eh!Jr"=,Ji;!s:i_ ]I)~}, I

T
II ~J r
~II .- • 2) ~
2 3 4 6' 34. Un gcm:Li~'la. (,I.uc pretendecanogrufiur h)5 gt:nes A, [J, C, D Y f, realiza dos cruzamientos de rL'Ueb~l dt' trcs puruos. E~ pfiuJ.ei' cruzrenlenro entre 1(l1ea~ p!Jm~ ~"'.

.It I. i ~l v(:~h~acJo.r cruza hi ego la I,' l~Ll1i L11l LJLU i viduo de prueba, homoci gl\licn TDJ::c~i V~). )' d~;-;~I'ica ] O~ dcsccnLlicIl.Lc~, ,egrin 1,1 contribueicn gaJt1cti ca de la F I:

A'f1-C'D'·E, 3J6

a-hoC-doE ,i\'B-C-j·f_ a b C·D·E A'h'C-d'E a-R'C-O'h' A,h'(_"IJ'I"; a [j'C d'!!

3.14 11 3Y 130 140 11

D I ~ )(Xl

AlA .. ErR· 0(' . DID - ElF x ala" Bill- - de - DID· elf!

III ,i.n ves~jJFld.lYr ~rULH luego I a J I de este cruzarn iel'lto em! un individue de prucba, h~trK.:.l.:ig&t~co reeesi-». y obticne

A'iJ'C'LY'E a'B-c·l)-e i\-B'C':D-e a ''&.("'l)- to' tt·B'(,'D'(:: {l'lJ,·c·D"E {1',8- c- n -I~ ..t l~' (: ' V . I~'

24J 237

62

5f': ];55 165 40 J4

~OOO

Por otro ladn, e I _gelnetil~t)l. ~lihe q~~e I (:I~ gen es D 'j E ~egrc· gan J c rnanera :l ndependiente.

a. l)ibllje el mAp.c! de esros genes, i ndicando ~ i m;pc~ih' i~:s distancias entre t·LI0~ en 'Llll.flJmle~ de mapa,

h. tHay al gun int.lfcitl de (Tll\.:dcmluill"!

.llil Atabidf)FIj~ trwtiamt. una ,;lLurcrf'?m '(jinlim]T,~, S5 hn i~ CIJ bieno UI] n U ev.o Icnoti po gl:aUcw ~carerHe de uicomasnlt,o sc hcrcda ('onw Uin curacicr reccsivo 'l.gff{). Sc oo~p;!clIrrq~ <':1 locua G est.'ii en el cromesoma 4. de manera que.se eru::! u~HI plauta Ji!g y otra d~ genotipo fly lye-Now, C01Qf!!Jl1~ 1,10 debide '" una rt·t1u~ci()I] cUlI.1 t:<lIUllli~J dlt c1uro~Jl.m S ahem os GUc ell oc UiR r es.ta en .d crornosoma 4. Al umt{; li.n.iz>'lt' lid'i uk: ftlflo[J:pu ~iheslJ:e,~ob~~et'le i8"'~guiehtef

sil yc~h't' 51 J,

arnaril lo ::3 '7

g]albro 234

amarillu. glabr» ] j

To'la~ lOW)

a.. ~·.EffU cl locus G en el cromosomu 4',1

h. S i bOll reapondl do af! rmarivameute a ~~~ anterier [R gUlHW, nLk:lIJe .. cuAIlLa~ !Jnid<\'i.des Je mapa esta del' !n~l' t

e, Explil.juc las FruCl.leL1l'·ill~ G'U q ue H~are.~'t:1l lase arnarsllo 'j glabJro de Ja F!.

36. 1;:'11 I a p IJJlt,i !1J'flbh:Jopsi~, I(lfi ~(')ci PaJ11 I a.longitud del ~[lJ. ~[., .largQ; t, cerro J 'j rll presencia de pelos en eSf~ ttl. fitJ 'pe'l tIUO: h. r[lHO pel ado) ~~Hlnl:ig3:do~1l1 l:1'1i8]110 CroJll0. Ina. se parados pnl- una di~l:a:[l;L:ia de 1.6 uniJuclt:s de 1lIll;!. S,e hkiemll los cruzamlentns ~1,gIriente~;

n ~ L tu: i1 x i hlf 11 -4 l (i~~ L fill. h x 1 Flil U .-TlF,

S,i se cruzan las dos F 1

a, (. Qu~ pmpfli"C i (in de Ia descende IlC i a xc csprm. qLl~'" r IdI h'r

b. f,QU~ proporcion tit: ~il ut$(;~nd,efKi~ ~e espera Cfllellil L hi{ h?

37, Rn e 1 III aJ:;o:, se ha obren ido un tri pie hetcrocigoio ~!!L~IJ:' los 011 ekili !1 tsece), w l a leu rona h'I'J.C)Ql) e y (eildw.~ eeroso), ctnparcjad us !.:~m~tI s respectlvos ,~Ijelo~ sih'eW\':

El tri pl'e hereroc ugolXl <;.of snm~l'c a. eruzumicntodc pJU~ ubten ie I1ldh~e unu descenden ciade 1.1 fi secos, hl.~l~Ic;!1'! ~ompLetntllotme sil vestres: 2533 secos: {lO I seC08. cer')'.!. {)26 b~hl.llml\,~; 27ns blanc: ~~. ccroscs; 2 ~l:()OS, blaJi;;Q3 • .:v ~O~ y ~ I:~ cernsos.

a. Derermlne ~i alguno de e~to~ tres loci c~tiiil l·ig.ID&Jl: si ll;~ esran, catcute las djst~nci<l~ de: mar.~,

b.. Mtle6ltt d orden de ~(T'S; alelos en IQ~ em IHOWI{lrilli: triple hetcrocigoto cmpkado en t.~ c:tLltamiemo de p$1I:'b;

c. Co lcule .I a interfere ncl a, si resuha apropi ado.

3!t IL~~ loci A y B e~r'~11 ligados, eSf.tuidfl ~epar,ad(l;i; pnf r eli stancin dt: :W un iJ.tllk~ de mapa, Demaes tre que-; rr C0m0 se rnuestra en el Cuarlm 5 -2, al cruzar entre ~{ill~i

rl~{J:" dihfbrid,~~ en fssede l'epuJs.iun. (.4. bta B), el valor z ~!td [f~ 0.001.

JlL a, Se uizo, con rl1~nne~, el cI11t:ami,e-nto Ala' BIb X eda _ Mb Y se obtuvo b descendeacia

:2-') % Ala ' Bib, 25 % aia _ h/h 25 % Ma . bib. 25 % ala ' Blh

bt,lhl!ue estes resLl.~tlldus:, ayudiind()~e de d!<1:gri!lthH~ SLIIJ,rlifun[(lo5 i!'t: Ia meiosis.

b. S~ hizo, con m({JHe~, d. cnL:eal'iniento eli: ' {lid x de . 1.:1111 'Ij' se obru \10 la desceudencia

4-5 % Cit ' .did, 45 % de ' Did 5 '{f de . did, :5 ct, C/c . DId

E~pllq1Je estes IT~uh,!Ki!o~, 1I.ylld~ ~(!(I~e de Jingtama~ sim.pLLncadtll., de la mej(l;i;~,

.w, En /\j.lw{d.OflJ'i~, una planta de porte p.eq1leiin, e.r alelo reee~~r'.'Q hY7 ennficre "[esls~enciJ de Ia seinilla a Ia droga higrn~Dr~ioa,.1j her, u~ aldo recesivo de UI] gell di8tiwilO, confiere .~sisrenQi,a de l~ semill H H herbicide, Se cnvz6 una planta .~!.1l1lfj;:::i.g0~ic<l hYX1hyg _ herlhu con e! sllvesue, y sc d{¥~ ~~e la ["J se :lJutupol1ni.1)lr8. Las se.twlfJJs .ru~l[harlllc~ de la .i!JL>~~ILnl1.:J.C'i611 de b 1-'1 se C()lo.e~LTOiIJl en placas de, perri }olU~ c~lIl~nfafl higrom icina ;' hC:L'1hidda_

i1; S.i ll,}g rJo~ .g~ne:~ no e,~.tl'in li,ga(j~~, ~qt:i6[DVrcentaji? de ~~[[Jil]l:b~ ~[wramo.s que g~[tnincn?

It, ~H[?tllttga ~]lJt: han gcrmi n;ldol,m 13 ti}, de [::IS semi U;,j8, , hi PQy~ cqe pm-.centaje la hipctesis de auscncia de tigamK:~lI:J7 hp~rq!l.clo. Si nO Io aroya. calcule cuamR~ u~kladJ:;l..~f"inilpa separanlos des loci,

~, .~ Igw~emJo su hipntesi~. S i se semete 1.'11 l" i .a cruzamien- 1),1 de prudm, Gy'I]G p:mlJ'Ord6n de las semillas gl:Tlnlnani en el 11R:\.lio que conrlene hlgl'omicllHl y hcrbl eida?

~1. En ehnatz O{;e(l itnaj!s), ('I map~~ g:e:I'IElrIcO del Cl'omOW!I[LI:I 4 ~wl }' roam ~l' represenra ~baj(i>, doude w, s y ercplrc.">el~~ [nil ilkkmmces.iv(lf; !1lt!!ltnme~ que ~fl:.CI:1m al Do~or y a la IUl]o" 061 I1nl~e:n.

~i S~' rea Ii?;] el eruzaruicnto siguienre

)' ~ SIJJ]J~!.C 111. F! a un crozernienro.de p'nl.~ba con w .• et~! ,~ Mmp.~mk"f1d{lq!le FKI hay interfercneia en est.a regidn del l~l¥,sptl'Ia, lAIn:: pr{]rorck~nd.e. I a, descendeneia Sera de g~wtiJ,-'II)

A" + + +i
I), 1.1, 8 a
~" f Ni"! d. 'It' + -:
e. +. + P."f
r. H.' s +'i
g. w +t{!
b. 4- s +? ·a-::t t::~l e~ mosquito, Anopheles, Ies !o~l j)8[1l el color (,k' los l)jns fale~o~ Fl, negI'D; f,. blanco) ':! 1.<1. l.onglluJ del cnc:rpo (L. I argo; t,W'flO ~ G~liin. li t'!adn~,ex istl endo entre e llos u na distancja de 1{) m_LI. Se haceun ctuzamienso i:!-lltre. dos dihfbridoa, u no en ft:! se de aC(lp1<lrn~~nW y cl OLIll en ] a de repuJsi6n, ~:Qr~e.feIl!Ollpf)~ sc oblklndir'J.n. 6nbd~f),e:nct,~:ncia y en tlllf' propcrctones?

43.. tlIl,1 plaeta dcgcnotipo Qlq . lUI"" Til produce los genotipos garm':tLCQI,; sigui entes, ell b~ proporclones (1 uc se indican

Q R 7' i
~
Q l? f
q R T I
~
Q r T 1
'B'
q r ! '1
i
Q r f I
,~
q N ~' .I
~
q r T J
~ 44 ..

D i huj e ell agnullCls dt la meiosisenlos que se ex pi i que: eDI1 cJal'idau {'urnu sc obtienen estas lrmpnl'dfmes ga.n€~k~I~, COIn!) parte de _~ll respuesta, mn~~~n:: I()~ t:rum.o~Qma~ i' I) anses de 1<1 rase prt:lrtt:i(jili~a. S (fase de replicaclorr); (2) dJesi;~u6 de la Iormacidn deb~ crom ati das: (3) CUJJ'!do estrtn etnparcjades; (4) durante la a.tl<lf~ se I; y (."i} durante Is. anatase IT.

En. las judIas, alto (1) domina sobrecorto (tJ." 11c~IrC~ rnj~'ls tm clollni na sobre flares blancas (n)' "oja.~ .anchas (W) dorniua sobre hojil~ estrechas (l,,.t Se hi7" el cruzamienro .~fguienre. obterriendosc ] a descendencla ql i<', Sf, nnle~~ ra:

alto, roja, ancha x cnrto, blaeca, esrrecha 478 11] to. hi anca, .mch 8

21 aloo,mj;"l, ~liW:M

19 corte. blanca. lmdl1l

4<'::::2 eorto I'l)ja, anch a

~. Expliq lie per que sc obtuvieron e;~tJ:lll' fell Cltifl"ls en I a, descendcneia y elm las proporeionea' ()tt&er"'acla.~ reuumere to.umlm genotipos y muestre las pl}&ici(}i1t'~ ell:' 101; ger.I!I::~

en lo~ .cmmoSO~l<18}.

b. Slg~uendo. su hipeitcsi«. Si se deja anropohnlzar el [,'1'<11- rcrual alto, mja, ancha, i',en,11 sel'A la propon'i.on de Ia desc~H1dendaQ('trE(l, blanca, ancha?

45. Ell el pe(li~g:rI de I~ FugHm. 5~J9" calcule la fI'l,mtuaci(ih Lod paru una frecuencia de reeomi-li ila.6on dd ~J4, %.

46. EL1. till organl:> mn dl ploi,de de genolipu Aia ; Bib ~ Did, los gell1,e~ esran en P.~IIT j a~ J!et;ruH'l~)l'mm~ls hmmllog(~~ diferenL~~, Los d:i"';grama~ slguieutes preeenden urostrnr asafases

~fu~e dot. SeP1lrnci.6~ de los CJ'(lr'l.1L)SOIm~S 0 cromrt~.id.8.s) en tf11l18.5 l.ndi".idtIH]eS. Una linea represcneaun cromosema.otina cromatida, y cl puntn blanco ill dica Ia p(l~ uci6n del

a,
.,s ,~ s-
;j J
,;, D
",
iii d
b. ,s ,~ ~,
A Q
.;;1: ~"
A
c,
,S , d
~,
,4-
,~ ~ ~;
"
8 _ .b D
.d.
,~ ~;,
IJ o
,~ .~ ~
'1$ .. ,
a d _
a, /'\ '{

ceniremero .. Diga, pam cada genotipo concreto. ~i el dib" represcuta I'll. mitnsis. 111 meiosis I, la Jtldos]S' fJ n ~i es m D()~ i hie determl mll'II~_

f.

/"X V'(

~.

/'\ ,I:

/'-' ,~

~, ~I

/\A V.

i.

,/,</'''/~/~~'A ,~,t:V<'~}/Y

/~~/"'/"'-A:;' '~'~';{,'4-;;",1}/

Вам также может понравиться